04.01.2015 Views

ZADANIA Z ARYTMETYKI SZKOLNEJ I TEORII LICZB

ZADANIA Z ARYTMETYKI SZKOLNEJ I TEORII LICZB

ZADANIA Z ARYTMETYKI SZKOLNEJ I TEORII LICZB

SHOW MORE
SHOW LESS

Create successful ePaper yourself

Turn your PDF publications into a flip-book with our unique Google optimized e-Paper software.

U N I W E R S Y T E T<br />

S Z C Z E C I Ń S K I<br />

GRZEGORZ SZKIBIEL, CZES̷LAW WOWK<br />

<strong>ZADANIA</strong><br />

Z <strong>ARYTMETYKI</strong> <strong>SZKOLNEJ</strong><br />

I <strong>TEORII</strong> <strong>LICZB</strong><br />

SZCZECIN 1999


SPIS TREŚCI<br />

Przedmowa . . . . . . . . . . . . . . . . . . . . . . . . . . . . . . . . . . . . . . . . . . . . . . . . . . .5<br />

Cze ↩<br />

ść I – Zadania . . . . . . . . . . . . . . . . . . . . . . . . . . . . . . . . . . . . . . . . . . . 7<br />

Cze ↩<br />

ść II – Rozwia ↩<br />

zania. . . . . . . . . . . . . . . . . . . . . . . . . . . . . . . . . . . .51<br />

1. Podstawowe w̷lasności liczb ca̷lkowitych . . . . . . . 7 51<br />

1.1. Podzielność liczb ca̷lkowitych. . . . . . . . . . . . . . . . . . . . . .7 51<br />

1.2. Zasada indukcji matematycznej . . . . . . . . . . . . . . . . . . . 8 53<br />

1.3. Dzielenie z reszta ↩<br />

. . . . . . . . . . . . . . . . . . . . . . . . . . . . . . . . 10 55<br />

1.4. Cze ↩<br />

ść ca̷lkowita . . . . . . . . . . . . . . . . . . . . . . . . . . . . . . . . . . 11 56<br />

1.5. Dzielenie z reszta ↩<br />

– dalsze w̷lasności. . . . . . . . . . . . . .12 59<br />

1.6. Najwie ↩<br />

kszy wspólny dzielnik . . . . . . . . . . . . . . . . . . . . . 13 61<br />

1.7. Najmniejsza wspólna wielokrotność . . . . . . . . . . . . . . 15 62<br />

1.8. Zasadnicze twierdzenie arytmetyki . . . . . . . . . . . . . . . 16 64<br />

2. Liczby pierwsze . . . . . . . . . . . . . . . . . . . . . . . . . . . . . . . . . . 17 66<br />

2.1. Poje ↩<br />

cie liczby pierwszej . . . . . . . . . . . . . . . . . . . . . . . . . . 17 66<br />

2.2. Ile jest liczb pierwszych . . . . . . . . . . . . . . . . . . . . . . . . . 17 67<br />

2.3. Wnioski z zasadniczego twierdzenia arytmetyki . . . 18 68


4<br />

2.4. Uwagi o funkcji π(x) . . . . . . . . . . . . . . . . . . . . . . . . . . . . 21 70<br />

2.5. Twierdzenie Dirichleta . . . . . . . . . . . . . . . . . . . . . . . . . . . 22 71<br />

2.6. Liczba dzielników oraz funkcja Eulera . . . . . . . . . . . . 23 73<br />

2.7. Rozk̷lad na czynniki dużych liczb naturalnych . . . . 24 75<br />

3. Liczby w różnych systemach pozycyjnych . . . . 26 77<br />

3.1. Poje ↩<br />

cie pozycyjnego systemu zapisu liczb. . . . . . . . .26 77<br />

3.2. Wykonywanie obliczeń w różnych systemach<br />

pozycyjnych . . . . . . . . . . . . . . . . . . . . . . . . . . . . . . . . . . . . . 28 82<br />

3.3. U̷lamki w różnych systemach pozycyjnych . . . . . . . . 30 85<br />

4. Algorytm Euklidesa . . . . . . . . . . . . . . . . . . . . . . . . . . . . . 34 89<br />

4.1. Szukanie NWD . . . . . . . . . . . . . . . . . . . . . . . . . . . . . . . . . . 34 89<br />

4.2. Równania liniowe . . . . . . . . . . . . . . . . . . . . . . . . . . . . . . . . 36 93<br />

4.3. Rozwia ↩<br />

zywanie równań liniowych . . . . . . . . . . . . . . . . 37 94<br />

5. Kongruencje . . . . . . . . . . . . . . . . . . . . . . . . . . . . . . . . . . . . . . 39 98<br />

5.1. Podstawowe w̷lasności kongruencji . . . . . . . . . . . . . . . 39 98<br />

5.2. Kongruencje a wielomiany . . . . . . . . . . . . . . . . . . . . . . . 40 100<br />

5.3. Kongruencje a równania . . . . . . . . . . . . . . . . . . . . . . . . . 41 102<br />

5.4. Ma̷le Twierdzenie Fermata . . . . . . . . . . . . . . . . . . . . . . . 42 103<br />

5.5. Pewne zastosowania twierdzenia Eulera . . . . . . . . . . 44 105<br />

5.6. Rozwinie ↩<br />

cie okresowe a kongruencje . . . . . . . . . . . . . . 46 108<br />

5.7. Zastosowania twierdzenia Wilsona . . . . . . . . . . . . . . . 48 109<br />

5.8. Jeszcze jedno twierdzenie o kongruencjach. . . . . . . .49 110<br />

Bibliografia . . . . . . . . . . . . . . . . . . . . . . . . . . . . . . . . . . . . . . . . . . . . . . . . 112


PRZEDMOWA<br />

Jeden z wybitnych matematyków naszego stulecia, G.H. Hardy<br />

powiedzia̷l:<br />

Elementarna teoria liczb powinna być uważana za jeden<br />

z najw̷laściwszych przedmiotów w pocza ↩<br />

tkach wykszta̷lcenia<br />

matematycznego. Wymaga ona bardzo ma̷lo uprzedniej wiedzy,<br />

a przedmiot jej jest uchwytny i znajomy. Metody, które stosuje,<br />

sa ↩<br />

proste, ogólne i nieliczne, i nie ma sobie równej wśród<br />

nauk matematycznych w odwo̷laniu sie ↩<br />

do naturalnej ludzkiej<br />

ciekawości 1 .<br />

Mamy nadzieje ↩<br />

, że umieszczenie w programie zawodowych<br />

studiów matematycznych takich przedmiotów jak arytmetyka szkolna<br />

i arytmetyka be ↩<br />

dzie okazja ↩<br />

do zilustrowania s̷lów G.H. Hardy’ego.<br />

Zadania z arytmetyki szkolnej i teorii liczb to skrypt adresowany<br />

w pierwszym rze ↩<br />

dzie do studentów studiów zawodowych,<br />

którzy zetkna ↩<br />

sie ↩<br />

z teoria ↩<br />

liczb w ramach przedmiotów arytmetyka<br />

szkolna i arytmetyka. Ponieważ w skrypcie umieściliśmy wiele<br />

zadań z olimpiad matematycznych, wie ↩<br />

c może on być przydatny<br />

także uczniom o zainteresowaniach matematycznych. Polecamy nasz<br />

skrypt również studentom starszych lat studiów matematycznych,<br />

zainteresowanym wyk̷ladem z kryptografii, ponieważ nie da sie ↩<br />

studiować<br />

tego przedmiotu bez znajomości elementarnej teorii liczb.<br />

Aby na wyk̷ladzie z kryptografii szybciej przejść do realizacji zasadniczych<br />

hase̷l, pewne zagadnienia z teorii liczb be ↩<br />

dzie można po-<br />

1 G.H. Hardy: A Mathematician’s Apology, 1940


6 Przedmowa<br />

zostawić s̷luchaczom do samodzielnego przeczytania w niniejszym<br />

skrypcie.<br />

Skrypt sk̷lada sie ↩<br />

z dwóch cze ↩<br />

ści. W pierwszej cze ↩<br />

ści znajduja ↩<br />

sie ↩<br />

najważniejsze twierdzenia przypadaja ↩<br />

ce na dany rozdzia̷l oraz<br />

zadania do rozwia ↩<br />

zania. Natomiast w drugiej cze ↩<br />

ści umieściliśmy<br />

szczegó̷lowe rozwia ↩<br />

zania. Ponieważ w nauce matematyki istotna ↩<br />

rzecza ↩<br />

jest umieje ↩<br />

tność rozwia ↩<br />

zywania różnych problemów, przeto<br />

rozwia ↩<br />

zania zadań (ze skryptu) należy czytać dopiero po wielu<br />

samodzielnych próbach wykonania zadania. Cze ↩<br />

ść zadań zamieszczonych<br />

w skrypcie jest pomys̷lu autorów, ale znaczna cze ↩<br />

ść zosta̷la<br />

zaczerpnie ↩<br />

ta z literatury, której spis znajduje sie ↩<br />

na końcu skryptu.<br />

Zadania u̷lożone sa ↩<br />

w kolejności od ̷latwiejszych do trudniejszych,<br />

ale najtrudniejsze zadania niekoniecznie znajduja ↩<br />

sie ↩<br />

na końcu paragrafu.<br />

Pomine ↩<br />

liśmy też stosowane cze ↩<br />

sto w literaturze oznaczenie *<br />

dla zadań trudniejszych, ponieważ wydaje sie ↩<br />

nam, że istnieje spora<br />

grupa Czytelników, którzy znieche ↩<br />

caja ↩<br />

sie ↩<br />

do pracy nad problemem<br />

z ,,gwiazdka ↩<br />

”.<br />

W skrypcie obowia ↩<br />

zuje powszechnie stosowana symbolika. Jest<br />

ona wyjaśniana w pocza ↩<br />

tkowych fragmentach odpowiednich paragrafów<br />

oraz w tekstach niektórych zadań.


CZE ↩ ŚĆ I – <strong>ZADANIA</strong><br />

1. Podstawowe w̷lasności liczb ca̷lkowitych<br />

1.1. Podzielność liczb ca̷lkowitych. Mówimy, że liczba<br />

ca̷lkowita m ≠ 0 dzieli liczbe ↩<br />

ca̷lkowita ↩<br />

a , jeżeli istnieje taka liczba<br />

ca̷lkowita n , że m · n = a . Fakt ten zapisujemy m|a . Na przyk̷lad<br />

3|276 , bo 276 = 3 · 92 . Jeśli liczba m nie dzieli a , co oznacza,<br />

że nie istnieje żadna liczba ca̷lkowita n , dla której mn = a , to<br />

piszemy m̸ | a . Jeżeli m|a , to mówimy też, że m jest dzielnikiem<br />

liczby a , natomiast liczbe ↩<br />

a nazywamy wielokrotnościa ↩<br />

liczby m .<br />

1.1.1. Rozstrzygnij, czy 5|12354 , czy 5|12345 .<br />

1.1.2. Pokaż, że jeśli m|a , to m|(−a) .<br />

1.1.3. Uzasadnij, że jeśli m|a oraz b jest dowolna ↩<br />

liczba ↩<br />

ca̷lkowita<br />

↩<br />

, to m|ab .<br />

1.1.4. Wiadomo, że 15|225 . Rozstrzygnij, czy 15|675 oraz<br />

czy 15|5775 .<br />

1.1.5. Za̷lóżmy, że m|ab dla pewnych liczb ca̷lkowitych m , a<br />

i b . Czy m musi wtedy dzielić a lub b <br />

1.1.6. Pokaż, że jeżeli m|a oraz m|b , to m|a + b i m|a − b .<br />

1.1.7. Wiadomo, że 14|784 . Pokaż, że 14|770 oraz że 14|812 .


8 Cze ↩<br />

ść I – Zadania<br />

1.1.8. Wiadomo, że 14|784 . Czy 7|784 A czy 7|817 <br />

1.1.9. Zgadnij, czy 14|790 , a naste ↩<br />

pnie sprawdź swoja ↩<br />

odpowiedź biora ↩<br />

c pod uwage ↩<br />

poprzednie zadanie.<br />

1.1.10. Wiadomo, że 56|2576 . Jaka jest naste ↩<br />

pna (po 2576)<br />

liczba podzielna przez 56<br />

1.1.11. Wiemy, że 7|315 . Wypisz wszystkie liczby wie ↩<br />

ksze od<br />

290 i mniejsze od 340, które sa ↩<br />

podzielne przez 7.<br />

1.1.12. Za̷lóżmy, że m|a + b . Czy oznacza to, że m|a i m|b <br />

A może oznacza to, że m|a lub m|b <br />

1.1.13. Za̷lóżmy, że m|a + b oraz m|a − b . Czy wtedy m|a i<br />

m|b Jeżeli nie, to czy potrafisz sformu̷lować dodatkowe za̷lożenia<br />

o m tak, by naste ↩<br />

puja ↩<br />

ce zdanie by̷lo prawdziwe.<br />

Jeśli m|a + b i m|a − b , to m|a i m|b .<br />

1.1.14. Pokaż, że jeśli m|a oraz n|m , to n|a .<br />

1.1.15. Pokaż, że jeżeli m|a i a ≠ 0 , to |m| ≤ |a| .<br />

1.1.16. Pokaż, że jeżeli m|a i a|m , to m = a lub m = −a .<br />

1.2. Zasada indukcji matematycznej. Podczas nauki<br />

matematyki w szkole średniej cze ↩<br />

sto korzystaliśmy z tak zwanej zasady<br />

indukcji matematycznej (ZIM). Przypomnijmy sformu̷lowanie<br />

tej zasady:<br />

Niech T (n) be ↩<br />

dzie zdaniem dotycza ↩<br />

cym liczby naturalnej n . Jeżeli<br />

1 0 T (m 0 ) jest zdaniem prawdziwym, gdzie m 0 jest pewna ↩<br />

liczba ↩<br />

należa ↩<br />

ca ↩<br />

do N 0 ;<br />

2 0 z prawdziwości zdania T (k) (gdzie k ∈ N 0 , k ≥ m 0 ) wynika<br />

prawdziwość zdania T (k + 1) ;<br />

to zdanie T (n) jest prawdziwe dla każdego n ≥ m 0 .<br />

Przez N 0 oznaczyliśmy tu zbiór liczb ca̷lkowitych nieujemnych,<br />

czyli zbiór {0,1,2,. . .} . Podobnie, przez N m oznaczymy zbiór<br />

wszystkich liczb ca̷lkowitych wie ↩<br />

kszych lub równych m , tj. zbiór


Podstawowe w̷lasności liczb ca̷lkowitych 9<br />

{m,m + 1,m + 2,. . .} . Stosuja ↩<br />

c powyższe oznaczenia możemy nadać<br />

zasadzie indukcji matematycznej naste ↩<br />

puja ↩<br />

ca ↩<br />

postać:<br />

Niech M ⊂ N 0 be ↩<br />

dzie zbiorem takim, że<br />

1 0 m 0 ∈ M ;<br />

2 0 dla dowolnego k ∈ N m0 , jeśli k ∈ M , to k + 1 ∈ M .<br />

Wówczas N m0 ⊂ M .<br />

Zasada indukcji matematycznej jest równoważna zasadzie minimum<br />

(ZM), która orzeka, że w każdym niepustym zbiorze A liczb<br />

ca̷lkowitych nieujemnnych istnieje liczba najmniejsza. Wykażemy<br />

teraz, że z ZIM wynika ZM 1 .<br />

Przypuśćmy, że A jest niepustym zbiorem liczb ca̷lkowitych<br />

nieujemnych, w którym nie ma liczby najmniejszej. Niech B<br />

be ↩<br />

dzie zbiorem liczb ca̷lkowitych nieujemnych zdefiniowanym w<br />

naste ↩<br />

puja ↩<br />

cy sposób: n ∈ B ⇐⇒ dla każdej liczby ca̷lkowitej<br />

nieujemnej m , jeżeli m ≤ n , to m /∈ A . Zauważmy, że 0 /∈ A , bo<br />

w przeciwnym wypadku w zbiorze A istnia̷laby liczba najmniejsza,<br />

która ↩<br />

by̷loby 0 . Zatem 0 ∈ B .<br />

Za̷lóżmy, że n ∈ B . Wtedy n + 1 /∈ A , gdyż w przeciwnym<br />

razie n + 1 by̷loby najmniejsza ↩<br />

liczba ↩<br />

zbioru A . Wynika to sta ↩<br />

d,<br />

że skoro n ∈ B , wie ↩<br />

c z definicji zbioru B mamy<br />

n /∈ A, n − 1 /∈ A, . . . , 0 /∈ A.<br />

Zatem w konsekwencji n + 1 ∈ B .<br />

Wykazaliśmy, że zbiór B spe̷lnia za̷lożenia ZIM (w drugim<br />

sformu̷lowaniu), wie ↩<br />

c B = N 0 .<br />

Biora ↩<br />

c pod uwage ↩<br />

definicje ↩<br />

zbioru B , wnioskujemy, że A jest<br />

zbiorem pustym, co przeczy naszemu za̷lożeniu.<br />

1.2.1. Udowodnij, że z zasady minimum wynika zasada indukcji<br />

matematycznej.<br />

1.2.2. Korzystaja ↩<br />

c z zasady indukcji uzasadnij, że 3|n 3 + 5n<br />

dla dowolnego n ∈ N 0 .<br />

1 Przy pierwszym czytaniu Czytelnik może pomina↩ ć to uzasadnienie.


10 Cze ↩<br />

ść I – Zadania<br />

1.2.3. Wykaż, że 7 jest ostatnia ↩<br />

cyfra ↩<br />

liczby 2 2n + 1 , gdy n ∈<br />

N 2 (liczby 2 2n + 1 , gdzie n ∈ N 0 nazywamy liczbami Fermata).<br />

1.2.4. Uzasadnij, że 10|2 2n − 6 dla n ∈ N 2 .<br />

1.2.5. Wykaż, że 1 jest ostatnia ↩<br />

cyfra ↩<br />

liczby 2 4n −5 dla n ∈ N<br />

( = N 1 ).<br />

1.3. Dzielenie z reszta ↩<br />

. Jak wiadomo, jeśli mamy ustalona ↩<br />

liczbe ↩<br />

ca̷lkowita ↩<br />

m , to nie każda liczba ca̷lkowita dzieli sie ↩<br />

przez m .<br />

Na przyk̷lad 34 nie dzieli sie ↩<br />

przez 5, ponieważ nie ma takiej liczby<br />

ca̷lkowitej, która pomnożona przez 5 da iloczyn równy 34. Oznacza<br />

to, że gdybyśmy chcieli rozdzielić 34 zeszyty mie ↩<br />

dzy pie ↩<br />

ciu uczniów,<br />

tak aby każdy otrzyma̷l jednakowa ↩<br />

ilość, to nie potrafilibyśmy tego<br />

dokonać. Możemy jednakże dać każdemu uczniowi po 6 zeszytów<br />

i pozostana ↩<br />

nam jeszcze 4. Dziela ↩<br />

c 34 przez 5 otrzymujemy zatem<br />

6 oraz reszte ↩<br />

4. Fakt ten zapisujemy 34 = 5 · 6 + 4 .<br />

Przypuśćmy, że mamy dwie liczby ca̷lkowite n oraz d , przy<br />

czym d ≠ 0 . Dzielenie (z reszta ↩<br />

) liczby n przez d polega na<br />

znalezieniu liczb ca̷lkowitych q oraz r takich, że n = qd + r oraz<br />

0 ≤ r < |d| . Liczbe ↩<br />

r nazywamy reszta ↩<br />

z dzielenia n przez d ,<br />

a liczbe ↩<br />

q niepe̷lnym ilorazem lub ilorazem cze ↩<br />

ściowym tego dzielenia.<br />

Oczywiste jest, że d|n wtedy i tylko wtedy, gdy r = 0 .<br />

1.3.1. Znajdź niepe̷lny iloraz i reszte ↩<br />

z dzielenia<br />

(a) 23 przez 3;<br />

(b) 43 przez 4;<br />

(c) 36 przez 12.<br />

1.3.2. Niech n i d be ↩<br />

da ↩<br />

liczbami ca̷lkowitymi, przy czym<br />

d ≥ 1 . Korzystaja ↩<br />

c z zasady minimum wykaż, że istnieje dok̷ladnie<br />

jedna para liczb ca̷lkowitych q i r taka, że n = dq + r , gdzie<br />

0 ≤ r < d .<br />

1.3.3. Pokaż, że kwadrat liczby ca̷lkowitej nieparzystej przy<br />

dzieleniu przez 8 daje reszte ↩<br />

1.<br />

1.3.4. Pokaż, że suma kwadratów dwóch kolejnych liczb naturalnych<br />

przy dzieleniu przez 4 daje reszte ↩<br />

1.


Podstawowe w̷lasności liczb ca̷lkowitych 11<br />

1.3.5. Udowodnij, że liczba naturalna postaci 3m + 2 ( m ∈<br />

N ) nie jest kwadratem żadnej liczby ca̷lkowitej.<br />

1.3.6. Uzasadnij, że suma kwadratów dwóch liczb nieparzystych<br />

nie jest kwadratem żadnej liczby ca̷lkowitej.<br />

1.4. Cze ↩<br />

ść ca̷lkowita. Jeżeli x jest dowolna ↩<br />

liczba ↩<br />

rzeczywista ↩<br />

, to istnieje najwie ↩<br />

ksza liczba ca̷lkowita n spe̷lniaja ↩<br />

ca<br />

warunek n ≤ x . Liczbe ↩<br />

n nazywamy cze ↩<br />

ścia ↩<br />

ca̷lkowita ↩<br />

liczby<br />

x i oznaczamy symbolem [x] lub E(x) . Z określenia liczby [x]<br />

wynika, że<br />

[x] ≤ x < [x] + 1.<br />

(∗)<br />

Istotnie, gdyby x ≥ [x]+1 , to liczba [x]+1 by̷laby liczba ↩<br />

ca̷lkowita ↩<br />

wie ↩<br />

ksza ↩<br />

od [x] spe̷lniaja ↩<br />

ca ↩<br />

warunek [x] + 1 ≤ x . Jest to sprzeczne<br />

z definicja ↩<br />

cze ↩<br />

ści ca̷lkowitej liczby x .<br />

Z nierówności (*) wynika, że 0 ≤ x − [x] < 1 . Liczbe ↩<br />

x − [x]<br />

nazywamy cze ↩<br />

ścia ↩<br />

u̷lamkowa ↩<br />

liczby x i oznaczamy symbolem {x} .<br />

̷Latwo zobaczyć, że x = [x] + {x} .<br />

Przyk̷lady: [− 1 2 ] = −1 , [4,7] = 4 , [−7,3] = −8 , { − 2} 1 =<br />

1<br />

2 ,<br />

{4,7} = 0,7 , {−7,3} = 0,7 .<br />

1.4.1. Wykaż, że jeżeli x , y ∈ R , oraz x ≤ y , to [x] ≤ [y] .<br />

1.4.2. Uzasadnij, że jeżeli α ∈ (0,1) oraz n ∈ N , to [−n +<br />

α] = −n .<br />

1.4.3. Uzasadnij, że jeżeli<br />

(a) x jest liczba ↩<br />

ca̷lkowita ↩<br />

, to [−x] = −[x] ;<br />

(b) x nie jest liczba ↩<br />

ca̷lkowita ↩<br />

, to [−x] = −[x] − 1 ;<br />

(c) x ∈ R , n ∈ Z , to [x + n] = [x] + n.<br />

1.4.4. Wykaż, że dla dowolnych liczb rzeczywistych x oraz y<br />

zachodzi nierówność<br />

[x + y] ≥ [x] + [y].<br />

1.4.5. Udowodnij, że jeżeli [x] = [y] , to |x − y| < 1 .


12 Cze ↩<br />

ść I – Zadania<br />

1.4.6. Wykaż, że jeśli n jest liczba ↩<br />

naturalna ↩<br />

, a x liczba ↩<br />

rzeczywista ↩<br />

, to<br />

[ ] [x]<br />

[ x<br />

= .<br />

n n]<br />

1.4.7. Rozwia ↩<br />

ż równanie<br />

5x + 4<br />

7<br />

[ ] 2x + 3<br />

= .<br />

5<br />

1.4.8. Uzasadnij, że dla dowolnej liczby rzeczywistej x i dowolnej<br />

liczby naturalnej n zachodzi równość<br />

[x] +<br />

[<br />

x + 1 ] [<br />

+ x + 2 ] [<br />

+ · · · + x + n − 1 ]<br />

= [nx].<br />

n n<br />

n<br />

1.4.9. Niech n i k be ↩<br />

da ↩<br />

liczbami naturalnymi. Wykaż, że<br />

[ n<br />

] [ ] [ ] [ ]<br />

n + 1 n + 2 n + k − 1<br />

+ + + · · · +<br />

= n.<br />

k k k<br />

k<br />

1.5. Dzielenie z reszta ↩<br />

– dalsze w̷lasności. Dziela ↩<br />

c<br />

34 przez 5 otrzymujemy 6 i reszte ↩<br />

. Zauważmy, że 6 = [ ]<br />

34<br />

5 .<br />

Wykorzystuja ↩<br />

c w̷lasności cze ↩<br />

ści ca̷lkowitej można podać algorytm<br />

dzielenia (z reszta ↩<br />

) liczby ca̷lkowitej m przez liczbe ↩<br />

ca̷lkowita ↩<br />

n > 0 . K̷lada ↩<br />

c<br />

[ m<br />

]<br />

q =<br />

n<br />

oraz<br />

r = m − nq<br />

mamy q ≤ m n < q + 1 . Sta ↩d qn ≤ m < qn + n . Zatem 0 ≤ r =<br />

m − qn < n .<br />

1.5.1. Podziel z reszta ↩<br />

(pamie ↩<br />

taja ↩<br />

c, że reszta z dzielenia ma<br />

być liczba ↩<br />

nieujemna ↩<br />

)<br />

(a) 83 przez −3 ;<br />

(b) −71 przez −4 .


Podstawowe w̷lasności liczb ca̷lkowitych 13<br />

1.5.2. Rozwia ↩<br />

zuja ↩<br />

c zadanie 1.5.1, można zauważyć, że algorytmu<br />

podanego we wste ↩<br />

pie nie można zastosować, jeśli n < 0 .<br />

Zmodyfikuj ten algorytm tak, aby można by̷lo go zastosować przy<br />

dzieleniu liczby ca̷lkowitej m przez liczbe ↩<br />

ca̷lkowita ↩<br />

n < 0 .<br />

1.5.3. Uzasadnij, [ że ] jeżeli p , n ∈ N , to wśród wyrazów cia ↩<br />

gu<br />

1 , 2 , 3 , . . . , n jest wielokrotności liczby p .<br />

n<br />

p<br />

1.5.4. Znajdź najmniejsza ↩<br />

liczbe ↩<br />

naturalna ↩<br />

n spe̷lniaja ↩<br />

ca ↩<br />

wszystkie poniższe warunki:<br />

– reszta z dzielenia n przez 2 jest równa 1,<br />

– reszta z dzielenia n przez 3 jest równa 2,<br />

– reszta z dzielenia n przez 4 jest równa 3,<br />

– reszta z dzielenia n przez 5 jest równa 4.<br />

1.6. Najwie ↩<br />

kszy wspólny dzielnik. Niech m i n be ↩<br />

da ↩<br />

dwiema liczbami ca̷lkowitymi, przy czym m ≠ 0 lub n ≠ 0 . Liczbe ↩<br />

ca̷lkowita ↩<br />

d ≥ 1 nazywamy najwie ↩<br />

kszym wspólnym dzielnikiem<br />

liczb m i n (co oznaczamy NWD(m, n) ), jeśli<br />

(a) d dzieli m i d dzieli n;<br />

(b) jeżeli liczba ca̷lkowita c dzieli m oraz n , to c dzieli d .<br />

Bezpośrednio z powyższej definicji wynika, że jeżeli m ≠ 0 , to<br />

wtedy NWD(m, 0) = |m| . Weźmy teraz dwie liczby ca̷lkowite m ≠ 0<br />

oraz n ≠ 0 . Uzasadnimy, że z zasady minimum wynika istnienie<br />

NWD(m, n) . W tym celu rozważmy zbiór<br />

X = {xm + yn ≥ 1 : x,y ∈ Z} .<br />

Ponieważ m·m+n·n ≥ 1 , wie ↩<br />

c X ≠ ∅ . Z zasady minimum wynika,<br />

że w zbiorze X istnieje liczba najmniejsza. Niech d = am + bn<br />

be ↩<br />

dzie ta ↩<br />

liczba ↩<br />

( a , b ∈ Z ). Zauważmy, że d|m oraz d|n . Istotnie,<br />

jeśli zapiszemy m = dq + r , gdzie 0 ≤ r < d , wtedy mamy<br />

r = m − dq = m − (am + bn)q = m(1 − aq) + n(−bq).<br />

Gdyby r ≥ 1 , to r ∈ X oraz r < d , co jest sprzeczne z wyborem d .<br />

Zatem r = 0 , czyli d|m . Analogicznie uzasadniamy, że d|n . Tak<br />

wie ↩<br />

c d spe̷lnia warunek (a) definicji NWD.


14 Cze ↩<br />

ść I – Zadania<br />

Aby pokazać, że warunek (b) także jest spe̷lniony, za̷lóżmy,<br />

że pewna liczba c dzieli zarówno m , jak i n . Sta ↩<br />

d wynika, że<br />

istnieja ↩<br />

takie liczby ca̷lkowite m 1 oraz n 1 , że m = cm 1 i n = cn 1 .<br />

Sta ↩<br />

d d = am + bn = c(am 1 + bn 1 ) , czyli c|d . Wykazaliśmy, że<br />

d = NWD(m, n) .


Podstawowe w̷lasności liczb ca̷lkowitych 15<br />

Z powyższych rozważań otrzymujemy naste ↩<br />

puja ↩<br />

cy<br />

Wniosek. Jeżeli NWD(m, n) = d , to istnieja ↩<br />

liczby ca̷lkowite x i y<br />

takie, że mx + ny = d .<br />

Uwaga. Zauważmy, że jeśli dla liczb m , n i d znajdziemy takie x<br />

i y , że mx + ny = d , to nie znaczy to jeszcze, że d = NWD(m, n) .<br />

Na przyk̷lad, dla m = 1 , n = 2 oraz d = 7 mamy 3m + 2n = d ,<br />

ale 7 nie jest oczywiście najwie ↩<br />

kszym wspólnym dzielnikiem liczb<br />

1 i 2.<br />

Jeżeli NWD(m, n) = 1 , to mówimy, że liczby m i n sa ↩<br />

wzgle ↩<br />

dnie pierwsze.<br />

1.6.1. Pokaż, że jeżeli m|n , to NWD(m, n) = |m| .<br />

1.6.2. Pokaż, że jeżeli m = nq+r , to NWD(m, n) = NWD(n, r) .<br />

1.6.3. Znajdź NWD(98, 56) .<br />

1.6.4. Pokaż, że NWD(n, n + 1) = 1 .<br />

1.6.5. Pokaż, że dla dowolnego k ∈ N zachodzi<br />

NWD(2k + 1, 2k + 3) = 1.<br />

1.6.6. Niech d = NWD(m, n) i niech m = dm 1 oraz n = dn 1 .<br />

Pokaż, że NWD(m 1 , n 1 ) = 1 .<br />

1.6.7. Za̷lóżmy, że u̷lamek a b<br />

jest nieskracalny. Sprawdź, czy<br />

u̷lamek jest nieskracalny.<br />

a<br />

a+b<br />

1.6.8. Pokaż, że jeżeli liczby m i n sa ↩<br />

wzgle ↩<br />

dnie pierwsze<br />

oraz m|nk , to m|k .<br />

1.6.9. Za̷lóżmy, że dane sa ↩<br />

trzy liczby ca̷lkowite m , n , p ,<br />

z których przynajmniej jedna jest różna od zera. Określ NWD dla<br />

tych liczb przez analogie ↩<br />

do NWD dla dwóch liczb, a naste ↩<br />

pnie oblicz<br />

NWD(24,36,120) .<br />

1.6.10. Uogólnij definicje ↩<br />

najwie ↩<br />

kszego wspólnego dzielnika<br />

na przypadek k liczb, tj. dla a 1 , a 2 , . . . , a k ∈ Z . Zdefiniuj<br />

NWD(a 1 ,a 2 ,. . . ,a k ) . Oblicz NWD(36,120,180,600) .


16 Cze ↩<br />

ść I – Zadania<br />

1.6.11. Za̷lóżmy, że dane sa ↩<br />

trzy liczby ca̷lkowite m , n i p .<br />

Zdefiniujmy PNWD(m,n,p) = df<br />

NWD(m, NWD(n, p)) . Pokaż, że tak zdefiniowany<br />

PNWD jest równy najwie ↩<br />

kszemu wspólnemu dzielnikowi<br />

liczb m , n i p (zdefiniowanemu w zadaniu 1.6.9).<br />

1.7. Najmniejsza wspólna wielokrotność. Za̷lóżmy, że<br />

n i m sa ↩<br />

liczbami ca̷lkowitymi różnymi od zera. Liczbe ↩<br />

ca̷lkowita ↩<br />

s nazywamy najmniejsza ↩<br />

wspólna ↩<br />

wielokrotnościa ↩<br />

liczb m i n (co<br />

zapisujemy NWW(m, n) = s ), jeśli<br />

1) s ≥ 1 ,<br />

2) m|s oraz n|s ,<br />

3) jeżeli liczba ca̷lkowita t spe̷lnia warunek n|t i m|t , to s|t .<br />

Na przyk̷lad NWW(6, 9) = 18 .<br />

Analogicznie określamy najmniejsza ↩<br />

wspólna ↩<br />

wielokrotność k<br />

różnych od zera liczb ca̷lkowitych a 1 , a 2 , . . . , a k i oznaczamy ja ↩<br />

przez NWW(a 1 , a 2 ,. . . ,a k ) .<br />

1.7.1.<br />

(a) Znajdź najmniejsza ↩<br />

liczbe ↩<br />

naturalna ↩<br />

, która po podzieleniu<br />

przez każda ↩<br />

z liczb 2, 3, 4, 5, 6, 7, 8, 9, 10 daje zawsze<br />

reszte ↩<br />

1.<br />

(b) Znajdź najmniejsza ↩<br />

liczbe ↩<br />

naturalna ↩<br />

, która po podzieleniu<br />

przez 2, 3, 4, 5, 6, 7, 8, 9, 10 daje, odpowiednio, reszty 1,<br />

2, 3, 4, 5, 6, 7, 8, 9.<br />

1.7.2. Za̷lóżmy, że NWD(a, b) = d i niech a = da 1 , b = db 1 .<br />

Uzasadnij, że NWW(a, b) = a 1 db 1 .<br />

1.7.3. Pokaż, że dla dowolnych liczb naturalnych a , b zachodzi<br />

równość ab = NWD(a, b) · NWW(a, b).<br />

1.7.4. Wykaż, że jeżeli liczby a i b sa ↩<br />

wzgle ↩<br />

dnie pierwsze, to<br />

NWW(a, b) = ab .<br />

1.7.5. Pokaż, że dla dowolnych liczb naturalnych a , b zachodzi<br />

nierówność<br />

a + b ≤ NWD(a, b) + NWW(a, b).


Podstawowe w̷lasności liczb ca̷lkowitych 17<br />

1.7.6. W podre ↩<br />

cznikach szkolnych można znaleźć naste ↩<br />

puja ↩<br />

ce<br />

definicje NWW i NWD. ,,Najmniejsza ↩<br />

wspólna ↩<br />

wielokrotnościa ↩<br />

liczb<br />

m i n nazywamy najmniejsza ↩<br />

nieujemna ↩<br />

liczbe ↩<br />

w zbiorze wspólnych<br />

wielokrotności m oraz n ”. ,,Najwie ↩<br />

kszym wspólnym dzielnikiem<br />

liczb m i n nazywamy najwie ↩<br />

ksza ↩<br />

liczbe ↩<br />

w zbiorze wspólnych dzielników<br />

m oraz n ”. Pos̷luguja ↩<br />

c sie ↩<br />

tylko tymi ,,szkolnymi” definicjami<br />

pokaż, że<br />

(a) wspólna wielokrotność liczb m i n jest podzielna przez<br />

najmniejsza ↩<br />

wspólna ↩<br />

wielokrotność liczb m i n ;<br />

(b) wspólny dzielnik liczb m i n jest dzielnikiem najwie ↩<br />

kszego<br />

wspólnego dzielnika liczb m i n .<br />

1.8. Zasadnicze twierdzenie arytmetyki. W literaturze<br />

zasadnicze twierdzenie arytmetyki przyjmuje różne (równoważne)<br />

sformu̷lowania. Jedno z nich jest naste ↩<br />

puja ↩<br />

ce<br />

Twierdzenie. Liczba naturalna be ↩<br />

da ↩<br />

ca dzielnikiem iloczynu dwóch<br />

liczb naturalnych i pierwsza wzgle ↩<br />

dem jednego z czynników jest<br />

dzielnikiem drugiego, tzn. jeżeli n|ab i NWD(n, a) = 1 , to n|b .<br />

Dowód. Ponieważ n|ab i a|ab , wie ↩<br />

c ab jest wspólna ↩<br />

wielokrotnościa<br />

↩<br />

liczb n oraz a . Z definicji NWW wynika, że NWW(n, a)|ab . Istnieje<br />

wie ↩<br />

c liczba ca̷lkowita t taka, że NWW(n, a) · t = ab . Ponieważ<br />

NWD(n, a) = 1 , wie ↩<br />

c NWW(n, a) = na . Sta ↩<br />

d nat = ab , a sta ↩<br />

d<br />

ostatecznie nt = b , czyli n|b .<br />

1.8.1. Pokaż, że jeżeli NWD(a, b) = 1 i c|a , to NWD(c, b) = 1 .<br />

1.8.2. Pokaż, że jeżeli NWD(a, c) = 1 oraz NWD(b, c) = 1 , to<br />

NWD(ab, c) = 1 .<br />

1.8.3. Uzasadnij, że jeżeli<br />

NWD(a 1 , a) = NWD(a 2 , a) = . . . = NWD(a n , a) = 1,<br />

to NWD(a 1 a 2 . . . a n , a) = 1 .<br />

1.8.4. Wykaż, że każda liczba wymierna dodatnia daje sie ↩<br />

przedstawić jednoznacznie w postaci ilorazu dwóch liczb naturalnych<br />

wzgle ↩<br />

dnie pierwszych (czyli w postaci u̷lamka nieskracalnego).


18 Cze ↩<br />

ść I – Zadania<br />

2. Liczby pierwsze<br />

2.1. Poje ↩<br />

cie liczby pierwszej. Liczbe ↩<br />

naturalna ↩<br />

p > 1<br />

nazywamy pierwsza ↩<br />

, jeśli ma ona dok̷ladnie dwa dzielniki naturalne,<br />

mianowicie 1 i p . Liczby, które nie sa ↩<br />

pierwszymi, ale sa ↩<br />

wie ↩<br />

ksze<br />

od 1, nazywamy z̷lożonymi.<br />

2.1.1. Wypisz wszystkie liczby pierwsze p < 20 .<br />

2.1.2. Wskaż jaka ↩<br />

kolwiek liczbe ↩<br />

pierwsza ↩<br />

wie ↩<br />

ksza ↩<br />

od<br />

(a) 26 , (b) 56 , (c) 100 .<br />

2.1.3. Uzasadnij, że reszta z dzielenia liczby pierwszej przez<br />

30 jest równa 1 lub pewnej liczbie pierwszej.<br />

2.1.4. Uzasadnij, że kwadrat dowolnej liczby pierwszej<br />

wie ↩<br />

kszej niż 3 daje przy dzieleniu przez 12 reszte ↩<br />

1.<br />

2.1.5. Wykaż, że dla każdej liczby naturalnej n > 2 , liczby<br />

a n = 2 n + 1 oraz b n = 2 n − 1 nie moga ↩<br />

być jednocześnie liczbami<br />

pierwszymi.<br />

2.1.6. Uzasadnij, że liczby naturalnej postaci 6k + 1 (k ∈ N)<br />

nie można przedstawić w postaci różnicy dwóch liczb pierwszych.<br />

2.1.7. Pokaż, że jeżeli p jest liczba ↩<br />

pierwsza ↩<br />

oraz 8p 2 + 1 jest<br />

liczba ↩<br />

pierwsza ↩<br />

, to 8p 2 + 2p + 1 też jest liczba ↩<br />

pierwsza ↩<br />

.<br />

2.1.8. Pokaż, że każda liczba naturalna wie ↩<br />

ksza od 1 ma przynajmniej<br />

jeden dzielnik pierwszy (tj. taki, który jest liczba ↩<br />

pierwsza<br />

↩<br />

).<br />

2.2. Ile jest liczb pierwszych Odpowiedź na pytanie o to,<br />

ile jest liczb pierwszych, zawiera naste ↩<br />

puja ↩<br />

ce<br />

Twierdzenie. Liczb pierwszych jest nieskończenie wiele.<br />

W literaturze można znaleźć wiele dowodów tego twierdzenia.<br />

Niżej umieszczamy jeden z nich, maja ↩<br />

c nadzieje ↩<br />

, że Czytelnik znajdzie<br />

co najmniej kilka innych. W dowodach tego twierdzenia cze ↩<br />

sto<br />

korzysta sie ↩<br />

z faktu zawartego w zadaniu 2.1.8.


18 Cze ↩<br />

ść I – Zadania<br />

Dowód. Przypuśćmy, że istnieja ↩<br />

tylko naste ↩<br />

puja ↩<br />

ce liczby pierwsze:<br />

p 1 , p 2 , . . . , p r . Niech M = p 1 p 2 . . . p r i niech M = st , gdzie<br />

s = p 1 p 2 , a t = p 3 p 4 . . . p r . Zauważmy, że liczba naturalna s + t nie<br />

jest podzielna przez żadna ↩<br />

z liczb p 1 , p 2 , . . . , p r . Otrzymujemy<br />

sprzeczność, ponieważ s + t jest różne od 1 (zobacz zadanie 2.1.8).<br />

2.2.1. Za̷lóżmy, że liczby p 1 , p 2 , . . . , p k sa ↩<br />

liczbami pierwszymi.<br />

Pokaż, że liczba p 1 p 2 . . . p k + 1 nie dzieli sie ↩<br />

z liczb p 1 , p 2 , . . . , p k .<br />

przez żadna ↩<br />

2.2.2. Wykaż, że dla dowolnej liczby naturalnej n , liczba<br />

n! + 1 nie dzieli sie ↩<br />

przez żadna ↩<br />

liczbe ↩<br />

1 < q ≤ n .<br />

2.2.3. Pokaż, że dla dowolnej liczby naturalnej n , istnieje<br />

liczba pierwsza wie ↩<br />

ksza od n .<br />

2.2.4. Niech F n oznacza liczbe ↩<br />

Fermata, tzn. F n = 2 2n + 1 .<br />

Pokaż, że F m oraz F n sa ↩<br />

wzgle ↩<br />

dnie pierwsze dla n ≠ m .<br />

2.2.5. Wykaż, że liczb pierwszych jest nieskończenie wiele,<br />

wykorzystuja ↩<br />

c<br />

(a) zadanie 2.2.1;<br />

(b) zadanie 2.2.3;<br />

(c) zadanie 2.2.4.<br />

2.3. Wnioski z zasadniczego twierdzenia arytmetyki.<br />

Z zasadniczego twierdzenia arytmetyki (zobacz 1.8) wynika naste ↩<br />

-<br />

puja ↩<br />

ce<br />

Twierdzenie. Każda ↩<br />

liczbe ↩<br />

naturalna ↩<br />

przedstawić w postaci<br />

n > 1 można jednoznacznie<br />

n = p 1 p 2 . . . p r ,<br />

gdzie p 1 , p 2 , . . . , p r<br />

sa ↩<br />

liczbami pierwszymi takimi, że<br />

p 1 ≤ p 2 ≤ · · · ≤ p r .<br />

Dowód 1. Wykażemy najpierw, że każda liczba naturalna n > 1<br />

da sie ↩<br />

przedstawić w postaci iloczynu liczb pierwszych. Uczynimy


Liczby pierwsze 19<br />

to stosuja ↩<br />

c indukcje ↩<br />

. Dla n = 2 fakt jest prawdziwy. Za̷lóżmy,<br />

że fakt jest prawdziwy dla wszystkich liczb naturalnych mniejszych<br />

od k . Jeżeli k jest liczba ↩<br />

pierwsza ↩<br />

, to fakt jest prawdziwy. Jeśli<br />

k jest liczba ↩<br />

z̷lożona ↩<br />

, to k = bc , gdzie b oraz c sa ↩<br />

liczbami<br />

mniejszymi od k . Z za̷lożenia indukcyjnego b = p 1 p 2 . . . p s oraz<br />

c = p s+1 p s+2 . . . p t . Sta ↩<br />

d<br />

k = p 1 p 2 . . . p s p s+1 . . . p t .<br />

2. Teraz wykażemy jednoznaczność przedstawienia. Uczynimy<br />

to znowu stosuja ↩<br />

c indukcje ↩<br />

. Dla n = 2 twierdzenie jest<br />

prawdziwe. Za̷lóżmy, że twierdzenie jest prawdziwe dla liczb naturalnych<br />

mniejszych od k . Przypuśćmy, że<br />

k = p 1 p 2 . . . p r = q 1 q 2 . . . q s , (1)<br />

gdzie p 1 , p 2 , . . . , p r , q 1 , q 2 , . . . , q s sa ↩<br />

liczbami pierwszymi,<br />

takimi że<br />

p 1 ≤ p 2 ≤ · · · ≤ p r , q 1 ≤ q 2 ≤ · · · ≤ q s .<br />

Ponieważ p 1 |q 1 q 2 . . . q s , wie ↩<br />

c p 1 |q 1 lub p 1 |q 2 , lub . . . , lub p 1 |q s<br />

(wynika to z zasadniczego twierdzenia arytmetyki). Niech na<br />

przyk̷lad p 1 |q 1 . Ponieważ p 1 oraz q 1 sa ↩<br />

liczbami pierwszymi, wie ↩<br />

c<br />

p 1 = q 1 . Dziela ↩<br />

c (1) przez p 1 mamy<br />

k<br />

p 1<br />

= p 2 p 3 . . . p r = q 2 q 3 . . . q s .<br />

Ponieważ k p 1<br />

< k , wie ↩<br />

c z za̷lożenia indukcyjnego wynika, że r = s<br />

oraz<br />

p 2 = q 2 , p 3 = q 3 , . . . , p r = q r .<br />

2.3.1. Pokaż, że każda ↩<br />

liczbe ↩<br />

naturalna ↩<br />

n ≥ 2 można przedstawić<br />

w postaci<br />

n = p α 1<br />

1 pα 2<br />

2 . . . pα s<br />

s , (2)<br />

gdzie p 1 , p 2 , . . . , p s sa ↩<br />

różnymi liczbami pierwszymi, takimi<br />

że p 1 < p 2 < · · · < p s , natomiast α 1 , α 2 , . . . , α s sa ↩<br />

liczbami<br />

ca̷lkowitymi dodatnimi. (Rozk̷lad (2) liczby naturalnej n nazywamy<br />

rozk̷ladem kanonicznym liczby n na czynniki pierwsze).


20 Cze ↩<br />

ść I – Zadania<br />

2.3.2. Znajdź NWD(a, b) oraz NWW(a, b) znaja ↩<br />

c rozk̷lady kanoniczne<br />

liczb a oraz b .<br />

2.3.3. Korzystaja ↩<br />

c z rozk̷ladu kanonicznego liczb naturalnych<br />

a oraz b uzasadnij, że ab = NWD(a, b)NWW(a, b) .<br />

2.3.4. Pokaż, że jeżeli liczba √ n jest wymierna, gdzie n > 1<br />

jest liczba ↩<br />

naturalna ↩<br />

, to rozk̷lad kanoniczny liczby n ma postać<br />

p 2α 1<br />

1 p 2α 2<br />

2 . . . p 2α s<br />

s .<br />

2.3.5. Za̷lóżmy, że liczba pierwsza p jest dzielnikiem pewnej<br />

liczby naturalnej n . Mówimy, że p α dzieli dok̷ladnie n , jeśli p α |n<br />

oraz p α+1̸ | n . Fakt ten oznaczamy p α ||n .<br />

(a) Udowodnij, że jeżeli p α ||m oraz p β ||n , to p α+β ||mn .<br />

(b) Udowodnij, że jeżeli p α ||m oraz p β ||n oraz α ≠ β , to<br />

p min(α, β) ||m + n .<br />

(c) Sprawdź, czy w (b) można pomina ↩<br />

ć za̷lożenie α ≠ β .<br />

2.3.6. Pokaż, że każda ↩<br />

liczbe ↩<br />

wymierna ↩<br />

można zapisać jednoznacznie<br />

jako iloczyn εp α 1<br />

1 pα 2<br />

2 . . . pα s<br />

s , gdzie ε jest równy 1 lub −1 ,<br />

natomiast p 1 , p 2 , . . . , p s sa ↩<br />

różnymi liczbami pierwszymi, a α 1 ,<br />

α 2 , . . . , α s sa ↩<br />

liczbami ca̷lkowitymi różnymi od zera.<br />

2.3.7. Pokaż, że wyk̷ladnik najwie ↩<br />

kszej pote ↩<br />

gi liczby pierwszej<br />

p , która dzieli n! wynosi α = ∑ [<br />

∞ n<br />

i=1<br />

.<br />

2.3.8. Podaj najwie ↩<br />

ksza ↩<br />

pote ↩<br />

ge ↩<br />

liczby (a) 2, (b) 5 oraz (c) 97,<br />

która dzieli 100! .<br />

2.3.9. Podaj najwie ↩<br />

ksza ↩<br />

pote ↩<br />

ge ↩<br />

liczby (a) 6, (b) 28, która<br />

dzieli 100! .<br />

2.3.10. Oblicz, ile kolejnych zer, licza ↩<br />

c od końca, ma liczba<br />

100! .<br />

p i ]


Liczby pierwsze 21<br />

2.4. Uwagi o funkcji π(x) . Dla każdej liczby rzeczywistej<br />

x > 0 , symbolem π(x) oznaczamy ilość liczb pierwszych p<br />

spe̷lniaja ↩<br />

cych nierówność p ≤ x . pierwszym historycznie stwierdzeniem<br />

na temat funkcji π(x) by̷lo spostrzeżenie J. Bertranda<br />

(z 1845 roku), że mie ↩<br />

dzy n i 2n , gdy n ≥ 2 , znajduje sie ↩<br />

liczba pierwsza. Spostrzeżenie J. Bertranda oznacza, że π(2n) −<br />

π(n) ≥ 1 (dla n ≥ 2 ), czyli że p n+1 < 2p n , gdzie p n oznacza<br />

n-ta ↩<br />

liczbe ↩<br />

pierwsza ↩<br />

. Stwierdzenie to by̷lo znane jako<br />

,,postulat Bertranda” i zosta̷lo udowodnione przez P.L. Czebyszewa<br />

w 1852 roku. Fakt ten be ↩<br />

dziemy w dalszych rozważaniach nazywać<br />

twierdzeniem Czebyszewa (elementarny dowód tego twierdzenia<br />

Czytelnik może znaleźć w [9]). Czebyszew udowodni̷l znacznie<br />

wie ↩<br />

cej, a mianowicie pokaza̷l, że istnieja ↩<br />

sta̷le a oraz b , takie że<br />

a < π(x) :<br />

x<br />

lnx < b dla x > 2 .<br />

Powyższa ↩<br />

nierówność nazywamy nierównościa ↩<br />

Czebyszewa.<br />

2.4.1. Korzystaja ↩<br />

c z nierówności Czebyszewa, pokaż, że<br />

π(x)<br />

lim<br />

x→∞ x = 0.<br />

2.4.2. Korzystaja ↩<br />

c z twierdzenia Czebyszewa, pokaż, że dla<br />

każdej liczby naturalnej n istnieja ↩<br />

co najmniej trzy liczby pierwsze<br />

maja ↩<br />

ce w uk̷ladzie dziesie ↩<br />

tnym n cyfr.<br />

2.4.3. Dowiedź, że dla k > 1 mamy p k+1 ≤ p 1 +p 2 +· · ·+p k ,<br />

gdzie p n oznacza n-ta ↩<br />

z kolei liczbe ↩<br />

pierwsza ↩<br />

.<br />

2.4.4. Pokaż, że 5 jest jedyna ↩<br />

liczba ↩<br />

pierwsza ↩<br />

, która jest<br />

równa sumie wszystkich liczb pierwszych mniejszych od niej.<br />

2.4.5. Pokaż, że 5 jest jedyna ↩<br />

liczba ↩<br />

naturalna ↩<br />

, która jest<br />

równa sumie wszystkich liczb pierwszych mniejszych od niej.<br />

2.4.6. Pokaż, że w rozmieszczeniu liczb pierwszych sa ↩<br />

,,dziury”<br />

dowolnej d̷lugości, tj. pokaż, że dla dowolnego m istnieje n takie,<br />

że p n+1 > p n + m , gdzie p n oznacza n-ta ↩<br />

z kolei liczbe ↩<br />

pierwsza ↩<br />

.<br />

Wskazówka. Pokaż, że wszystkie liczby w cia ↩<br />

gu m liczb (m+1)!+2 ,<br />

(m + 1)! + 3 , . . . , (m + 1)! + (m + 1) sa ↩<br />

z̷lożone.


22 Cze ↩<br />

ść I – Zadania<br />

2.4.7.<br />

(a) Pokaż, że jeżeli p jest liczba ↩<br />

pierwsza ↩<br />

, to π(p−1)<br />

p−1<br />

(b) Pokaż, że jeżeli m jest liczba ↩<br />

z̷lożona ↩<br />

, to π(m)<br />

m<br />

< π(p)<br />

p<br />

.<br />

< π(m−1)<br />

m−1<br />

.<br />

2.5. Twierdzenie Dirichleta. Jak już wiadomo, w poste ↩<br />

pie<br />

arytmetycznym 1, 2, 3, 4, . . . o różnicy 1 istnieje nieskończenie wiele<br />

liczb pierwszych. A jak jest w innych poste ↩<br />

pach arytmetycznych<br />

W 1837 roku P.G.L. Dirichlet udowodni̷l twierdzenie, które orzeka:<br />

Jeśli liczby m , n sa ↩<br />

wzgle ↩<br />

dnie pierwsze, to w cia ↩<br />

gu (mk + n) k<br />

istnieje nieskończenie wiele liczb pierwszych. Dowód twierdzenia<br />

Dirichleta jest dosyć trudny i nie be ↩<br />

dziemy go tu przytaczać. Jednakże<br />

pewne szczególne przypadki tego twierdzenia dowodzi sie ↩<br />

̷latwo, o czym przekona sie ↩<br />

Czytelnik, rozwia ↩<br />

zuja ↩<br />

c niżej umieszczone<br />

zadania.<br />

2.5.1. Podaj przyk̷lad takich liczb m oraz n , że w cia ↩<br />

gu<br />

(mk + n) k (a) jest tylko skończona ilość liczb pierwszych; (b) nie<br />

ma żadnej liczby pierwszej.<br />

2.5.2. Uzasadnij, że istnieje nieskończenie wiele liczb pierwszych<br />

postaci 2k + 1 .<br />

2.5.3.<br />

(a) Pokaż, że w każdym rozk̷ladzie na iloczyn liczby postaci<br />

3k + 2 przynajmniej jeden czynnik jest postaci 3k + 2 .<br />

(b) Udowodnij, że każda liczba naturalna postaci 3k + 2 ma<br />

przynajmniej jeden dzielnik pierwszy tej postaci.<br />

2.5.4. Uzasadnij, że jeśli p 1 , p 2 . . . , p n sa ↩<br />

nieparzystymi<br />

liczbami pierwszymi postaci 3k + 2 , to liczba 3p 1 p 2 . . . p n + 2 nie<br />

dzieli sie ↩<br />

przez żadna ↩<br />

z liczb 2 , p 1 , p 2 , . . . , p n .<br />

2.5.5. Wykaż, że w cia ↩<br />

gu (3k + 2) istnieje nieskończenie wiele<br />

liczb pierwszych.<br />

Wskazówka. Skorzystaj z dwóch poprzednich zadań.<br />

2.5.6. Uzasadnij, że istnieje nieskończenie wiele liczb pierwszych<br />

postaci 4k + 3 .


Liczby pierwsze 23<br />

2.5.7. Pokaż, że w cia ↩<br />

gu (6k + 5) istnieje nieskończenie wiele<br />

liczb pierwszych.<br />

2.6. Liczba dzielników oraz funkcja Eulera. Oznaczmy<br />

przez θ(n) liczbe ↩<br />

naturalnych dzielników liczby n , a przez<br />

ϕ(n) ilość liczb naturalnych wzgle ↩<br />

dnie pierwszych z n , które sa ↩<br />

nie<br />

wie ↩<br />

ksze od n .<br />

2.6.1. Wyznacz θ(n) dla n ≤ 20 .<br />

2.6.2. Pokaż, że jeżeli n = p α 1<br />

1 pα 2<br />

2 . . . pα s<br />

s , to każdy dzielnik<br />

naturalny liczby n jest postaci p β 1<br />

1 pβ 2<br />

2 . . . pβ s<br />

s , gdzie 0 ≤ β i ≤ α i<br />

dla i ∈ {1,2,. . . ,s} .<br />

2.6.3. Wykaż, że<br />

θ(p α 1<br />

1 pα 2<br />

2 . . . pα s<br />

s ) = (α 1 + 1)(α 2 + 1) . . . (α s + 1).<br />

2.6.4. Oblicz θ(100) oraz θ(1024) .<br />

2.6.5. Znajdź wszystkie liczby naturalne n , takie że<br />

θ(n) = 3.<br />

2.6.6. Wyznacz ϕ(n) dla n ≤ 20 .<br />

2.6.7. Pokaż, że liczba n jest pierwsza wtedy i tylko wtedy,<br />

gdy ϕ(n) = n − 1 .<br />

2.6.8. Uzasadnij, że ϕ(pq) = (p − 1)(q − 1) , gdzie p oraz q<br />

sa ↩<br />

różnymi liczbami pierwszymi.<br />

2.6.9. Pokaż, że dla dowolnej liczby pierwszej p i dowolnej<br />

liczby naturalnej n , mamy ϕ(p n ) = p n − p n−1 .<br />

2.6.10. Niech m oraz n oznaczaja ↩<br />

dwie liczby naturalne<br />

wzgle ↩<br />

dnie pierwsze, a r liczbe ↩<br />

ca̷lkowita ↩<br />

. Pokaż, że wówczas reszty<br />

z dzielenia liczb<br />

r, n + r, 2n + r, 3n + r, . . . , (m − 1)n + r<br />

przez m różnia ↩<br />

sie ↩<br />

od liczb 0, 1, 2, 3, . . . , m − 1 co najwyżej<br />

porza ↩<br />

dkiem.


24 Cze ↩<br />

ść I – Zadania<br />

2.6.11. Udowodnij, że jeśli NWD(m, n) = 1 , to ϕ(m)ϕ(n) =<br />

ϕ(mn) .<br />

Wskazówka. Zauważmy najpierw, że jeśli jedna z liczb m , n<br />

jest równa 1, to wzór jest prawdziwy. Możemy zatem za̷lożyć, że<br />

m ≠ 1 i n ≠ 1 . Wypiszmy wszystkie liczby nie wie ↩<br />

ksze od mn<br />

w naste ↩<br />

puja ↩<br />

cy sposób:<br />

1, 2, . . . , r, . . . , n,<br />

n + 1, n + 2, . . . , n + r, . . . , 2n,<br />

2n + 1, 2n + 2, . . . , 2n + r, . . . , 3n,<br />

. . . . . . . . . . . . , . . . . . . . . . . . . , . . . , . . . . . . . . . . . . , . . . , . . . ,<br />

(m − 1)n + 1, (m − 1)n + 2, . . . , (m − 1)n + r, . . . , mn.<br />

Dalej skorzystaj z zadania 2.6.10.<br />

2.6.12. Udowodnij wzór<br />

ϕ (p α 1<br />

1 pα 2<br />

2 . . . pα s<br />

s<br />

gdzie p 1 , p 2 , . . . , p s<br />

) = n<br />

(1 − 1 ) (1 − 1 )<br />

· · ·<br />

(1 − 1 )<br />

,<br />

p 1 p 2 p s<br />

sa ↩<br />

różnymi liczbami pierwszymi.<br />

2.6.13. Wiadomo, że NWD(m, n) > 1 . Ustal, która z liczb<br />

ϕ(mn) oraz ϕ(m)ϕ(n) jest wie ↩<br />

ksza.<br />

Wskazówka. Skorzystaj z zadania 2.6.12.<br />

2.6.14. Pokaż, że<br />

(a) ϕ(4n + 2) = ϕ(2n + 1) ;<br />

{<br />

2ϕ(n) gdy NWD(n, 2) = 1<br />

(b) ϕ(4n) =<br />

2ϕ(2n) gdy NWD(n, 2) = 2 .<br />

2.7. Rozk̷lad na czynniki dużych liczb naturalnych.<br />

Mnożenie dwóch liczb naturalnych jest zdecydowanie ̷latwiejsze niż<br />

rozk̷ladanie danej liczby na czynniki. Jeśli jednak pewna liczba<br />

z̷lożona ma dwa dzielniki pierwsze, które sa ↩<br />

,,blisko” siebie, możemy<br />

zastosować tak zwana ↩<br />

metode ↩<br />

faktoryzacji Fermata. Metode ↩<br />

te ↩<br />

opiszemy w poniższych zadaniach.<br />

2.7.1. Niech n be ↩<br />

dzie dowolna ↩<br />

liczba ↩<br />

naturalna ↩<br />

. Udowodnij,<br />

że istnieje wzajemnie jednoznaczna odpowiedniość mie ↩<br />

dzy dzielnikami<br />

liczby n nie mniejszymi od √ n i nie wie ↩<br />

kszymi od √ n .


Liczby pierwsze 25<br />

2.7.2. Niech n be ↩<br />

dzie liczba ↩<br />

naturalna ↩<br />

nieparzysta ↩<br />

. Pokaż, że<br />

istnieje wzajemnie jednoznaczna odpowiedniość mie ↩<br />

dzy dzielnikami<br />

liczby n nie mniejszymi od √ n a parami (s,t) liczb naturalnych<br />

spe̷lniaja ↩<br />

cymi warunek s 2 − t 2 = n .<br />

2.7.3. Wypisz wszystkie sposoby przedstawienia liczby 945<br />

w postaci różnicy dwóch kwadratów.<br />

2.7.4. Jeśli pewna liczba nieparzysta n daje sie ↩<br />

zapisać<br />

w postaci różnicy dwóch kwadratów t 2 − s 2 , to wówczas, n =<br />

(t − s)(t + s) , wie ↩<br />

c wiemy, jak znaleźć nietrywialne dzielniki liczby<br />

n . Problem polega zatem na zapisaniu liczby n w postaci różnicy<br />

kwadratów. Jeśli dzielniki liczby n sa ↩<br />

,,blisko siebie”, to liczba t<br />

jest niewiele wie ↩<br />

ksza od √ n . Sprawdzamy wie ↩<br />

c kolejno, czy dla<br />

t = [ √ n] + 1,[ √ n] + 2,. . . , liczba t 2 − n jest kwadratem liczby<br />

naturalnej. Na przyk̷lad dla n = 200819 mamy [ √ n] = 448 oraz<br />

450 2 − 200819 = 1681 = 41 2 . Zatem 200819 = 491 · 409 .<br />

Stosuja ↩<br />

c opisana ↩<br />

wyżej metode ↩<br />

roz̷lóż na czynniki liczby 8633,<br />

809009, 92296873, 88169891 oraz 4601.<br />

2.7.5. Czasami ̷latwiej jest roz̷lożyć na czynniki liczbe ↩<br />

kn niż<br />

n , a naste ↩<br />

pnie podzielić otrzymany rozk̷lad przez k , by otrzymać<br />

rozk̷lad liczby n . Znajdź rozk̷lad liczby n na czynniki znajduja ↩<br />

c<br />

najpierw rozk̷lad liczby kn , jeśli<br />

(a) n = 141467 , k = 3 ; (c) n = 68987 , k = 3 ;<br />

(b) n = 29895581 , k = 3 , (d) n = 19578079 , k = 5 .


3. Liczby w różnych systemach pozycyjnych<br />

3.1. Poje ↩<br />

cie pozycyjnego systemu zapisu liczb. Liczby<br />

możemy zapisywać różnymi sposobami. Powszechnie stosowanym<br />

sposobem zapisu liczb jest dziesie ↩<br />

tny system pozycyjny. Nazywa<br />

sie ↩<br />

on dziesie ↩<br />

tnym, ponieważ jego podstawa ↩<br />

jest liczba 10, a pozycyjnym,<br />

ponieważ znaczenie danej cyfry tworza ↩<br />

cej liczbe ↩<br />

zależy od<br />

pozycji, na której sie ↩<br />

ona znajduje. Na przyk̷lad 478 = 4 · 10 2 + 7 ·<br />

10 1 +8·10 0 . Systemy niepozycyjne nie znalaz̷ly zastosowania z uwagi<br />

na duża ↩<br />

trudność wykonywania nawet prostych rachunków. Oprócz<br />

dziesie ↩<br />

tnego systemu pozycyjnego w użyciu jest jeszcze dwójkowy<br />

system pozycyjny. Teoretycznie można rozważać system pozycyjny<br />

o dowolnej podstawie b . My be ↩<br />

dziemy rozważać systemy<br />

o podstawie naturalnej b ≥ 2 . Liczbe ↩<br />

w tym systemie zapiszemy<br />

(d k d k−1 . . . d 1 d 0 ) b , gdzie d i ∈ {0,1,. . . ,b − 1} oraz 0 ≤ i ≤ k , i<br />

be ↩<br />

dziemy rozumieli jako d k · b k + d k−1 · b k−1 + · · · + d 1 · b 1 + d 0 · b 0 .<br />

3.1.1. Podaj przyk̷lad niepozycyjnego systemu zapisu liczb.<br />

3.1.2. Podane niżej liczby przedstaw w systemie dziesie ↩<br />

tnym.<br />

(a) (3423) 5 ;<br />

(b) (11000111) 2 ;<br />

(c) (910) 11 .<br />

3.1.3. Znaki d i , które pojawiaja ↩<br />

sie ↩<br />

w zapisie liczby w systemie<br />

o podstawie b , nazywamy cyframi. Zauważmy, że gdy b ≥ 10<br />

zaczyna nam brakować pojedynczych znaków dla oznaczenia cyfr.<br />

Jak zrozumia̷leś liczbe ↩<br />

(910) 11 – jako 9 · 11 2 + 1 · 11 , czy też jako<br />

9 · 11 + 10 Aby unikna ↩<br />

ć nieporozumień, ,,pożyczamy” kolejnych<br />

liter alfabetu ̷lacińskiego (ba ↩<br />

dź też używamy nawiasów). I tak, na<br />

przyk̷lad w systemie o podstawie 11, A oznacza cyfre ↩<br />

10. Podane<br />

niżej liczby przedstaw w systemie dziesie ↩<br />

tnym.<br />

(a) (910) 11 ;<br />

(b) (1AD) 16 ;<br />

(c) (A2B) 12 .<br />

3.1.4. Liczbe ↩<br />

437 (zapisana ↩<br />

w systemie dziesie ↩<br />

tnym) zapisz<br />

w każdym z naste ↩<br />

puja ↩<br />

cych systemów:


Liczby w różnych systemach pozycyjnych 27<br />

(a) w systemie o podstawie 3,<br />

(b) w systemie o podstawie 5,<br />

(c) w systemie o podstawie 16,<br />

(d) w systemie dwójkowym.<br />

3.1.5. Przy zamianie zapisu liczby n w systemie o podstawie<br />

b 1 na zapis w systemie o podstawie b 2 stosujemy różne metody.<br />

Najbardziej ,,naturalna ↩<br />

” jest taka, w której z systemu o podstawie<br />

b 1 przechodzimy do systemu dziesie ↩<br />

tnego, a potem z systemu<br />

dziesie ↩<br />

tnego do systemu o podstawie b 2 . W 3.2.5 opiszemy metode ↩<br />

zamiany bez ,,pośrednictwa” zapisu dziesie ↩<br />

tnego. Zmień zapis<br />

(a) (12) 3 na zapis w systemie dwójkowym,<br />

(b) (1A) 12 na zapis w systemie o podstawie 16,<br />

(c) (3455) 6 na zapis w systemie o podstawie 4,<br />

(d) (101011101011) 2 na zapis w systemie o podstawie 16.<br />

3.1.6. Zamiany liczby x w systemie o podstawie b na zapis<br />

w systemie o podstawie b s , gdzie s > 1 , możemy dokonać w prosty<br />

sposób: cyfry liczby x zapisanej w systemie o podstawie b grupujemy<br />

w bloki po s cyfr zaczynaja ↩<br />

c od końca (jeżeli ostatni blok ma<br />

mniej niż s cyfr, dodajemy zera), a naste ↩<br />

pnie zamieniamy nasze<br />

bloki na cyfry w systemie o podstawie b s . Na przyk̷lad, jeśli chcemy<br />

zamienić (11101001) 2 na zapis w systemie o podstawie 8, tworzymy<br />

bloki 001 , 101 oraz 011 , które oznaczaja ↩<br />

odpowiednio 1, 5 oraz 3.<br />

Nasza liczba ma wie ↩<br />

c postać (351) 8 . Zamień<br />

(a) (1112) 3 na zapis w systemie o podstawie 27,<br />

(b) (1A) 12 na zapis w systemie o podstawie 144,<br />

(c) (101011101011) 2 na zapis w systemie o podstawie 16.<br />

3.1.7. Opracuj algorytm odwrotny do przedstawionego w poprzednim<br />

zadaniu, tj. algorytm na zamiane ↩<br />

zapisu liczby x<br />

w systemie o podstawie b s na zapis w systemie o podstawie b .<br />

Korzystaja ↩<br />

c z opracowanego algorytmu zmień<br />

(a) (2B) 16 na zapis w systemie o podstawie 4,<br />

(b) (ABCF ) 25 na zapis w systemie o podstawie 5,<br />

(c) (3A56) 16 na zapis w systemie o podstawie 2.<br />

3.1.8. Niech m = (c n c n−1 . . . c 1 c 0 ) b , p = (d k d k−1 . . . d 1 d 0 ) b ,


28 Cze ↩<br />

ść I – Zadania<br />

gdzie c n oraz d k sa ↩<br />

wie ↩<br />

ksze od zera. Pokaż, że jeżeli k < n , to<br />

p < b k+1 oraz p < m .<br />

3.2. Wykonywanie obliczeń w różnych systemach<br />

pozycyjnych. Aby dodać dwie liczby m oraz p zapisane w systemie<br />

o podstawie b , poste ↩<br />

pujemy podobnie jak przy dodawaniu<br />

liczb zapisanych w systemie dziesie ↩<br />

tnym. Dok̷ladniej, zapiszmy<br />

gdzie n ≥ k . Mamy teraz<br />

m = c n b n + c n−1 b n−1 + · · · + c 1 b + c 0<br />

p = d k b k + d k−1 b k−1 + · · · + d 1 b + d 0 ,<br />

m + p = c n b n + · · · + c k b k + c k−1 b k−1 + · · · + c 1 b + c 0<br />

+ 0 · b n + · · · + d k b k + d k−1 b k−1 + · · · + d 1 b + d 0<br />

= (c n + 0)b n + · · · + (c k+1 + 0)b k+1 + (c k + d k )b k<br />

+ (c k−1 + d k−1 )b k−1 + · · · + (c 1 + d 1 )b + (c 0 + d 0 ).<br />

Jeżeli c 0 + b 0 jest mniejsza od b , to jest to ostatnia cyfra liczby<br />

m + p (w systemie o podstawie b ). W przeciwnym wypadku mamy<br />

0 ≤ c 0 + d 0 − b ≤ b − 1 i b jednostek niższego rze ↩<br />

du zamieniamy na<br />

jedna ↩<br />

jednostke ↩<br />

wyższego rze ↩<br />

du otrzymuja ↩<br />

c<br />

(c 1 + d 1 )b + (c 0 + d 0 ) = (c 1 + d 1 + 1)b + (c 0 + d 0 − b).<br />

Tak samo poste ↩<br />

pujemy z kolejnymi cyframi, posuwaja ↩<br />

c sie ↩<br />

od strony<br />

prawej do lewej. Przy powyższym rozumowaniu wygodnie jest<br />

stosować zapis<br />

c n c n−1 . . . c k c k−1 . . . c 2 c 1 c 0<br />

+ d k d k−1 . . . d 2 d 1 d 0<br />

. . . . . . . . . . . . . . . . . . . . . . . . . . . . . .<br />

gdzie pod kreska ↩<br />

(w miejsce kropek) zapisujemy kolejne cyfry sumy<br />

m + p .<br />

Dla przyk̷ladu dodamy liczby (3021) 5 i (433) 5 . 1 + 3 = 4 ,<br />

wie ↩<br />

c 4 jest ostatnia ↩<br />

cyfra ↩<br />

wyniku dodawania. 2 + 3 = 5 , a 5 w systemie<br />

o podstawie 5, to 10, zatem nasza suma kończy sie ↩<br />

cyframi<br />

04, a 1, jako jednostke ↩<br />

wyższego rze ↩<br />

du, ,,trzymamy w pamie ↩<br />

ci”.


Liczby w różnych systemach pozycyjnych 29<br />

0 + 4 + 1 = 5 , wie ↩<br />

c sytuacja nam sie ↩<br />

powtarza i dopisujemy 0 do<br />

wyniku oraz pamie ↩<br />

tamy 1. 1 + 3 + 0 = 4 , wie ↩<br />

c nasza suma to<br />

(4004) 5 . Nasze rozumowanie zapisujemy w znajomy sposób:<br />

3021<br />

+ 433<br />

4004<br />

3.2.1. Wykonaj poniższe dodawania.<br />

(a) (AB123) 14 + (CDA) 14 ;<br />

(b) (1111001101) 2 + (1101101101) 2 ;<br />

(c) (12321) 5 + (23132) 5 .<br />

3.2.2. Opisz algorytm odejmowania liczb w systemie o podstawie<br />

b i wykonaj odejmowania.<br />

(a) (1110101110) 2 − (11101011) 2 ;<br />

(b) (4532)) 6 − (3401) 6 ;<br />

(c) (AB12) 16 − (CDA) 16 .<br />

3.2.3. Opisz algorytm mnożenia liczb w systemie o podstawie<br />

b i wykonaj mnożenia.<br />

(a) (234) 5 · (4) 5 ;<br />

(b) (11110101) 2 · (1101) 2 ;<br />

(c) (AB32) 13 · (9) 13 .<br />

3.2.4. Opisz algorytm dzielenia liczb w systemie o podstawie b<br />

i wykonaj dzielenia.<br />

(a) (11011) 2 : (11) 2 ;<br />

(b) (2361) 7 : (15) 7 ;<br />

(c) (ABC) 16 : (6) 16 .<br />

3.2.5. Przypuśćmy, że mamy liczbe ↩<br />

n = (11011101) 2 zapisana ↩<br />

w systemie dwójkowym. Gdy chcemy ja ↩<br />

przedstawić w systemie<br />

trójkowym, zapisujemy nowa ↩<br />

podstawe ↩<br />

b = 3 w ,,starym” systemie,<br />

tj. b = (11) 2 w naszym przypadku. Również wszystkie obliczenia<br />

wykonujemy w ,,starym” systemie. Obliczamy kolejne pote ↩<br />

gi liczby<br />

b , dopóki nie przekrocza ↩<br />

one liczby n:<br />

b 2 = (1001) 2 , b 3 = (11011) 2 , b 4 = (1010001) 2 , b 5 = (11110011) 2 ,


30 Cze ↩<br />

ść I – Zadania<br />

wie ↩<br />

c b 4 < n < b 5 . Dzielimy (z reszta ↩<br />

) liczbe ↩<br />

n przez b 4 otrzymuja ↩<br />

c<br />

(11011101) 2 = (10) 2 · (1010001) 2 + (111011) 2 .<br />

Reszte ↩<br />

z powyższego dzielenia dzielimy przez b 3 i otrzymujemy<br />

(111011) 2 = (10) 2 · (11011) 2 + (101) 2 .<br />

Ponownie dzielimy otrzymana ↩<br />

reszte ↩<br />

przez b 2 dostaja ↩<br />

c wynik 0<br />

oraz reszte ↩<br />

(101) 2 . Te ↩<br />

reszte ↩<br />

dzielimy przez b i otrzymujemy<br />

Zatem<br />

(101) 2 = (1) 2 · (11) 2 + (10) 2 .<br />

(11011101) 2 = (10) 2 · b 4 + (10) 2 · b 3 + 0 · b 2 + 1 · b + (10) 2<br />

= 2 · 3 4 + 2 · 3 3 + 0 · 3 2 + 1 · 3 + 2<br />

= (22012) 3 .<br />

Zapisz liczbe ↩<br />

n = (11011101) 2 w systemie trójkowym pos̷luguja<br />

↩<br />

c sie ↩<br />

drugim algorytmem z rozwia ↩<br />

zania zadania 3.1.4.<br />

3.2.6. ̷Latwość wykonywania rachunków jest g̷lówna ↩<br />

przyczyna<br />

↩<br />

, dla której używamy pozycyjnych systemów zapisu liczb. Spróbuj<br />

dodać liczby MCMXXIV (1924) oraz CDXCIII (493) zapisane w systemie<br />

rzymskim (niepozycyjnym) nie korzystaja ↩<br />

c z pośrednictwa<br />

żadnego systemu pozycyjnego. Jeśli wydaje Ci sie ↩<br />

to ̷latwe, spróbuj<br />

pomnożyć te dwie liczby...<br />

3.3. U̷lamki w różnych systemach pozycyjnych. Przypomnijmy,<br />

że dla d i ,c j ∈ {0,1,. . . ,9} , zapis<br />

x = d k d k−1 . . . d 0 ,c 1 c 2 . . .<br />

oznacza rozwinie ↩<br />

cie dziesie ↩<br />

tne liczby x , tj.<br />

x = d k · 10 k + d k−1 · 10 k−1 + · · · + d 0 + c 1<br />

} {{ } 10 1 + c 2<br />

10 2 + · · ·<br />

} {{ }<br />

cze ↩<br />

ść ca̷lkowita<br />

cze ↩<br />

ść u̷lamkowa<br />

(1)


Liczby w różnych systemach pozycyjnych 31<br />

Sume ↩<br />

w cze ↩<br />

ści u̷lamkowej wzoru (1) należy rozumieć jako granice ↩<br />

cia ↩<br />

gu sum cze ↩<br />

ściowych<br />

s 1 = c 1<br />

10<br />

s 2 = s 1 + c 2<br />

10 2<br />

. . . . . . . . . . . . . . .<br />

Podobnie interpretujemy zapis ,,z przecinkiem” w innym niż dziesie ↩<br />

-<br />

tny systemie pozycyjnym. Zapis x = (d k d k−1 . . . d 0 ,c 1 c 2 . . .) b , gdzie<br />

d i ,c j ∈ {0,1,. . . ,b − 1} , oznacza, że x ma rozwinie ↩<br />

cie w systemie<br />

o podstawie b , czyli że<br />

x = d k · b k + d k−1 · b k−1 + · · · + d 0 + c 1<br />

b 1 + c 2<br />

b 2 + · · ·<br />

Podobnie jak w (1) wyróżniamy tu cze ↩<br />

ść ca̷lkowita ↩<br />

i u̷lamkowa ↩<br />

.<br />

Mówimy, że liczba x ma rozwinie ↩<br />

cie okresowe o podstawie b ,<br />

gdy to rozwinie ↩<br />

cie ma postać<br />

x = d k · b k + d k−1 · b k−1 + · · · + d 0<br />

} {{ }<br />

cze ↩<br />

ść ca̷lkowita<br />

c 1<br />

+<br />

b 1 + c 2<br />

b 2 + · · · + c m<br />

} {{<br />

b m<br />

}<br />

nieokresowe wyrazy pocza ↩<br />

tkowe<br />

+ a 1<br />

b m+1 + a 2<br />

b m+2 + · · · + a n<br />

b m+n<br />

} {{ }<br />

okres<br />

+ a 1<br />

b m+n+1 + a 2<br />

b m+n+2 + · · · + a n<br />

} {{<br />

b m+2n<br />

}<br />

okres<br />

+ · · · · · · ,<br />

gdzie d 1 , d 2 , . . . , d k ; c 1 , c 2 , . . . , c m oraz a 1 , a 2 , . . . , a n<br />

sa ↩<br />

elementami zbioru {0,1,. . . b − 1} .<br />

Jeżeli mamy liczbe ↩<br />

x zapisana ↩<br />

w systemie dziesie ↩<br />

tnym i chcemy<br />

znaleźć jej rozwinie ↩<br />

cie w systemie o podstawie b , to najpierw


32 Cze ↩<br />

ść I – Zadania<br />

znajdujemy rozwinie ↩<br />

cie cze ↩<br />

ści ca̷lkowitej liczby x , a naste ↩<br />

pnie cze ↩<br />

ści<br />

u̷lamkowej tej liczby. Oznaczmy przez y cze ↩<br />

ść u̷lamkowa ↩<br />

liczby x .<br />

Mnożymy liczbe ↩<br />

y przez b i szukamy najwie ↩<br />

kszej liczby ca̷lkowitej<br />

d 1 , takiej że d 1 ≤ yb , czyli cze ↩<br />

ści ca̷lkowitej liczby yb . Liczba<br />

d 1 jest pierwsza ↩<br />

cyfra ↩<br />

po przecinku. Dok̷ladnie, mamy wtedy y =<br />

d 1<br />

b<br />

+ r 1 , gdzie 0 ≤ r 1 < 1 b . Naste ↩pnie szukamy najwie ↩<br />

kszej liczby<br />

ca̷lkowitej d 2 , takiej że d 2 ≤ r 1 b 2 . Liczba d 2 jest druga ↩<br />

cyfra ↩<br />

po<br />

przecinku. Mamy zatem r 1 = d 2<br />

b 2 + r 2 , gdzie 0 ≤ r 2 < 1 b<br />

, a sta 2 ↩<br />

d<br />

y = d 1<br />

b<br />

+ d 2<br />

b 2 + r 2 . Po n takich krokach otrzymujemy<br />

y = d 1<br />

b + d 2<br />

b 2 + · · · + d n<br />

b n + r n, gdzie 0 ≤ r n < 1<br />

b n .<br />

3.3.1. Zapisz<br />

1<br />

(a)<br />

3<br />

i 0,5 w systemie o podstawie 3,<br />

(b) 5 pierwszych cyfr rozwinie ↩<br />

cia liczby π w systemie<br />

o podstawie dwa,<br />

(c)<br />

5<br />

13<br />

w systemie o podstawie 5.<br />

3.3.2. Pokaż, że u̷lamek w̷laściwy o mianowniku b − 1 ma<br />

w systemie o podstawie b ≠ 2 rozwinie ↩<br />

cie okresowe o okresie jednocyfrowym.<br />

3.3.3. Pokaż, że u̷lamek w̷laściwy o mianowniku (11) b , tj.<br />

b + 1 , zapisany w systemie o podstawie b ma w tym systemie<br />

rozwinie ↩<br />

cie okresowe o okresie dwucyfrowym.<br />

3.3.4. Pokaż, że w systemie o podstawie b u̷lamek ma<br />

rozwinie ↩<br />

cie skończone wtedy i tylko wtedy, gdy mianownik tego<br />

u̷lamka (zapisanego w postaci nieskracalnej) jest iloczynem pote ↩<br />

g<br />

dzielników b .<br />

3.3.5. Udowodnij, że u̷lamek w̷laściwy a c<br />

ma w systemie<br />

o podstawie b rozwinie ↩<br />

cie okresowe f -cyfrowe czyste (tzn. okres<br />

zaczyna sie ↩<br />

od cyfry po przecinku i ma f cyfr) wtedy i tylko wtedy,<br />

gdy b f − 1 jest wielokrotnościa ↩<br />

c .<br />

3.3.6. Oblicz<br />

(a) 111,101010 + 1,11 w systemie dwójkowym,<br />

(b) 121,2101 − 12,11211 w systemie trójkowym,


Liczby w różnych systemach pozycyjnych 33<br />

(c) A,BC : 0,6 w systemie o podstawie 16,<br />

(d) 43201 : 3 w systemie o podstawie 5,<br />

(e) 0,00012 · 0,0023 w systemie o podstawie 4.


4. Algorytm Euklidesa<br />

4.1. Szukanie NWD. Aby znaleźć NWD dwóch liczb naturalnych,<br />

rozk̷ladamy je na czynniki pierwsze, a naste ↩<br />

pnie wyszukujemy<br />

wspólne dla obu liczb czynniki. Iloczyn tych wspólnych czynników<br />

jest równy NWD. Na przyk̷lad, aby znaleźć NWD(24, 36) ,<br />

rozk̷ladamy liczby 24 oraz 36 na czynniki pierwsze<br />

24 2 36 2<br />

12 2 18 2<br />

6 2 9 3<br />

3 3 3 3<br />

1 1<br />

Wspólnymi czynnikami sa ↩<br />

tu 2, 2 oraz 3, zatem<br />

NWD(24, 36) = 2 · 2 · 3 = 12.<br />

Opisana wyżej metoda zawodzi już przy liczbach trzycyfrowych.<br />

Nie jest bowiem ̷latwo roz̷lożyć ,,duża ↩<br />

” liczbe ↩<br />

na czynniki.<br />

Znacznie skuteczniejszy jest algorytm Euklidesa. Podstawa ↩<br />

tego algorytmu<br />

jest fakt, że jeżeli a = qb + r , to NWD(a, b) = NWD(b, r)<br />

(zobacz 1.6.2). Przypuśćmy, że chcemy znaleźć NWD(a, b) , gdzie a<br />

oraz b sa ↩<br />

liczbami naturalnymi, przy czym a > b . Przyjmujemy<br />

r −1 = a oraz r 0 = b , a naste ↩<br />

pnie określamy rekurencyjnie cia ↩<br />

g (r k )<br />

jako kolejne reszty z dzielenia r k−2 przez r k−1 , tj.<br />

r k−2 = q k r k−1 + r k , (*)<br />

gdzie q k dla k ∈ N sa ↩<br />

pewnymi liczbami ca̷lkowitymi.<br />

Za̷lóżmy, że chcemy znaleźć NWD(704, 341) . Wówczas, aby<br />

otrzymać cia ↩<br />

g (r k ) , wykonujemy kolejne dzielenia z reszta ↩<br />

:<br />

704 = 2 · 341 + 22<br />

341 = 15 · 22 + 11<br />

22 = 2 · 11 + 0.<br />

Zatem r 1 = 22 , r 2 = 11 , r 3 = 0 . Zauważmy, że z (*) bezpośrednio<br />

wynika, iż cia ↩<br />

g (r k ) jest ściśle maleja ↩<br />

cy. Ponieważ jest to cia ↩<br />

g liczb


Algorytm Euklidesa 35<br />

ca̷lkowitych nieujemnych, wie ↩<br />

c musi istnieć takie n , że r n+1 = 0 .<br />

Z zadania 1.6.2 wynika natomiast, że<br />

NWD(a, b) = NWD(r −1 , r 0 ) = NWD(r 0 , r 1 ) = NWD(r 1 , r 2 )<br />

= · · · = NWD(r n−1 , r n ).<br />

Ale, ponieważ r n+1 = 0 , wie ↩<br />

c z (*) wynika, że r n |r n−1 , czyli r n =<br />

NWD(r n−1 , r n ) = NWD(a, b) .<br />

Zatem najwie ↩<br />

kszy wspólny dzielnik liczb a i b , to ostatni<br />

niezerowy element w opisanym wyżej cia ↩<br />

gu (r n ) . Wracaja ↩<br />

c do<br />

naszego przyk̷ladu, otrzymujemy NWD(704, 341) = 11 .<br />

4.1.1. Znajdź NWD(704, 341) nie korzystaja ↩<br />

c z algorytmu Euklidesa.<br />

4.1.2. Znajdź NWD(128, 1024) nie korzystaja ↩<br />

c z algorytmu<br />

Euklidesa, a naste ↩<br />

pnie korzystaja ↩<br />

c z tego algorytmu.<br />

4.1.3. W 1.6 pokazaliśmy, że jeżeli NWD(a, b) = d , to istnieja ↩<br />

takie liczby ca̷lkowite x i y , że ax+by = d . Zastosuj algorytm Euklidesa<br />

do znalezienia NWD podanych par liczb, a naste ↩<br />

pnie znajdź<br />

odpowiadaja ↩<br />

ce tym parom x i y .<br />

(a) 26, 19; (b) 187, 34; (c) 841, 160; (d) 2613, 2171.<br />

4.1.4. Wykaż, że w cia ↩<br />

gu (r k ) , określonym przez algorytm<br />

Euklidesa, zachodzi zależność r k+2 < 1 2 r k .<br />

4.1.5. Dzia̷lanie algorytmu Euklidesa można nieznacznie<br />

przyspieszyć dopuszczaja ↩<br />

c tzw. ujemne reszty, to jest wybieraja ↩<br />

c<br />

r k−2 = q k r k−1 + r k lub r k−2 = q k r k−1 − r k w zależności od tego,<br />

które dzia̷lanie daje mniejsza ↩<br />

reszte ↩<br />

r k . Wykorzystaj te ↩<br />

metode ↩<br />

w czterech przyk̷ladach zadania 4.1.3.<br />

4.1.6. Pokaż, że dla cia ↩<br />

gu (r k ) z zadania 4.1.5 zachodzi<br />

r k ≤ 1 2 r k−1.<br />

4.1.7. Korzystaja ↩<br />

c z algorytmu Euklidesa, znajdź<br />

(a) NWD(10n + 9, n + 1) , gdzie n ∈ N ;<br />

(b) NWD(3n + 1, 10n + 3) , gdzie n ∈ N .


36 Cze ↩<br />

ść I – Zadania<br />

4.1.8. Pokaż, że liczba naturalna n = 10a + b , gdzie b ∈<br />

{0,1,. . . ,9} dzieli sie ↩<br />

przez m = 10q + 1 wtedy i tylko wtedy, gdy<br />

m|a − bq .<br />

4.1.9. Rozwia ↩<br />

ż w liczbach naturalnych uk̷lad równań<br />

x + y = 96, NWD(x, y) = 12.<br />

4.1.10. Rozwia ↩<br />

ż równanie NWD(x, a) = b , gdzie a i b sa ↩<br />

liczbami ca̷lkowitymi, przy czym b ≠ 0 .<br />

4.1.11. Sprawdź, czy opisany wyżej algorytm Euklidesa można<br />

wykorzystać do wyznaczania NWD dowolnych liczb ca̷lkowitych<br />

niezerowych.<br />

4.1.12. Uzasadnij, że suma dwóch kolejnych liczb naturalnych<br />

oraz suma ich kwadratów sa ↩<br />

wzgle ↩<br />

dnie pierwsze.<br />

4.2. Równania liniowe. Zajmiemy sie ↩<br />

teraz równaniami<br />

postaci<br />

ax + by = c, (1)<br />

gdzie a , b , c ∈ Z .<br />

Twierdzenie. Równanie (1) ma rozwia ↩<br />

zanie w liczbach ca̷lkowitych<br />

wtedy i tylko wtedy, gdy NWD(a, b)|c .<br />

Dowód. Za̷lóżmy, że równanie (1) ma rozwia ↩<br />

zanie (x 0 ,y 0 ) . Zatem,<br />

jeśli p|a oraz p|b , to p|ax 0 + by 0 , czyli p|c . W szczególności, jeśli<br />

p = NWD(a, b) , to p|c .<br />

Za̷lóżmy teraz, że d = NWD(a, b) i d|c . Niech e = c d . Istnieja ↩<br />

liczby x ′ oraz y ′ takie, że ax ′ + by ′ = d . Zatem para (ex ′ ,ey ′ )<br />

jest rozwia ↩<br />

zaniem (1).<br />

4.2.1. Sprawdź, które z poniższych równań maja ↩<br />

rozwia ↩<br />

zania<br />

w liczbach ca̷lkowitych.<br />

(a) 12x + 18y = 36 (c) 2613x + 2171y = 39 (e) 119x + 105y = 28<br />

(b) 65x + 39y = 16 (d) 12x + 15y = 333 (f) 28x + 35y = 347 .<br />

4.2.2. Wykaż, że jeśli NWD(a, b) = 2 , to równanie (1) ma<br />

rozwia ↩<br />

zanie wtedy i tylko wtedy, gdy c jest liczba ↩<br />

parzysta ↩<br />

.


Algorytm Euklidesa 37<br />

4.2.3. Niech A be ↩<br />

dzie podzbiorem liczb ca̷lkowitych zawieraja<br />

↩<br />

cym dwie liczby wzgle ↩<br />

dnie pierwsze oraz takim, że wraz z dwiema<br />

liczbami (różnymi lub nie) zawiera ich sume ↩<br />

i różnice ↩<br />

. Pokaż, że<br />

A = Z .<br />

4.2.4. Niech A be ↩<br />

dzie podzbiorem liczb ca̷lkowitych, który<br />

zawiera co najmniej jedna ↩<br />

liczbe ↩<br />

różna ↩<br />

od zera oraz takim, że wraz<br />

z dwiema liczbami (różnymi lub nie) zawiera ich sume ↩<br />

i różnice ↩<br />

.<br />

Pokaż, że A sk̷lada sie ↩<br />

ze wszystkich liczb postaci mt , gdzie m<br />

jest najmniejsza ↩<br />

liczba ↩<br />

naturalna ↩<br />

zbioru A , a t dowolna ↩<br />

liczba ↩<br />

ca̷lkowita ↩<br />

.<br />

4.2.5. Napisz równanie prostej na p̷laszczyźnie, do której nie<br />

należa ↩<br />

punkty o obu wspó̷lrze ↩<br />

dnych ca̷lkowitych.<br />

4.3. Rozwia ↩<br />

zywanie równań liniowych. Pokazanie, że<br />

rozwia ↩<br />

zanie istnieje, a znalezienie tego rozwia ↩<br />

zania, to dwie różne<br />

sprawy. Twierdzenie z podrozdzia̷lu 4.2 mówi nam jedynie o istnieniu<br />

rozwia ↩<br />

zania. By znaleźć to rozwia ↩<br />

zanie, pos̷lużymy sie ↩<br />

algorytmem<br />

podobnym do algorytmu Euklidesa.<br />

Za̷lóżmy, że mamy dane równanie (1), w którym a ≥ b . Oznaczmy<br />

a = r −1 oraz b = r 0 . Niech r n be ↩<br />

dzie ostatnia ↩<br />

niezerowa ↩<br />

reszta ↩<br />

w cia ↩<br />

gu reszt uzyskanym przez zastosowanie algorytmu Euklidesa<br />

do liczb r −1 i r 0 . Możemy zapisać r −1 = q 1 r 0 + r 1 , gdzie<br />

0 ≤ r 1 < r 0 . Wstawiaja ↩<br />

c do (1) mamy<br />

(q 1 r 0 + r 1 )x + r 0 y = c.<br />

Podstawiaja ↩<br />

c y 1 = x , x 1 = y + q 1 x , otrzymujemy<br />

r 0 x 1 + r 1 y 1 = c. (2)<br />

̷Latwo spostrzec, że jeśli x oraz y spe̷lniaja ↩<br />

(1), to wyznaczone<br />

przez nie liczby ca̷lkowite x 1 , y 1 spe̷lniaja ↩<br />

(2). Na odwrót, jeżeli<br />

x 1 , y 1 spe̷lniaja ↩<br />

(2), to x = y 1 , y = x 1 − q 1 y 1 spe̷lniaja ↩<br />

(1).<br />

Zatem badanie równania (1) sprowadza sie ↩<br />

do badania równania (2),<br />

w którym wspó̷lczynniki sa ↩<br />

mniejsze.<br />

Od równania (2) przechodzimy w podobny sposób do równania<br />

r 1 x 2 + r 2 y 2 = c, (3)


38 Cze ↩<br />

ść I – Zadania<br />

i tak dalej, aż dojdziemy do równania<br />

r n−1 x n + r n y n = c. (4)<br />

Ale r n jest ostatnia ↩<br />

niezerowa ↩<br />

reszta ↩<br />

, wie ↩<br />

c r n = NWD(r n−1 , r n ) =<br />

NWD(a, b) . Zatem r n |r n−1 i r n |c . Jeżeli r n−1 = q n+1 r n , to (4)<br />

przyjmuje postać<br />

r n x n+1 = c, (5)<br />

gdzie x n+1 = q n+1 x n + y n . Zauważmy, że jeśli x n+1 spe̷lnia (5), to<br />

dobieraja ↩<br />

c dowolna ↩<br />

wartość ca̷lkowita ↩<br />

dla x n i podstawiaja ↩<br />

c y n =<br />

x n+1 − q n+1 x n otrzymamy rozwia ↩<br />

zanie (4). Na odwrót, jeżeli x n<br />

i y n spe̷lniaja ↩<br />

(4), to liczba x n+1 = q n+1 x n + y n spe̷lnia (5).<br />

Aby otrzymać rozwia ↩<br />

zanie (1) podstawiamy wie ↩<br />

c x n = t<br />

i w zależności od t obliczamy kolejno y n , x n−1 , y n−1 . . . , x 1 ,<br />

y 1 , x , y .<br />

4.3.1. Rozwia ↩<br />

ż równania.<br />

(a) 12x + 18y = 36 (d) 12x + 15y = 333<br />

(b) 65x + 39y = 26 (e) 4x + 4y = 8<br />

(c) 2613x + 2171y = 39 (f) 119x + 105y = 28 .<br />

4.3.2. W zależności od parametru u rozwia ↩<br />

ż równanie<br />

12x + 15y = 3(7u − 22).<br />

4.3.3. Rozwia ↩<br />

ż równania.<br />

(a) 12x + 15y + 7z = 11 ; (b) 6x + 10y + 12z = 4 .<br />

Wskazówka. Równanie (a) jest równoważne równaniu 12x + 15y =<br />

11 − 7z . Zatem NWD(12, 15) = 3 musi dzielić (jeśli rozwia ↩<br />

zanie<br />

istnieje) różnice ↩<br />

11−7z . Zapisz 3t = 11−7z i rozwia ↩<br />

ż to równanie.<br />

Niewiadoma z be ↩<br />

dzie już obliczona. Aby znaleźć x oraz y rozwia ↩<br />

ż<br />

12x + 15y = 3t , gdzie t jest otrzymanym wcześniej rozwia ↩<br />

zaniem.<br />

4.3.4. Równanie 24x + 18y = 19 nie ma rozwia ↩<br />

zania. Nie<br />

bacza ↩<br />

c na to, spróbuj je ,,rozwia ↩<br />

zać” stosuja ↩<br />

c opisany algorytm.<br />

Gdzie wyste ↩<br />

puje problem<br />

4.3.5. Niech para liczb ca̷lkowitych (x 0 ,y 0 ) be ↩<br />

dzie rozwia ↩<br />

zaniem<br />

równania (1). Pokaż, że wszystkie rozwia ↩<br />

zania równania (1)<br />

dane sa ↩<br />

wzorami x = x 0 + b 1 t , y = y 0 − a 1 t , gdzie a 1 =<br />

b 1 =<br />

b<br />

NWD(a,b) , t ∈ Z .<br />

a<br />

NWD(a,b) ,<br />

4.3.6. Korzystaja ↩<br />

c z 4.3.5 znajdź wszystkie rozwia ↩<br />

zania<br />

równania 852x + 192y = 24 .


5. Kongruencje<br />

5.1. Podstawowe w̷lasności kongruencji. Niech a , b<br />

oraz m be ↩<br />

da ↩<br />

liczbami ca̷lkowitymi, przy czym m > 1 . Jeżeli<br />

m|a − b , to piszemy<br />

a ≡ b(mod m ) (*)<br />

i mówimy, że liczby a i b przystaja ↩<br />

do siebie modulo m . Na<br />

przyk̷lad −2 ≡ 4(mod 3 ) , 9 ≡ 1(mod 2 ) itp. Znak ,, ≡ ” nazywamy<br />

znakiem kongruencji, a liczbe ↩<br />

m , modu̷lem kongruencji.<br />

Relacje ↩<br />

oznaczona ↩<br />

symbolelm (*) nazywamy kongruencja ↩<br />

.<br />

5.1.1. Sprawdź, czy kongruencja jest relacja ↩<br />

równoważności.<br />

5.1.2. Pokaż, że dwie liczby ca̷lkowite przystaja ↩<br />

do siebie<br />

modulo m wtedy i tylko wtedy, gdy przy dzieleniu przez m<br />

maja ↩<br />

one jednakowe reszty, i wywnioskuj sta ↩<br />

d, że każda klasa<br />

równoważności relacji ,, ≡ ” ma dok̷ladnie jednego reprezentanta<br />

w zbiorze {0,1,2,. . . ,m − 1} .<br />

5.1.3. Pokaż, że kongruencja liczb a i b modulo m jest<br />

równoważna z możliwościa ↩<br />

przedstawienia liczby a w postaci a =<br />

b + mt , gdzie t jest liczba ↩<br />

ca̷lkowita ↩<br />

.<br />

5.1.4. Pokaż, że kongruencje o tym samym module można<br />

dodawać, odejmować i mnożyć stronami.<br />

5.1.5. Zauważmy, że chociaż 48 ≡ 18(mod 10 ) oraz 12 ≡<br />

2(mod 10 ) , to 4 ≢ 9(mod10) , czyli że nie mamy prawa dzielić<br />

stronami kongruencji.<br />

(a) Pokaż, że dla każdego c > 0 , jeśli ac ≡ bc(mod mc ),<br />

to a ≡ b(mod m ).<br />

(b) Wykaż, że jeżeli ac ≡ bc(mod m ) oraz NWD(m, c) =<br />

1 , to a ≡ b(mod m ).<br />

(c) Pokaż, że jeżeli a s ≡ 1(mod m ) i a t ≡ 1(mod n ) ,<br />

przy czym NWD(m, n) = 1, to zachodzi kongruencja<br />

a NWW(s,t) ≡ 1(mod mn ).<br />

(d) Wykaż, że jeżeli a ≡ b(mod n ) oraz a ≡ b(mod m ),<br />

to a ≡ b(mod NWW(m, n) ).


40 Cze ↩<br />

ść I – Zadania<br />

5.1.6. Pokaż, że 11|2 55 + 1 . Jaka jest ostatnia cyfra liczby<br />

2 55 + 1 w systemie o podstawie 11<br />

5.1.7. Wykaż, że 78|53 103 + 103 53 .<br />

5.1.8.<br />

(a) Znajdź ostatnia ↩<br />

cyfre ↩<br />

liczby 2 1000 .<br />

(b) Wyznacz cia ↩<br />

g ostatnich cyfr dla cia ↩<br />

gu liczb Fermata F n =<br />

2 2n + 1 ( n = 0,1,2,3,. . . ).<br />

5.1.9. Zbadaj, kiedy suma sześcianów trzech kolejnych liczb<br />

naturalnych jest podzielna przez 18.<br />

5.1.10. Udowodnij, że jeżeli n jest liczba ↩<br />

ca̷lkowita ↩<br />

nieujemna ↩<br />

,<br />

to liczba 2 n+2 + 3 2n+1 jest podzielna przez 7 .<br />

5.1.11. Pokaż, że jeżeli p jest liczba ↩<br />

pierwsza ↩<br />

, to<br />

(a) ( p<br />

k)<br />

≡ 0(mod p ) dla 1 ≤ k ≤ p − 1 ;<br />

(b) ( )<br />

p n<br />

k ≡ 0(mod p ) dla dowolnego ca̷lkowitego n ≥<br />

1 oraz 1 ≤ k ≤ p n − 1 ;<br />

(c) (a + b) pn ≡ a pn + b pn (mod p ) dla dowolnych liczb<br />

ca̷lkowitych a , b i dowolnego ca̷lkowitego n ≥ 0 .<br />

5.1.12. Uzasadnij, że dla każdej liczby naturalnej n , liczba<br />

2 n + 1 nie jest podzielna przez 7. Jaka może być ostatnia cyfra<br />

liczby 2 n + 1 w systemie o podstawie 7<br />

5.2. Kongruencje a wielomiany. Niech W (x) = w 0 +<br />

w 1 x + w 2 x 2 + · · · + w n x n be ↩<br />

dzie wielomianem o wspó̷lczynnikach<br />

ca̷lkowitych i niech a ≡ b(mod m ) . Z w̷lasności kongruencji<br />

wynika, że<br />

w 0 ≡ w 0 (modm)<br />

w 1 a ≡ w 1 b(modm)<br />

w 2 a 2 ≡ w 2 b 2 (modm)<br />

. . . . . . . . . . . . . . . . . . . . . . . .<br />

w n a n ≡ w n b n (modm).<br />

Dodaja ↩<br />

c stronami powyższe kongruencje otrzymujemy<br />

w 0 +w 1 a+w 2 a 2 +· · ·+w n a n ≡ w 0 +w 1 b+w 2 b 2 +· · ·+w n b n (mod m ),<br />

czyli W (a) ≡ W (b)(mod m ) .


Kongruencje 41<br />

5.2.1. Pokaż, że jeżeli A = 1 + 8 + 2 · 8 2 + · · · + n8 n , B =<br />

1 + 3 + 2 · 3 2 + · · · + n3 n , to 5|A − B .<br />

5.2.2. Udowodnij, że jeżeli wielomian W (x) o wspó̷lczynnikach<br />

ca̷lkowitych jest taki, że W (2) jest podzielne przez 5 i W (5)<br />

jest podzielne przez 2, to W (7) jest podzielne przez 10.<br />

5.2.3. Niech n be ↩<br />

dzie liczba ↩<br />

naturalna ↩<br />

, a n 0 , n 1 , . . . , n s<br />

jej kolejnymi cyframi w uk̷ladzie dziesie ↩<br />

tnym, tzn. n = n 0 + n 1 10 +<br />

· · · + n s 10 s . Rozważmy wielomian N(x) = n 0 + n 1 x + · · · + n s x s .<br />

Jest to wielomian o wspó̷lczynnikach ca̷lkowitych oraz N(10) = n .<br />

Z kongruencji 10 ≡ 1(mod 3 ) wynika, że n = N(10) ≡ N(1) =<br />

n 0 + n 1 + · · · n s (mod 3 ) . Wyprowadź sta ↩<br />

d ceche ↩<br />

podzielności<br />

przez 3.<br />

5.2.4. Wyprowadź ceche ↩<br />

podzielności przez 9.<br />

5.2.5. Opieraja ↩<br />

c sie ↩<br />

na kongruencji 10 ≡ −1(mod 11 ) ,<br />

wyprowadź ceche ↩<br />

podzielności przez 11.<br />

5.2.6. Dane sa ↩<br />

trzy różne liczby ca̷lkowite a , b i c . Udowodnij,<br />

że nie istnieje wielomian W (x) o wspó̷lczynnikach ca̷lkowitych,<br />

taki że W (a) = b , W (b) = c , W (c) = a .<br />

5.2.7. Uzasadnij, że nie istnieje wielomian o wspó̷lczynnikach<br />

ca̷lkowitych, którego wartość dla każdej liczby naturalnej jest liczba ↩<br />

pierwsza ↩<br />

.<br />

5.2.8. Poproś kogoś, aby wybra̷l dowolna ↩<br />

liczbe ↩<br />

naturalna ↩<br />

wie ↩<br />

ksza ↩<br />

od 10 i wykona̷l naste ↩<br />

puja ↩<br />

ce dzia̷lania:<br />

i) utworzy̷l sume ↩<br />

cyfr wybranej liczby i odja ↩̷l ja ↩<br />

od wybranej<br />

liczby;<br />

ii) w otrzymanej liczbie skreśli̷l pewna ↩<br />

niezerowa ↩<br />

cyfre ↩<br />

, a sume ↩<br />

pozosta̷lych cyfr poda̷l Tobie.<br />

Wtedy Ty jesteś w stanie wykryć skreślona ↩<br />

cyfre ↩<br />

. Podaj<br />

matematyczna ↩<br />

podstawe ↩<br />

regu̷ly wyznaczania ,,skreślonej cyfry”.


42 Cze ↩<br />

ść I – Zadania<br />

5.3. Kongruencje a równania. Niech W be ↩<br />

dzie wielomianem<br />

o wspó̷lczynnikach ca̷lkowitych, m be ↩<br />

dzie liczba ↩<br />

naturalna ↩<br />

wie ↩<br />

ksza ↩<br />

od jedynki i niech liczba ca̷lkowita c be ↩<br />

dzie pierwiastkiem<br />

wielomianu W (x) . Jeżeli r jest reszta ↩<br />

z dzielenia c przez m ,<br />

to c ≡ r(mod m ) , sta ↩<br />

d W (r) ≡ W (c) ≡ 0(mod m ) . Zatem,<br />

jeżeli istnieje liczba ca̷lkowita c taka, że W (c) = 0 , to istnieje<br />

r ∈ {0,1,2,. . . ,m − 1} takie, że W (r) ≡ 0(mod m ) .<br />

5.3.1. Udowodnij, że jeżeli wielomian W (x) o wspó̷lczynnikach<br />

ca̷lkowitych przybiera dla x = 0 i x = 1 wartości nieparzyste,<br />

to równanie W (x) = 0 nie ma pierwiastków ca̷lkowitych.<br />

5.3.2. Uzasadnij, że równanie<br />

100x 6 − 80x 5 + 36x 4 + 12x 3 + x 2 + 1001 = 0<br />

nie ma rozwia ↩<br />

zania w liczbach ca̷lkowitych.<br />

5.3.3. Udowodnij, że równanie 2x 2 − 215y 2 = 1 nie ma<br />

rozwia ↩<br />

zań w liczbach ca̷lkowitych.<br />

5.3.4. Wielomian o wspó̷lczynnikach ca̷lkowitych daje przy<br />

dzieleniu przez wielomian x 2 −12x+11 reszte ↩<br />

990x−889 . Wykaż,<br />

że wielomian ten nie ma pierwiastków ca̷lkowitych.<br />

5.4. Ma̷le Twierdzenie Fermata. Wiele obliczeń<br />

zwia ↩<br />

zanych z kongruencjami skraca<br />

Ma̷le Twierdzenie Fermata (MTF). Jeżeli liczba ca̷lkowita a<br />

jest niepodzielna przez liczbe ↩<br />

pierwsza ↩<br />

p , to a p−1 ≡ 1(mod p ) .<br />

Dowód. Niech r k oznacza reszte ↩<br />

z dzielenia liczby ak przez p (dla<br />

k ∈ {1,2,. . . p − 1} ). Wtedy ak ≡ r k (mod p ) . Sta ↩<br />

d<br />

a · 2a · 3a · · · (p − 1)a ≡ r 1 r 2 . . . r p−1 (mod p ). (*)<br />

Reszty r k sa ↩<br />

wszystkie różne, gdyż jeśli zachodzi̷laby równość r s =<br />

r l dla pewnych s i l , to wówczas as ≡ al(mod p ) , czyli p|(s−l)a ,<br />

co jest niemożliwe, gdyż p̸ | a i p̸ | s − l . Cia ↩<br />

g r 1 , r 2 , . . . , r p−1<br />

zawiera wie ↩<br />

c te same liczby co cia ↩<br />

g 1, 2, . . . , p − 1 , co najwyżej<br />

u̷lożone w innym porza ↩<br />

dku. Zatem r 1 r 2 . . . r p−1 = 1 · 2 · · · (p − 1) .


Kongruencje 43<br />

Sta ↩<br />

d wobec (*) mamy 1 · 2 · · · (p − 1)a p−1 ≡ 1 · 2 · · · (p − 1)(mod p ) ,<br />

czyli p|1 · 2 · · · (p − 1)(a p−1 − 1) . Ponieważ p̸ | 1 · 2 · · · (p − 1) , wie ↩<br />

c<br />

p|a p−1 − 1 , co dowodzi prawdziwości twierdzenia.<br />

Wniosek. Jeżeli p jest liczba ↩<br />

pierwsza ↩<br />

, to dla każdej liczby<br />

ca̷lkowitej a jest a p ≡ a(mod p ) .<br />

Dowód (wniosku). Jeżeli p̸ | a , to z MTF mamy a p−1 ≡ 1(mod p ) ,<br />

a ponieważ a ≡ a(mod p ) , wie ↩<br />

c po wymnożeniu stronami powyższych<br />

kongruencji mamy a p ≡ a(mod p ) .<br />

Jeżeli p|a , to p|a p − a , wie ↩<br />

c a p ≡ a(mod p ) .<br />

5.4.1. Stosuja ↩<br />

c metode ↩<br />

indukcji matematycznej, udowodnij,<br />

że dla dowolnej liczby naturalnej n jest n p ≡ n(mod p ) , gdzie p<br />

oznacza liczbe ↩<br />

pierwsza ↩<br />

.<br />

5.4.2. Udowodnij, że dla dowolnej liczby naturalnej n<br />

(a) 30|n 5 − n ;<br />

(b) 15|n 7 − n 5 − n 3 + n .<br />

5.4.3. Udowodnij, że każda liczba pierwsza p > 5 jest dzielnikiem<br />

liczby n p = 11 . . . 1 napisanej w uk̷ladzie dziesie ↩<br />

tnym za<br />

pomoca ↩<br />

p − 1 jedynek.<br />

5.4.4. Wykaż, że jeżeli liczby a 1 , a 2 , . . . , a k sa ↩<br />

liczbami<br />

ca̷lkowitymi, p zaś liczba ↩<br />

pierwsza ↩<br />

, to p|a p 1 + ap 2 + · · · + ap k<br />

wtedy<br />

i tylko wtedy, gdy p|a 1 + a 2 + · · · + a k .<br />

5.4.5. Udowodnij, że jeżeli liczby naturalne a , b , c i d<br />

spe̷lniaja ↩<br />

warunek a 2 + b 2 = c 2 + d 2 , to liczba a + b + c + d nie jest<br />

liczba ↩<br />

pierwsza ↩<br />

.<br />

5.4.6. Wykaż, że dla każdej liczby pierwszej p istnieje<br />

nieskończenie wiele liczb postaci 2 n − n podzielnych przez p , gdzie<br />

n ∈ N .<br />

5.4.7. Uzasadnij, że jeżeli a i b sa ↩<br />

liczbami ca̷lkowitymi, to<br />

liczba K = ab(a 2 − b 2 )(a 2 + b 2 ) jest podzielna przez 30.<br />

5.4.8. Niech p be ↩<br />

dzie liczba ↩<br />

pierwsza ↩<br />

. Pokaż, że dla każdego<br />

ca̷lkowitego a takiego, że p̸ | a , istnieje dok̷ladnie jedno b takie, że<br />

1 ≤ b ≤ p − 1 oraz ab ≡ 1(mod p ) .


44 Cze ↩<br />

ść I – Zadania<br />

5.4.9. Dana jest liczba pierwsza p > 3 . Wykaż równoważność<br />

zdań:<br />

(i) Istnieje taka liczba ca̷lkowita n , że liczba n 2 − n + 3 jest<br />

podzielna przez p .<br />

(ii) Istnieje taka liczba ca̷lkowita m , że liczba m 2 − m + 25<br />

jest podzielna przez p .<br />

5.4.10. Pokaż, że w cia ↩<br />

gu arytmetycznym postaci 4k + 1 istnieje<br />

nieskończenie wiele liczb pierwszych.<br />

Wskazówka. Przypuśćmy, że liczb pierwszych postaci 4k + 1 jest<br />

tylko skończona ilość. Niech to be ↩<br />

da ↩<br />

liczby p 1 , p 2 , . . . , p n .<br />

Rozważmy liczbe ↩<br />

K = 2p 1 p 2 . . . p n , a naste ↩<br />

pnie liczbe ↩<br />

K 2 + 1 .<br />

5.5. Pewne zastosowania twierdzenia Eulera. Uogólnieniem<br />

MTF jest naste ↩<br />

puja ↩<br />

ce<br />

Twierdzenie Eulera. Jeżeli liczby ca̷lkowite a i m sa ↩<br />

wzgle ↩<br />

dnie<br />

pierwsze, przy czym m > 1 , to a ϕ(m) ≡ 1(mod m ) , gdzie ϕ(m)<br />

oznacza ilość liczb naturalnych mniejszych od m i wzgle ↩<br />

dnie pierwszych<br />

z m .<br />

Dowód. Udowodnimy najpierw twierdzenie w przypadku, gdy m<br />

jest pote ↩<br />

ga ↩<br />

liczby pierwszej, tj. gdy m = p α . Zastosujemy indukcje ↩<br />

wzgle ↩<br />

dem α . Z MTF wynika, ze twierdzenie jest prawdziwe dla<br />

α = 1 . Za̷lóżmy, że twierdzenie jest prawdziwe dla α − 1 , tzn.<br />

a ϕ(pα−1) ≡ 1(mod p α−1 ) . Ponieważ ϕ(p α−1 ) = p α−1 − p α−2 , wie ↩<br />

c<br />

a pα−1 −p α−2<br />

= 1 + p α−1 b dla pewnej liczby ca̷lkowitej b . Podnosza ↩<br />

c<br />

obie strony ostatniej równości do pote ↩<br />

gi p mamy<br />

(<br />

a pα−1 −p α−2) p (<br />

= 1 + p α−1 b ) p<br />

( ( p p<br />

= 1 + p<br />

1)<br />

α−1 b + (p<br />

2)<br />

α−1 b) 2 + · · ·<br />

( ) p<br />

+ (p α−1 b) p−1 + (p α−1 b) p .<br />

p − 1<br />

Ponieważ wspó̷lczynniki ( (<br />

p<br />

1)<br />

,<br />

p<br />

) (<br />

2 , . . . , p<br />

p−1)<br />

sa↩ podzielne przez p<br />

(zobacz 5.1.11(a)), wie ↩<br />

c a pα −p α−1 = 1 + p α c , czyli a ϕ(pα )<br />


Kongruencje 45<br />

1(mod p α ) . Zatem twierdzenie jest prawdziwe dla pote ↩<br />

g liczb<br />

pierwszych.<br />

Niech teraz m = p α 1<br />

1 pα 2<br />

2 . . . pα s<br />

s . Z udowodnionej cze ↩<br />

ści twierdzenia<br />

wynika, że a ϕ(pα i<br />

)<br />

i ≡ 1(mod p α i<br />

i ) dla i ∈ {1,2,. . . ,s} .<br />

Podnosza ↩<br />

c do odpowiedniej pote ↩<br />

gi, mamy<br />

a ϕ(pα 1<br />

1 )ϕ(pα 2<br />

2 )...ϕ(pα s<br />

s<br />

) ≡ 1(mod p α i<br />

i ).<br />

Wykorzystuja ↩<br />

c multyplikatywność funkcji ϕ (zobacz 2.6.11) mamy<br />

a ϕ(m) ≡ 1(mod p α i<br />

i ) dla i ∈ {1,2,. . . ,s} . Z 5.1.5(d) wynika, że<br />

a ϕ(m) ≡ 1(mod m ) .<br />

Z twierdzenia Eulera wynika, że jeżeli liczby a i m sa ↩<br />

wzgle ↩<br />

dnie pierwsze, to kongruencja wyk̷ladnicza a x ≡ 1(mod m )<br />

ma zawsze pierwiastek naturalny, mianowicie x = ϕ(m) . Kongruencja<br />

a x ≡ 1(mod m ) może mieć pierwiastek mniejszy od<br />

ϕ(m) . Na przyk̷lad pierwiastkiem kongruencji 10 x ≡ 1(mod 41 )<br />

jest liczba 5 mniejsza od ϕ(41) = 40 .<br />

Oznaczmy przez w m (a) najmniejsza ↩<br />

liczbe ↩<br />

naturalna ↩<br />

, taka ↩<br />

że<br />

a wm(a) ≡ 1(mod m ) . Liczba w m (a) nazywa sie ↩<br />

wyk̷ladnikiem, do<br />

jakiego należy a wed̷lug modu̷lu m . Zauważmy, że jeżeli a k ≡<br />

1(mod m ) , to w m (a)|k . Istotnie, jeżeli k = qw m (a) + r , gdzie<br />

0 ≤ r < w m (a) (dzielenie z reszta ↩<br />

), to 1 ≡ a qwm(a)+r ≡ ( a ) w q<br />

m(a) ·<br />

a r ≡ 1 q a r ≡ a r (mod m ) . Sta ↩<br />

d wobec minimalności w m (a) i nierówności<br />

r < w m (a) musi być r = 0 , tzn. w m (a)|k . Z powyższych<br />

rozważań i twierdzenia Eulera otrzymujemy naste ↩<br />

puja ↩<br />

cy<br />

Wniosek. Jeżeli liczba a jest wzgle ↩<br />

dnie pierwsza z m oraz a k ≡<br />

1(mod m ) , to w m (a)|k . W szczególności w m (a)|ϕ(m) .<br />

5.5.1. Niech NWD(a, m) = 1 i niech r be ↩<br />

dzie reszta ↩<br />

z dzielenia<br />

n przez ϕ(m) . Pokaż, że a n ≡ a r (mod m ) .<br />

5.5.2. Niech a k ≡ 1(mod m ) i niech a l ≡ 1(mod m ) .<br />

Pokaż, że a NWD(k,l) ≡ 1(mod m ) .<br />

5.5.3. Znajdź reszte ↩<br />

z dzielenia<br />

(a) 317 259 przez 15;<br />

(b) 7 67 przez 12;<br />

(c) 4 113 przez 92;<br />

(d) 2 1000000 przez 77.


46 Cze ↩<br />

ść I – Zadania<br />

5.5.4. Pokaż, że jeżeli n = 73 · 37 , to 2 n−1 ≡ 1(mod n ) .<br />

5.5.5. Wykaż, że dla liczby naturalnej n ≥ 3 , liczba n 7 −<br />

14n 5 + 49n 3 − 36n dzieli sie ↩<br />

przez 20.<br />

5.5.6. Wyznacz wszystkie wartości naturalne n , dla których<br />

2 n − 1 jest liczba ↩<br />

podzielna ↩<br />

przez 7.<br />

5.5.7. Udowodnij, że dla dowolnej liczby nieparzystej m<br />

równanie 2 x = my +1 ma nieskończenie wiele rozwia ↩<br />

zań w liczbach<br />

naturalnych x , y .<br />

5.5.8. Wykaż, że iloczyn trzech kolejnych liczb naturalnych,<br />

z których środkowa jest sześcianem liczby naturalnej wie ↩<br />

kszej od 1,<br />

jest podzielny przez 504.<br />

5.5.9. Udowodnij twierdzenie Eulera wzoruja ↩<br />

c sie ↩<br />

na dowodzie<br />

MTF znajduja ↩<br />

cego sie ↩<br />

w 5.4.<br />

5.6. Rozwinie ↩<br />

cie okresowe a kongruencje. Rozwinie ↩<br />

cie<br />

dziesie ↩<br />

tne u̷lamka 5<br />

jak w Obliczeniu 1.<br />

13<br />

otrzymujemy przez ,,nieprzerwane” dzielenie<br />

0 ,384615(384615)<br />

5 :13<br />

5 0<br />

3 9<br />

1 1 0<br />

1 0 4<br />

60<br />

52<br />

80<br />

78<br />

20<br />

13<br />

70<br />

65<br />

5<br />

.<br />

Obliczenie 1.


Kongruencje 47<br />

Gdy tylko wysta ↩<br />

pi reszta 5, ca̷ly rachunek, który zacza ↩̷l sie ↩<br />

od 5,<br />

powtórzy sie ↩<br />

od pocza ↩<br />

tku, a zatem cia ↩<br />

g reszt 5, 11, 6, 8, 2, 7 i cia ↩<br />

g<br />

ilorazów 3, 8, 4, 6, 1, 5 be ↩<br />

dzie sie ↩<br />

stale powtarza̷l.<br />

5<br />

Zamieniaja ↩<br />

c<br />

13<br />

na u̷lamek okresowy, dokonaliśmy dzieleń<br />

przedstawionych w Obliczeniach 2.<br />

10 · 5 = 13 · 3 + 11<br />

10 · 11 = 13 · 8 + 6<br />

10 · 6 = 13 · 4 + 8<br />

10 · 8 = 13 · 6 + 2<br />

10 · 2 = 13 · 1 + 7<br />

10 · 7 = 13 · 5 + 5<br />

Obliczenia 2.<br />

Ogólnie, aby znaleźć rozwinie ↩<br />

cie dziesie ↩<br />

tne w̷laściwego nieskracalnego<br />

u̷lamka<br />

a b , którego mianownik jest wzgle ↩dnie pierwszy<br />

z liczba ↩<br />

10, dokonujemy naste ↩<br />

puja ↩<br />

cych dzieleń:<br />

10a = bq 1 + r 1<br />

10r 1 = bq 2 + r 2<br />

. . . . . . . . . . . . . . . . . .<br />

10r k−1 = bq k + r k<br />

(*)<br />

gdzie r i < b oraz q i < 10 dla i ∈ {1,2,. . . ,k} . Ponieważ<br />

NWD(10, b) = 1 i NWD(a, b) = 1 , wie ↩<br />

c NWD(r 1 , b) = 1 , a sta ↩<br />

d kolejno<br />

otrzymujemy, że NWD(r 2 , b) = NWD(r 3 , b) = · · · = NWD(r k , b) = 1 .<br />

5.6.1. Wykorzystuja ↩<br />

c powyższe oznaczenia, uzasadnij, że<br />

jeżeli liczba równości wyste ↩<br />

puja ↩<br />

cych w (*) jest równa wyk̷ladnikowi<br />

do jakiego należy 10 wed̷lug modu̷lu b , to r k = a oraz q k+1 = q 1 .<br />

5.6.2. Uzasadnij, że liczba cyfr w okresie rozwinie ↩<br />

cia w systemie<br />

o podstawie 10 nieskracalnego w̷laściwego u̷lamka a b , którego<br />

mianownik jest wzgle ↩<br />

dnie pierwszy z liczba ↩<br />

10, dzieli liczbe ↩<br />

ϕ(b)<br />

i jest równa najmniejszemu naturalnemu pierwiastkowi kongruencji<br />

10 x ≡ 1(mod b ) .


48 Cze ↩<br />

ść I – Zadania<br />

5.6.3. Znajdź ilość cyfr w okresie rozwinie ↩<br />

cia 1 5 w systemie<br />

o podstawie 2.<br />

5.7. Zastosowania twierdzenia Wilsona. Poznamy<br />

teraz pewne fakty pozwalaja ↩<br />

ce scharakteryzować liczby pierwsze.<br />

Twierdzenie Wilsona. Jeżeli p jest liczba ↩<br />

pierwsza ↩<br />

, to<br />

(p − 1)! ≡ −1(mod p ).<br />

Dowód. Niech p > 2 . Z zadania 5.4.8 wynika, że dla każdego<br />

a ∈ {1,2,. . . ,p − 1} istnieje b ∈ {1,2,. . . ,p − 1} takie, że ab ≡<br />

1(mod p ) . Nazwijmy liczbe ↩<br />

b ,,odpowiednia ↩<br />

” 1 dla liczby a .<br />

Oczywiste jest, że jeżeli b jest ,,odpowiednia” dla a , to a jest<br />

,,odpowiednia” dla b . Zauważmy, że dla 1, ,,odpowiednia ↩<br />

” jest 1,<br />

a dla p−1 , ,,odpowiednia ↩<br />

” jest p−1 . Oprócz 1 i p−1 w cia ↩<br />

gu liczb<br />

1 , 2 , . . . , p − 1 nie ma innej liczby, która by by̷la równa swojej<br />

,,odpowiedniej”. Istotnie, gdyby a by̷lo taka ↩<br />

liczba ↩<br />

, to p|a 2 − 1 ,<br />

czyli p|a+1 lub p|a−1 . Nie jest to możliwe, gdy 3 ≤ a+1 ≤ p−1<br />

i 1 ≤ a − 1 ≤ p − 3 . Zatem liczby cia ↩<br />

gu 2 , 3 , . . . , p − 2 ,<br />

gdzie p > 2 rozpadaja ↩<br />

sie ↩<br />

na pary różnych liczb ,,odpowiednich”:<br />

m 1 , n 1 ; m 2 , n 2 ; . . . ; m p−3 , n p−3 . Ponieważ m i n i ≡ 1(mod p )<br />

2 2<br />

dla i ∈ { 1,2,. . . , p−3 }<br />

2 , wie↩ c<br />

m 1 n 1 · m 2 n 2 · · · · · m p−3 n p−3<br />

2 2<br />

· 1 · (p − 1) ≡ p − 1(mod p ).<br />

Czynniki iloczynu m 1 n 1 · m 2 n 2 · · · · · m p−3 n p−3 · 1 · (p − 1) różnia ↩<br />

sie ↩<br />

2 2<br />

od czynników iloczynu 1 · 2 · · · · · (p − 1) tylko porza ↩<br />

dkiem. Zatem<br />

(p − 1)! ≡ p − 1 ≡ −1(mod p ).<br />

Twierdzenie jest też prawdziwe dla p = 2 , gdyż<br />

(2 − 1)! = 1 ≡ −1(mod 2 ).<br />

5.7.1. Uzasadnij, że 18! + 1 ≡ 0(mod 23 ) .<br />

1 Czytelnik obeznany z arytmetyka↩ modulo zapewne zauważy̷l, że liczba ,,odpowiednia”<br />

dla liczby a jest odwrotna ↩ modulo p do a .


Kongruencje 49<br />

5.7.2. Uzasadnij, że liczba 712! + 1 jest z̷lożona.<br />

5.7.3. Udowodnij, że jeżeli liczba n > 1 jest taka, że (n−1)! ≡<br />

−1(mod n ) , to n jest liczba ↩<br />

pierwsza ↩<br />

.<br />

5.7.4. Udowodnij, że liczba naturalna p > 1 jest liczba ↩<br />

pierwsza ↩<br />

wtedy i tylko wtedy, gdy (p − 2)! ≡ 1(mod p ) .<br />

5.7.5. Udowodnij, że istnieje nieskończenie wiele liczb naturalnych<br />

n , dla których liczba n! + 1 jest z̷lożona.<br />

5.7.6. Udowodnij, że jeżeli liczba n jest z̷lożona i n ≠ 4 , to<br />

(n − 1)! ≡ 0(mod n ).<br />

5.7.7. Znajdź wszystkie liczby naturalne nieparzyste n , dla<br />

których zachodzi (n − 1)! ≢ 0(modn 2 ) .<br />

5.8. Jeszcze jedno twierdzenie o kongruencjach.<br />

Przed rozpocze ↩<br />

ciem studiowania dowodu kolejnego twierdzenia, proponujemy<br />

naste ↩<br />

puja ↩<br />

ca ↩<br />

zabawe ↩<br />

:<br />

Poproś kogoś, żeby wybra̷l liczbe ↩<br />

naturalna ↩<br />

mniejsza ↩<br />

od 60 i wykona̷l naste ↩<br />

puja ↩<br />

ce czynności:<br />

(i) podzieli̷l ja ↩<br />

przez 3 i poda̷l reszte ↩<br />

. Niech ta ↩<br />

reszta ↩<br />

be ↩<br />

-<br />

dzie a ;<br />

(ii) podzieli̷l te ↩<br />

liczbe ↩<br />

przez 4 i poda̷l reszte ↩<br />

. Niech ta ↩<br />

reszta ↩<br />

be ↩<br />

dzie b ;<br />

(iii) podzieli̷l te ↩<br />

liczbe ↩<br />

przez 5 i poda̷l reszte ↩<br />

. Niech ta ↩<br />

reszta ↩<br />

be ↩<br />

dzie c .<br />

Wybrana liczba jest reszta ↩<br />

otrzymana ↩<br />

z dzielenia<br />

liczby 40 a + 45 b + 36 c przez 60.<br />

Czy jesteś w stanie podać matematematyczna ↩<br />

podstawe ↩<br />

przytoczonej<br />

zabawy Jeżeli nie, to przestudiuj dowód twierdzenia<br />

zamieszczonego niżej.<br />

Chińskie twierdzenie o resztach. Niech n 1 , n 2 , . . . , n r be ↩<br />

da ↩<br />

liczbami naturalnymi parami wzgle ↩<br />

dnie pierwszymi, n = n 1 n 2 . . . n r<br />

oraz a 1 , a 2 , . . . , a r sa ↩<br />

dowolnymi liczbami ca̷lkowitymi. Wówczas


50 Cze ↩<br />

ść I – Zadania<br />

istnieje dok̷ladnie jedna liczba ca̷lkowita x należa ↩<br />

ca do przedzia̷lu<br />

[1,n] , taka że x ≡ a i (mod n i ) dla i ∈ {1,2,. . . ,r} .<br />

Dowód. Oznaczmy iloczyn n 1 n 2 . . . n i−1 n i+1 . . . n r przez m i . Ponieważ<br />

NWD(n i , n j ) = 1 dla i ≠ j , wie ↩<br />

c NWD(m i , n i ) = 1 . Istnieja ↩<br />

zatem liczby ca̷lkowite k i , l i , takie że k i m i + l i n i = 1 . Wówczas<br />

k i m i ≡ 1(mod n i ) oraz k i m i ≡ 0(mod n j ) dla j ≠ i . Oznaczmy<br />

przez x 0 liczbe ↩<br />

a 1 k 1 m 1 + a 2 k 2 m 2 + · · · + a i k i m i + · · · + a r k r m r .<br />

Z powyższych kongruencji wynika, że x 0 ≡ a i k i m i ≡ a i (mod n i )<br />

dla dowolnego i ∈ {1,2,. . . ,r} . Niech x be ↩<br />

dzie reszta ↩<br />

z dzielenia<br />

x 0 przez n , czyli x 0 ≡ x(mod n i ) . Zatem x ≡ a i (mod n i ) .<br />

Pokazaliśmy, że ża ↩<br />

dana liczba x istnieje.<br />

Należy jeszcze pokazać, że w przedziale [1,n] istnieje dok̷ladnie<br />

jedna taka liczba. Przypuśćmy, że x,˜x ∈ [1,n] sa ↩<br />

rozwia ↩<br />

zaniami<br />

rozważanego uk̷ladu kongruencji, czyli x ≡ a i (mod n i ) oraz ˜x ≡<br />

a i (mod n i ) dla i ∈ {1,2,. . . ,r} . Wówczas ˜x − x ≡ 0(mod n i ) dla<br />

i ∈ {1,2,. . . ,r} , sta ↩<br />

d ˜x ≡ x(mod n ) . Zatem x = ˜x .<br />

5.8.1. Analizuja ↩<br />

c dowód chińskiego twierdzenia o resztach, podaj<br />

matematyczna ↩<br />

podstawe ↩<br />

zabawy przytoczonej na pocza ↩<br />

tku 5.8.<br />

5.8.2. Oblicz, ile jest liczb naturalnych mniejszych od 2000,<br />

które przy dzieleniu przez 11 daja ↩<br />

reszte ↩<br />

1, a przy dzieleniu przez 4<br />

daja ↩<br />

reszte ↩<br />

3.<br />

5.8.3. Znajdź liczbe ↩<br />

trzycyfrowa ↩<br />

(w systemie dziesie ↩<br />

tnym),<br />

która daje reszte ↩<br />

4 przy dzieleniu przez 7, 9 i 11.<br />

5.8.4. Znajdź najmniejsza ↩<br />

liczbe ↩<br />

dodatnia ↩<br />

, która daje reszte ↩<br />

1 przy dzieleniu przez 11, reszte ↩<br />

2 przy dzieleniu przez 12 i reszte ↩<br />

3<br />

przy dzieleniu przez 13.<br />

5.8.5. Pokaż, że dla dowolnych liczb naturalnych parami<br />

wzgle ↩<br />

dnie pierwszych p , q i r równanie x p + y q = z r ma<br />

rozwia ↩<br />

zanie w liczbach naturalnych x , y , z .<br />

a n+1<br />

5.8.6. Niech dane be ↩<br />

da ↩<br />

liczby naturalne a 1 , a 2 , . . . , a n ,<br />

parami wzgle ↩<br />

dnie pierwsze. Udowodnij, że równanie<br />

x a 1<br />

1 + xa 2<br />

2 + · · · + xa n<br />

n = x a n+1<br />

n+1<br />

ma nieskończenie wiele rozwia ↩<br />

zań w liczbach naturalnych.


CZE ↩ ŚĆ II – ROZWIA ↩<br />

ZANIA<br />

1. Podstawowe w̷lasności liczb ca̷lkowitych<br />

1.1. Podzielność liczb ca̷lkowitych<br />

1.1.1. Zauważmy, że jeśli pomnożymy 5 przez liczbe ↩<br />

parzysta ↩<br />

,<br />

to otrzymamy liczbe ↩<br />

z zerem na końcu, a jeśli pomnożymy 5 przez<br />

liczbe ↩<br />

nieparzysta ↩<br />

, to ostatnia ↩<br />

cyfra ↩<br />

iloczynu be ↩<br />

dzie 5. Istotnie,<br />

5 · (2k + 1) = k · 10 + 5 , a ponieważ ostatnia ↩<br />

cyfra ↩<br />

k · 10 jest 0,<br />

wie ↩<br />

c k · 10 + 5 ma na końcu 5. Sta ↩<br />

d wynika, że jeśli liczba n dzieli<br />

sie ↩<br />

przez 5, to jej ostatnia ↩<br />

cyfra ↩<br />

jest 0 lub 5. Zatem 5̸ | 12354 , ale<br />

5|12345 .<br />

1.1.2. Jeżeli m|a , to istnieje taka liczba ca̷lkowita n , że m ·<br />

n = a . Ale wtedy m · (−n) = −a . Ponieważ −n jest liczba ↩<br />

ca̷lkowita ↩<br />

, wie ↩<br />

c m|(−a) .<br />

1.1.3. Jeśli m|a , to istnieje liczba ca̷lkowita n taka, że mn =<br />

a . Ale wtedy m · (nb) = ab . Ponieważ nb jest liczba ↩<br />

ca̷lkowita ↩<br />

,<br />

wie ↩<br />

c mamy m|ab .<br />

1.1.4. Ponieważ 15|225 , wie ↩<br />

c 15|3 · 225 , a zatem 15|675 . Co<br />

wie ↩<br />

cej, 15|6000 (bo 15·400 = 6000 ) oraz 5775 = 6000−225 . Z za-<br />

̷lożenia wiemy, że istnieje liczba ca̷lkowita n taka, że 15 · n = 225 .<br />

Mamy zatem 5775 = 15(400 − n) . Ponieważ 400 − n jest liczba ↩<br />

ca̷lkowita ↩<br />

, wie ↩<br />

c 15|5775 .<br />

1.1.5. Nie. Niech m = 6 , a = 2 i b = 3 . Wówczas m̸ | a ani<br />

m̸ | b , chociaż m|ab .


52 Cze ↩<br />

ść II – Rozwia ↩<br />

zania<br />

1.1.6. Jeżeli m|a oraz m|b , to istnieja ↩<br />

takie liczby ca̷lkowite<br />

n i k , że mn = a oraz mk = b . Sta ↩<br />

d a + b = m(n + k) , wie ↩<br />

c<br />

m|a + b . Podobnie a − b = m(n − k) , ska ↩<br />

d m|a − b .<br />

1.1.7. Mamy 770 = 784 − 14 oraz 812 = 784 + 28 . Zatem<br />

z poprzedniego zadania wynika, że 14|770 oraz 14|812 .<br />

1.1.8. Skoro istnieje taka liczba ca̷lkowita n , że 14n = 784 ,<br />

to 7 · 2n = 784 . Zatem 7|784 . Z poprzedniego zadania wynika<br />

też, że 7|812 (bo 14|812 ). Ale gdyby 7 dzieli̷lo 817, to dzieli̷loby<br />

też różnice ↩<br />

817 − 812 = 5 . Ponieważ 7̸ | 5 , wie ↩<br />

c mamy sprzeczność.<br />

Czyli 7̸ | 817 .<br />

1.1.9. 790 nie wydaje sie ↩<br />

być podzielna ↩<br />

przez 14, ponieważ<br />

wtedy 790 − 784 = 6 dzieli̷loby sie ↩<br />

przez 14, ale to nie jest prawda.<br />

1.1.10. Za̷lóżmy, że 56|m oraz m > 2576 . Wtedy 56|m −<br />

2576 . Szukamy wie ↩<br />

c najmniejszej liczby dodatniej, która sie ↩<br />

dzieli<br />

przez 56. Jest to oczywiście 56. Zatem najmniejsza ↩<br />

po 2576 liczba ↩<br />

podzielna ↩<br />

przez 56 jest 2576 + 56 = 2632 .<br />

1.1.11. Jeśli 7|m , to 7|m − 315 . Zatem m − 315 jest jedna ↩<br />

z liczb 7, 14, 21, 28 itd. lub 0, −7 , −14 , −21 itd. Zatem liczby<br />

z przedzia̷lu (290,340) podzielne przez 7, to 294, 301, 308, 315, 322,<br />

329 i 336.<br />

1.1.12. Z tego, że m|a + b , nie można wiele wywnioskować.<br />

Jeśli bowiem m = 5 , a = 2 oraz b = 3 , to nie zachodzi ani<br />

koniunkcja m|a i m|b , ani alternatywa m|a lub m|b .<br />

1.1.13. Jeśli m|a+b oraz m|a−b , to wtedy m|(a+b)+(a−b)<br />

i m|(a + b) − (a − b) , wie ↩<br />

c m|2a oraz m|(−2b) (sta ↩<br />

d m|2b ).<br />

Za̷lóżmy, że m jest liczba ↩<br />

nieparzysta ↩<br />

, powiedzmy m = 2k + 1<br />

dla pewnego k , i m|2a . Istnieje zatem n ca̷lkowite, takie że mn =<br />

2kn + n = 2a . Ponieważ n = 2a − 2kn = 2(a − kn) , wie ↩<br />

c n jest<br />

liczba ↩<br />

parzysta ↩<br />

. Zatem n 2<br />

jest liczba ↩<br />

ca̷lkowita ↩<br />

oraz m · n<br />

2 = a .<br />

Zatem m|a . Podobnie pokazujemy, że m|b (jeśli m jest liczba ↩<br />

nieparzysta ↩<br />

).<br />

Jeśli m jest liczba ↩<br />

parzysta ↩<br />

, to wtedy m|2a nie implikuje<br />

m|a .


Podstawowe w̷lasności liczb ca̷lkowitych 53<br />

1.1.14. Jeśli m|a oraz n|m , to istnieja ↩<br />

liczby ca̷lkowite k<br />

oraz l takie, że mk = a i nl = m . Zatem n(lk) = a , czyli n|a .<br />

1.1.15. Jeżeli m|a , to istnieje takie n ca̷lkowite, że mn = a .<br />

Z w̷lasności wartości bezwzgle ↩<br />

dnej mamy |m||n| = |a| . Ponieważ<br />

a ≠ 0 , wie ↩<br />

c n ≠ 0 . Zatem |m| = |a|<br />

|n| ≤ |a| .<br />

1.1.16. Z poprzedniego zadania mamy |m| ≤ |a| oraz |a| ≤<br />

|m| . Zatem |a| = |m| , czyli m = a lub m = −a .<br />

1.2. Zasada indukcji matematycznej<br />

1.2.1. Rozważmy zdanie T (n) mówia ↩<br />

ce o liczbie naturalnej<br />

n . Zak̷ladamy, że T (m) jest zdaniem prawdziwym dla pewnego<br />

m ∈ N 0 oraz że ze zdania T (k) wynika zdanie T (k+1) dla k ≥ m .<br />

Niech X = {n ∈ N m : T (n) jest fa̷lszywe} . Należy pokazać, że<br />

X = ∅ .<br />

Przypuśćmy, że X ≠ ∅ . Z zasady minimum wynika, że<br />

w zbiorze X istnieje liczba najmniejsza. Niech l be ↩<br />

dzie ta ↩<br />

liczba ↩<br />

.<br />

Ponieważ T (m) jest zdaniem prawdziwym, wie ↩<br />

c l > m . Liczba<br />

l − 1 /∈ X , wie ↩<br />

c zdanie T (l − 1) jest prawdziwe. Jednakże<br />

z prawdziwości zdania T (l − 1) wynika natychmiast prawdziwość<br />

zdania T (l) . Zatem l /∈ X . Otrzymana sprzeczność dowodzi<br />

prawdziwości zasady indukcji.<br />

1.2.2. Rozważmy zbiór M = { n ∈ N 0 : 3|n 3 + 5n } .<br />

1 0 Ponieważ 3|0 , wie ↩<br />

c 0 ∈ M .<br />

2 0 Przypuśćmy, że k ∈ M , tj. że 3|k 3 + 5k . Uzasadnimy,<br />

że k + 1 ∈ M , czyli że<br />

3|(k + 1) 3 + 5(k + 1). (*)<br />

Z za̷lożenia mamy, że k 3 + 5k = 3t dla pewnego t ∈ N 0 .<br />

Zatem<br />

(k + 1) 3 + 5(k + 1) = k 3 + 5k + 3(k 2 + k + 2)<br />

= 3(t + k 2 + k + 2),<br />

sta ↩<br />

d (*).<br />

Na mocy zasady indukcji, M = N 0 , co oznacza, że 3|n 3 + 5n dla<br />

dowolnego n ∈ N 0 .


54 Cze ↩<br />

ść II – Rozwia ↩<br />

zania<br />

1.2.3.<br />

1 0 Jeżeli n = 2 , to mamy 2 22 + 1 = 17 . Ostatnia ↩<br />

cyfra ↩<br />

tej<br />

liczby jest wie ↩<br />

c 7.<br />

2 0 Jeżeli ostatnia ↩<br />

cyfra ↩<br />

liczby 2 2k + 1 jest 7, to 2 2k + 1 =<br />

10q + 7 , czyli 2 2k = 10q + 6 dla pewnego q ∈ N 0 . Sta ↩<br />

d<br />

2 2k+1 + 1 = 2 2k·2 + 1 =<br />

= (10q + 6) 2 + 1<br />

(<br />

2 2k) 2<br />

+ 1<br />

= 100q 2 + 120q + 36 + 1<br />

= 10(10q 2 + 12q + 3) + 7,<br />

co oznacza, że ostatnia ↩<br />

cyfra ↩<br />

liczby 2 2k+1 jest 7.<br />

Na mocy zasady indukcji otrzymujemy, że ostatnia ↩<br />

cyfra ↩<br />

liczby 2 2n +<br />

1 jest 7, jeśli n ∈ N 2 .<br />

1.2.4. Wynika bezpośrednio z zadania 1.2.3.<br />

1.2.5.<br />

1 0 Jeżeli n = 1 , to 2 41 − 5 = 11 . Ostatnia ↩<br />

cyfra ↩<br />

liczby 11<br />

jest 1.<br />

2 0 Jeżeli ostatnia ↩<br />

cyfra ↩<br />

liczby 2 4k − 5 jest 1, to 2 4k − 5 =<br />

10q + 1 . Sta ↩<br />

d<br />

2 4k+1 − 5 = 2 4k·4 − 5<br />

= (10q + 6) 4 − 5<br />

= 10000q 4 + 24000q 3 + 21600q 2<br />

+ 8640q + 1296 − 5<br />

= 10(1000q 4 + 2400q 3 + 2160q 2<br />

+ 864q + 129) + 1,<br />

co oznacza, że ostatnia ↩<br />

cyfra ↩<br />

2 4k +1 − 5 jest 1.<br />

Na mocy zasady indukcji otrzymujemy, że ostatnia ↩<br />

cyfra ↩<br />

liczby 2 4n −<br />

5 jest 1 dla n ∈ N 1 .


Podstawowe w̷lasności liczb ca̷lkowitych 55<br />

1.3. Dzielenie z reszta ↩<br />

1.3.1.<br />

(a) 23 = 3 · 7 + 2 ;<br />

(b) 43 = 4 · 10 + 3 ;<br />

(c) 36 = 12 · 3 + 0 .<br />

1.3.2. Rozważamy zbiór X = {n − dk ∈ N 0 : k ∈ Z} . Zbiór<br />

ten nie jest pusty. Istotnie, jeśli n ≥ 0 , to n = n − d · 0 ∈ X ,<br />

a jeżeli n < 0 , to n − nd = n(1 − d) ∈ X . Z zasady minimum<br />

wynika, że w X istnieje liczba najmniejsza. Niech r be ↩<br />

dzie ta ↩<br />

liczba ↩<br />

. Ponieważ r ∈ X , wie ↩<br />

c istnieje liczba ca̷lkowita q taka, że<br />

r = n − dq , ska ↩<br />

d n = dq + r , gdzie r ≥ 0 . Zauważmy, że r < d .<br />

Gdyby tak nie by̷lo, tj. r ≥ d , to wtedy<br />

0 ≤ r − d = n − dq − d = n − d(q + 1) ∈ X<br />

oraz r − d < r . By̷loby to sprzeczne z wyborem liczby r . Zatem<br />

r < d .<br />

Pozosta̷lo jeszcze pokazać, że istnieje tylko jedna para liczb<br />

ca̷lkowitych q , r taka, że n = dq + r , gdzie 0 ≤ r < d .<br />

Przypuśćmy nie wprost, że n = dq 1 + r 1 = dq 2 + r 2 , przy czym<br />

0 ≤ r 1 < d , 0 ≤ r 2 < d . Jeżeli np. r 1 < r 2 , to<br />

Ale r 2 − r 1 = d(q 1 − q 2 ) , wie ↩<br />

c<br />

0 < r 2 − r 1 < d. (1)<br />

d|r 2 − r 1 . (2)<br />

Wyrażenia (1) oraz (2) nie moga ↩<br />

być jednocześnie spe̷lnione, wie ↩<br />

c<br />

otrzymujemy sprzeczność. Podobnie rozumujemy, gdy r 1 > r 2 .<br />

Zatem musi być r 1 = r 2 . Co za tym idzie, to 0 = r 2 − r 1 =<br />

d(q 1 − q 2 ) , czyli wobec d ≠ 0 , mamy q 1 = q 2 .<br />

1.3.3.<br />

(2n + 1) 2 = 4n 2 + 4n + 1 = 4n(n + 1) + 1.<br />

Liczba n(n + 1) dzieli sie ↩<br />

przez 2, wie ↩<br />

c jest postaci 2k , k ∈ Z .<br />

Zatem (2n + 1) 2 = 8k + 1 .


56 Cze ↩<br />

ść II – Rozwia ↩<br />

zania<br />

1.3.4. n 2 +(n+1) 2 = n 2 +n 2 +2n+1 = 2n(n+1)+1 . Liczba<br />

n(n + 1) jest postaci 2q , gdzie q ∈ N , wie ↩<br />

c n 2 + (n + 1) 2 = 4q + 1 .<br />

1.3.5. Przypuśćmy, że istnieje liczba ca̷lkowita x taka, że<br />

3m + 2 = x 2 . Dowolna liczba ca̷lkowita jest postaci 3k , 3k + 1 lub<br />

3k + 2 ( k ∈ Z ). Jeśli x = 3k , to mamy 3m + 2 = 9k 2 , a sta ↩<br />

d<br />

3(3k 2 − m) = 2 , co oczywiście oznacza, że 3|2 . Otrzymaliśmy<br />

sprzeczność. Podobnie dochodzimy do sprzeczności zak̷ladaja ↩<br />

c, że<br />

x = 3k + 1 lub x = 3k + 2 .<br />

1.3.6.<br />

(2q 1 + 1) 2 + (2q 2 + 1) 2 = 4q 2 1 + 4q 1 + 1 + 4q 2 2 + 4q 2 + 1<br />

= 4(q 2 1 + q 2 2 + q 1 + q 2 ) + 2.<br />

Wystarczy zatem pokazać, że liczba ca̷lkowita postaci 4k + 2 nie<br />

jest kwadratem liczby ca̷lkowitej.<br />

Przypuśćmy, że istnieje liczba ca̷lkowita x taka, że 4k + 2 =<br />

x 2 . Jeśli x jest liczba ↩<br />

parzysta ↩<br />

, tj. x = 2l , to x 2 = 4l 2 , wie ↩<br />

c<br />

4(k − l 2 ) = −2 , czyli 4|2 . Otrzymaliśmy wie ↩<br />

c sprzeczność. Jeżeli<br />

x = 2l + 1 , to 4k + 2 = (2l + 1) 2 , czyli 4(k − l 2 − l) = −1 . Zatem<br />

4|1 i znowu otrzymujemy sprzeczność.<br />

1.4. Cze ↩ ść ca̷lkowita<br />

1.4.1. Z określenia cze ↩<br />

ści ca̷lkowitej wynika, że [x] ≤ x , a z za-<br />

̷lożenia, że x ≤ y . Zatem [x] ≤ y . Z ostatniej nierówności wynika,<br />

że [x] jest liczba ↩<br />

ca̷lkowita ↩<br />

nie wie ↩<br />

ksza ↩<br />

od y . Ponieważ [y] jest<br />

najwie ↩<br />

ksza ↩<br />

spośród takich liczb ca̷lkowitych, wie ↩<br />

c [x] ≤ [y] .<br />

1.4.2. Ponieważ −n ≤ −n+α < −n+1 , wie ↩<br />

c [−n+α] = −n .<br />

1.4.3.<br />

(a) Zauważmy, że jeżeli x jest liczba ↩<br />

ca̷lkowita ↩<br />

, to −x jest<br />

także liczba ↩<br />

ca̷lkowita ↩<br />

. Mamy zatem [x] = x oraz [−x] =<br />

−x . Sta ↩<br />

d [−x] = −x = −[x] .<br />

(b) Jeżeli x nie jest liczba ↩<br />

ca̷lkowita ↩<br />

, to x = [x] + α , gdzie<br />

α ∈ (0,1) . Wtedy<br />

−x = −[x] − α = −[x] − 1 + (1 − α).


Podstawowe w̷lasności liczb ca̷lkowitych 57<br />

Ponieważ 1 − α ∈ (0,1) , wie ↩<br />

c [−x] = −[x] − 1 (zobacz 1.4.2).<br />

(c) Zapiszmy x w postaci x = [x] + {x} . Wówczas<br />

[x + n] = [[x] + n + {x}] = [x] + n.<br />

1.4.4. Ponieważ x = [x] + α oraz y = [y] + β , gdzie α,β ∈<br />

[0,1) , wie ↩<br />

c x + y = [x] + [y] + α + β . Tutaj α + β ∈ [0,1) ba ↩<br />

dź<br />

α + β ∈ [1,2) . Sta ↩<br />

d<br />

{<br />

[x] + [y], gdy α + β ∈ [0,1)<br />

[x + y] =<br />

[x] + [y] + 1, gdy α + β ∈ [1,2) ,<br />

czyli [x + y] ≥ [x] + [y] .<br />

1.4.5. Niech x = [x] + α oraz y = [y] + β , gdzie α,β ∈ [0,1) .<br />

Ponieważ [x] = [y] , wie ↩<br />

c<br />

|x − y| = |α − β| < 1.<br />

[ ]<br />

1.4.6. Liczba [x]<br />

n<br />

jest liczba ↩<br />

ca̷lkowita ↩<br />

. Oznaczmy ja ↩<br />

przez α . Mamy zatem nierówność<br />

czyli nierówność<br />

α ≤ [x]<br />

n < α + 1,<br />

αn ≤ [x] < n(α + 1).<br />

Ale x = [x] + β , gdzie β ∈ [0,1) oraz αn i n(α + 1) sa ↩<br />

liczbami<br />

ca̷lkowitymi, wie ↩<br />

c<br />

αn ≤ x < n(α + 1).<br />

Sta ↩<br />

d α ≤ x n < α + 1 , wie ↩c [ x<br />

n]<br />

= α .<br />

1.4.7. Z określenia cze ↩<br />

ści ca̷lkowitej wynika, że<br />

5x + 4<br />

7<br />

≤ 2x + 3<br />

5<br />

< 5x + 4<br />

7<br />

Rozwia ↩<br />

zuja ↩<br />

c powyższa ↩<br />

nierówność mamy<br />

+ 1.<br />

− 34<br />

11 < x ≤ 1 11 ,


58 Cze ↩<br />

ść II – Rozwia ↩<br />

zania<br />

sta ↩<br />

d<br />

− 126<br />

77 < 5x + 4<br />

7<br />

≤ 49<br />

77 .<br />

Ponieważ 5x+4<br />

7<br />

jest liczba ↩<br />

ca̷lkowita ↩<br />

, może przyjmować wartości −1<br />

lub 0 (jedyne liczby ca̷lkowite z przedzia̷lu ( − 126<br />

77 , 49<br />

77]<br />

). Z równań<br />

5x + 4<br />

7<br />

= −1 lub<br />

wynika, że x = − 11 5<br />

lub x = − 4 5 .<br />

5x + 4<br />

7<br />

= 0<br />

1.4.8. Zapisuja ↩<br />

c x w postaci x = [x] + {x} , mamy<br />

[<br />

[[x] + {x}] + [x] + {x} + 1 ] [<br />

+ [x] + {x} + 2 ]<br />

+<br />

n<br />

n<br />

[<br />

+ · · · + [x] + {x} + n − 1 ]<br />

.<br />

n<br />

Korzystaja ↩<br />

c z zadania 1.4.3(c) przekszta̷lcamy (1) do postaci<br />

[<br />

[x] + [{x}] + [x] + {x} + 1 ] [<br />

+ [x] + {x} + 2 ]<br />

+<br />

n<br />

n<br />

[<br />

+ · · · + [x] + {x} + n − 1 ]<br />

.<br />

n<br />

(1)<br />

(2)<br />

Upraszczaja ↩<br />

c zapis w (2) otrzymujemy<br />

n−1<br />

∑<br />

[<br />

n[x] + {x} + j ]<br />

. (3)<br />

n<br />

j=0<br />

Ale dla dowolnej liczby rzeczywistej x można tak dobrać i należa ↩<br />

ce<br />

do zbioru {0,1,. . . n − 1} , że<br />

i − 1<br />

n < {x} < i n . (4)<br />

Drugi sk̷ladnik sumy (3) możemy wie ↩<br />

c zapisać w postaci<br />

∑n−i<br />

[<br />

{x} + j ]<br />

+<br />

n<br />

j=0<br />

n−1<br />

∑<br />

j=n−i+1<br />

[<br />

{x} + j ]<br />

. (5)<br />

n


Podstawowe w̷lasności liczb ca̷lkowitych 59<br />

Dla j ∈ {0,1,. . . n − i} mamy<br />

0 ≤ {x} + j n < i n + n − i<br />

n<br />

= 1. (6)<br />

Natomiast dla j ∈ {n − i + 1,n − i + 2,. . . n − 1} mamy<br />

oraz<br />

{x} + j n ≥ i − 1<br />

n + n − i + 1<br />

n<br />

{x} + j n < i n + n − 1<br />

n<br />

= n + i − 1<br />

n<br />

= 1 (7)<br />

≤ 2. (8)<br />

Z (6) wynika, że pierwsza suma w (5) jest równa 0, a z (7) oraz (8)<br />

dostajemy, że druga suma w (5) jest równa (n − 1) − (n − i) , czyli<br />

i − 1 .<br />

Sta ↩<br />

d mamy, że wyrażenie (1) jest równe n[x] + i − 1 . Z drugiej<br />

strony,<br />

[nx] = [n([x] + {x})] = n[x] + [n {x}].<br />

Wobec (4) otrzymujemy<br />

i − 1 ≤ n {x} < i,<br />

co oznacza, że [n {x}] = i − 1 . Zatem [nx] = n[x] + i − 1 .<br />

1.4.9. Ponieważ<br />

[ n<br />

] [ ] [ ]<br />

n + 1 n + k − 1<br />

+ + · · · +<br />

k k<br />

k<br />

[ n<br />

] [ n<br />

= +<br />

k k + 1 ] [ n<br />

+ · · · +<br />

k k + k − 1 ]<br />

,<br />

k<br />

wie ↩<br />

c wystarczy skorzystać z 1.4.8.<br />

1.5. Dzielenie z reszta ↩<br />

– dalsze w̷lasności<br />

1.5.1.<br />

(a) 83 = (−3) · (−27) + 2 ;<br />

(b) −71 = (−4) · 18 + 1 .


60 Cze ↩<br />

ść II – Rozwia ↩<br />

zania<br />

1.5.2. K̷lada ↩<br />

c<br />

[ m<br />

]<br />

q = + 1<br />

n<br />

oraz r = m − qn,<br />

mamy m n < q ≤ m n + 1 , ska ↩d wynika<br />

m > qn ≥ m + n,<br />

czyli −m < −qn ≤ −m − n . Zatem dodaja ↩<br />

c m do wszystkich<br />

sk̷ladników otrzymanej nierówności mamy<br />

Sta ↩<br />

d<br />

0 < m − qn ≤ −n.<br />

0 < r ≤ |n|.<br />

1.5.3. Niech n = kp + r , gdzie 0 ≤ r < p . Wtedy<br />

wielokrotnościami liczby p mniejszymi od n sa ↩<br />

liczby (i tylko one):<br />

[<br />

Jest ich dok̷ladnie k =<br />

n<br />

p<br />

p, 2p, . . . , kp.<br />

]<br />

.<br />

1.5.4. Mamy n = 2m + 1 = 3k + 2 = 4s + 3 = 5l + 4 .<br />

Ponieważ n = 5l + 4 = (4l + 4) + l , wie ↩<br />

c reszta z dzielenia liczby<br />

(4l+4)+l przez 4 jest równa reszcie z dzielenia l przez 4. Ponieważ<br />

(4l +4)+l = 4s+3 , wie ↩<br />

c reszta z dzielenia l przez 4 jest równa 3 .<br />

Zatem<br />

l ∈ {3,7,11,15,19 . . .} .<br />

Jeżeli l = 3 , to n = 19 nie spe̷lnia warunków zadania. Podobnie,<br />

gdy l = 7 , to n = 39 nie spe̷lnia naszych warunków. Natomiast<br />

dla l = 11 , liczba n = 59 spe̷lnia warunki zadania.


Podstawowe w̷lasności liczb ca̷lkowitych 61<br />

1.6. Najwie ↩<br />

kszy wspólny dzielnik<br />

1.6.1. Jeśli m < 0 , to NWD(m, n) = −m . Istotnie, (−m)|m<br />

oraz (−m)|n . Jeżeli c|m i c|n , to c|(−m) .<br />

Jeżeli m > 0 , to NWD(m, n) = m . Zatem NWD(m, n) = |m| .<br />

1.6.2. Niech d = NWD(m, n) . Wtedy d|m i d|n . Sta ↩<br />

d wobec<br />

równości m = nq + r , mamy d|r . Czyli d|n i d|r . Jeśli c|n oraz<br />

c|r , to z równości m = nq +r wynika, że c|m . Ponieważ c|n i c|m<br />

oraz d = NWD(m, n) , wie ↩<br />

c c|d . Zatem d = NWD(n, r) .<br />

1.6.3. Zauważmy, że<br />

98 = 1 · 56 + 42,<br />

56 = 1 · 42 + 14,<br />

42 = 3 · 14 + 0.<br />

Zatem NWD(98, 56) = NWD(56, 42) = NWD(42, 14) = NWD(14, 0) =<br />

14 .<br />

1.6.4. Przypuśćmy, że NWD(n, n + 1) = d . Wtedy d|n oraz<br />

d|n + 1 , czyli n = dm oraz n + 1 = dp dla pewnych liczb<br />

ca̷lkowitych m i p . Sta ↩<br />

d 1 = (n + 1) − n = dp − dm = d(p − m) ,<br />

co oznacza, że d|1 . Ponieważ d ≥ 1 , wie ↩<br />

c d = 1 .<br />

1.6.5. Jeżeli d = NWD(2k + 1, 2k + 3) , to 2k + 1 = da i<br />

2k + 3 = db dla pewnych liczb ca̷lkowitych a i b . Sta ↩<br />

d 2 =<br />

(2k + 3) − (2k + 1) = d(b − a) . Zatem d|2 , czyli d = 2 lub<br />

d = 1 . Jeżeli jednak d = 2 , to z warunku 2k + 1 = da wynika, że<br />

2(k − a) = −1 , czyli 2|(−1) , co nie jest możliwe. Zatem d = 1 .<br />

1.6.6. Przypuśćmy, że NWD(m 1 , n 1 ) = s > 1 . Wtedy m 1 =<br />

sm 2 oraz n 1 = sn 2 dla pewnych m 2 , n 2 ∈ Z . Sta ↩<br />

d m = dsm 2<br />

i n = dsn 2 . Ponieważ s > 1 , wie ↩<br />

c liczby m i n maja ↩<br />

wspólny<br />

dzielnik ds wie ↩<br />

kszy od d wbrew temu, że NWD(m, n) = d . Zatem<br />

s = 1 .<br />

1.6.7. Za̷lóżmy, że d = NWD(a, a + b) . Wtedy a = dx oraz<br />

a+b = dy dla pewnych liczb ca̷lkowitych x i y . Sta ↩<br />

d b = dy−dx =<br />

d(y − x) , czyli d|b . Wynika sta ↩<br />

d, że d jest wspólnym dzielnikiem<br />

liczb a i b . Ponieważ jednak NWD(a, b) = 1 , wie ↩<br />

c d = 1 , czyli<br />

u̷lamek<br />

a<br />

a+b<br />

jest nieskracalny.


62 Cze ↩<br />

ść II – Rozwia ↩<br />

zania<br />

1.6.8. Ponieważ NWD(m, n) = 1 , wie ↩<br />

c istnieja ↩<br />

liczby ca̷lkowite<br />

x oraz y takie, że mx + ny = 1 . Mnoża ↩<br />

c ostatnia ↩<br />

równość przez<br />

k otrzymujemy mkx + nky = k . Ponieważ m|nk , wie ↩<br />

c istnieje<br />

liczba ca̷lkowita a taka, że nk = ma . Sta ↩<br />

d mkx + may = k , czyli<br />

m(kx + ay) = k , co oznacza, że m|k .<br />

1.6.9. Najwie ↩<br />

kszym wspólnym dzielnikiem liczb m , n i p<br />

nazywamy taka ↩<br />

liczbe ↩<br />

d ≥ 1 , że<br />

(a) d|m , d|n oraz d|p ;<br />

(b) jeśli c|m , c|n i c|p , to c|d .<br />

Zauważmy, że najwie ↩<br />

kszy dzielnik w̷laściwy liczby 24 (tj. różny<br />

od 1 i od 24), czyli 12, jest również dzielnikiem 36 oraz 120.<br />

Ponieważ 24 nie jest dzielnikiem 36 , wie ↩<br />

c NWD(24,36,120) = 12 .<br />

1.6.10. Zauważmy najpierw, że przynajmniej jedna z liczb<br />

a 1 , a 2 , . . . , a k musi być różna od zera. Najwie ↩<br />

kszym wspólnym<br />

dzielnikiem liczb a 1 , a 2 , . . . , a k nazywamy taka ↩<br />

liczbe ↩<br />

d ≥ 1 , że<br />

(a) d|a j dla każdego j ∈ {1,2,. . . k} ;<br />

(b) jeśli c|a j dla każdego j ∈ {1,2,. . . k} , to c|d .<br />

Aby znaleźć NWD(36,120,180,600) , sprawdzamy, czy dzielniki<br />

liczby 36 (od najwie ↩<br />

kszego do najmniejszego) sa ↩<br />

też dzielnikami pozosta̷lych<br />

trzech liczb. Mamy 36̸ | 120 , wie ↩<br />

c 36 nie jest wspólnym<br />

dzielnikiem, podobnie jak 18, które także nie dzieli 120. Ponieważ<br />

12 dzieli 120, 180 oraz 600, jest to szukany najwie ↩<br />

kszy wspólny<br />

dzielnik.<br />

1.6.11. Oznaczmy d = NWD(m,n,p) (zdefiniowanemu w zadaniu<br />

1.6.9) i d ′ = PNWD(m,n,p) . Trzeba pokazać, że d =<br />

d ′ . Ponieważ d ′ |m oraz d ′ |NWD(n, p) , wie ↩<br />

c d ′ dzieli każda ↩<br />

z liczb m , n i p . Zatem d ′ |d . Z drugiej strony, skoro<br />

d|n i d|p , wie ↩<br />

c d|NWD(n, p) . Ponieważ zachodzi też d|m , wie ↩<br />

c<br />

d|NWD(m, NWD(n, p)) , co oznacza, że d|d ′ . Z definicji NWD wynika,<br />

że d ′ ≥ 1 . Sta ↩<br />

d i z faktów, że d|d ′ oraz d ′ |d mamy d = d ′ .<br />

1.7. Najmniejsza wspólna wielokrotność<br />

1.7.1.<br />

(a) Liczba NWW(2,3,4,5,6,7,8, 9,10) + 1 spe̷lnia warunki zadania.


Podstawowe w̷lasności liczb ca̷lkowitych 63<br />

(b) Liczba NWW(2,3,4,5,6,7,8, 9,10) − 1 spe̷lnia warunki zadania.<br />

Istotnie,<br />

NWW(2,3,4,5,6,7,8, 9,10) − 1 = 2k 1 − 1 = 2(k 1 − 1) + 1<br />

= 3k 2 − 1 = 3(k 2 − 1) + 2<br />

= 4k 3 − 1 = 4(k 3 − 1) + 3<br />

= . . . . . . . . . . . . . . . . . . . . .<br />

= 10k 9 − 1 = 10(k 9 − 1) + 9.<br />

1.7.2. Ponieważ a|a 1 db 1 oraz b|a 1 db 1 , wie ↩<br />

c NWW(a, b)|a 1 db 1 .<br />

Sta ↩<br />

d wynika, że<br />

a 1 db 1 = NWW(a, b)s (*)<br />

dla pewnej liczby ca̷lkowitej s . Wystarczy pokazać, że s = 1 .<br />

Przypuśćmy, nie wprost, że s > 1 . Ponieważ a|NWW(a, b)<br />

oraz b|NWW(a, b) , wie ↩<br />

c NWW(a, b) = ar = bt dla pewnych liczb<br />

ca̷lkowitych r i t . Podstawiaja ↩<br />

c do (*) otrzymujemy<br />

a 1 db 1 = a 1 drs = b 1 dts.<br />

Sta ↩<br />

d a 1 = ts oraz b 1 = rs , czyli a = dts i b = drs , co z kolei<br />

oznacza, że NWD(a, b) ≥ ds > d , ska ↩<br />

d sprzeczność.<br />

1.7.3. Niech NWD(a, b) = d i niech a = da 1 , b = db 1 . Wtedy<br />

NWW(a, b) = a 1 db 1 (zobacz zadanie 1.7.2). Sta ↩<br />

d<br />

ab = a 1 db 1 d = NWD(a, b)NWW(a, b).<br />

1.7.4. Jeżeli liczby a i b sa ↩<br />

wzgle ↩<br />

dnie pierwsze (czyli<br />

NWD(a, b) = 1 ), to NWD(a, b)NWW(a, b) = NWW(a, b) . Zatem z zadania<br />

1.7.3 wynika ża ↩<br />

dana równość.<br />

1.7.5. Niech d = NWD(a, b) . Wtedy d ≤ a oraz d ≤ b . Sta ↩<br />

d<br />

(d − a)(d − b) ≥ 0 , czyli d 2 − db − ad + ab ≥ 0 . Sta ↩<br />

d i z zadania<br />

1.7.3,<br />

d 2 − d(a + b) + dNWW(a, b) ≥ 0<br />

i ostatecznie<br />

d + NWW(a, b) ≥ a + b.


64 Cze ↩<br />

ść II – Rozwia ↩<br />

zania<br />

1.7.6.<br />

(a) Niech w be ↩<br />

dzie wspólna ↩<br />

wielokrotnościa ↩<br />

liczb m oraz n .<br />

Dziela ↩<br />

c w przez NWW(m, n) , mamy w = qNWW(m, n)+r ,<br />

gdzie q ∈ Z oraz 0 ≤ r < NWW(m, n) . Zauważmy, że m|r<br />

i n|r . Jeżeli r > 0 , to r jest wspólna ↩<br />

wielokrotnościa ↩<br />

liczb m oraz n mniejsza ↩<br />

od NWW(m, n) . Z otrzymanej<br />

sprzeczności wynika, że r = 0 , czyli, że NWW(m, n)|w .<br />

(b) Niech d 1 be ↩<br />

dzie wspólnym dzielnikiem liczb m oraz n<br />

i niech d be ↩<br />

dzie najwie ↩<br />

kszym wspólnym dzielnikiem liczb<br />

m i n . Liczba m jest wspólna ↩<br />

wielokrotnościa ↩<br />

liczb<br />

d oraz d 1 , wie ↩<br />

c NWW(d, d 1 )|m . Liczba n jest również<br />

wspólna ↩<br />

wielokrotnościa ↩<br />

liczb d i d 1 , wie ↩<br />

c NWW(d, d 1 )|n .<br />

Oznacza to, iż NWW(d, d 1 ) jest wspólnym dzielnikiem<br />

liczb m oraz n , wie ↩<br />

c NWW(d, d 1 ) ≤ d . Z drugiej<br />

strony, NWW(d, d 1 ) ≥ d (co wynika z definicji najmniejszej<br />

wspólnej wielokrotności). Zatem NWW(d, d 1 ) = d . Sta ↩<br />

d<br />

ostatecznie mamy d 1 |d .<br />

1.8. Zasadnicze twierdzenie arytmetyki<br />

1.8.1. Przypuśćmy, że NWD(c, b) = d . Wtedy d|c oraz d|b .<br />

Ponieważ c|a , wie ↩<br />

c d|a oraz d|b . Ale NWD(a, b) = 1 , wie ↩<br />

c d = 1 .<br />

1.8.2. Przypuśćmy, że NWD(ab, c) = d . Wtedy d|ab oraz<br />

d|c . Ponieważ d|c oraz NWD(a, c) = 1 , wie ↩<br />

c w myśl 1.8.1 mamy<br />

NWD(a, d) = 1 . Sta ↩<br />

d, wobec tego że d|ab oraz zasadniczego twierdzenia<br />

arytmetyki, mamy d|b . Zatem d|b oraz d|c . Ponieważ<br />

NWD(b, c) = 1 , wie ↩<br />

c d = 1 .<br />

1.8.3. Zastosuj indukcje ↩<br />

wzgle ↩<br />

dem n .<br />

1.8.4. Liczby wymierne dodatnie sa ↩<br />

to ilorazy dwóch liczb<br />

naturalnych a b . Niech NWD(a, b) = d . Wówczas a = a 1d , b = b 1 d ,<br />

gdzie NWD(a 1 , b 1 ) = 1 . Sta ↩<br />

d<br />

a<br />

b = a 1d<br />

b 1 d = a 1<br />

b 1<br />

.<br />

Aby pokazać jednoznaczność przedstawienia przypuśćmy, że m n =<br />

, gdzie NWD(m, n) = 1 oraz NWD(r, s) = 1 . Wtedy ms = nr ,<br />

r<br />

s


Podstawowe w̷lasności liczb ca̷lkowitych 65<br />

sta ↩<br />

d r|ms . Z zasadniczego twierdzenia arytmetyki wynika, że r|m .<br />

Istnieje zatem liczba naturalna t taka, że m = rt . Podstawiaja ↩<br />

c<br />

mamy rts = nr , czyli ts = n . Z powyższych rozważań wynika,<br />

że t jest wspólnym dzielnikiem m i n . Ponieważ NWD(m, n) = 1 ,<br />

wie ↩<br />

c t = 1 . Sta ↩<br />

d ostatecznie mamy r = m oraz s = n .


2.1. Poje ↩<br />

cie liczby pierwszej<br />

2. Liczby pierwsze<br />

2.1.1. Sa ↩<br />

to 2, 3, 5, 7, 11, 13, 17 i 19.<br />

2.1.2. Najmniejszymi liczbami pierwszymi wie ↩<br />

kszymi od podanych<br />

liczb sa ↩<br />

(a) 29, (b) 59, (c) 101.<br />

2.1.3. Niech p be ↩<br />

dzie liczba ↩<br />

pierwsza ↩<br />

. Za̷lóżmy najpierw, że<br />

p < 30 . Wówczas p = 0 · 30 + p . Jeśli za̷lożymy, że p > 30 , to<br />

otrzymamy p = q30 + r , gdzie r ∈ {0,1,2,. . . ,29} . Ponieważ p<br />

jest liczba ↩<br />

pierwsza ↩<br />

, wie ↩<br />

c NWD(30, r) = 1 . Bezpośrednio ̷latwo jest<br />

sprawdzić, że r może być równe jednej z naste ↩<br />

puja ↩<br />

cych liczb: 1, 7,<br />

11, 13, 17, 19, 23, 29.<br />

2.1.4. Niech p > 3 be ↩<br />

dzie liczba ↩<br />

pierwsza ↩<br />

. Należy pokazać, że<br />

p 2 = 12k + 1 , czyli że 12|p 2 − 1 . Ponieważ 3|(p − 1)p(p + 1) (jako<br />

iloczyn kolejnych trzech liczb naturalnych) oraz 3̸ | p , wie ↩<br />

c 3|(p −<br />

1)(p+1) . Liczby p−1 oraz p+1 sa ↩<br />

parzyste, wie ↩<br />

c 4|(p−1)(p+1) .<br />

Zatem 12|(p − 1)(p + 1) .<br />

2.1.5. Jeżeli liczba a n = 2 n + 1 , gdzie n > 2 jest liczba ↩<br />

pierwsza ↩<br />

, to nie może być postaci 3k i nie może być też postaci<br />

3k +1 (bo wtedy 2 n by̷loby postaci 3k ). Zatem, jeżeli a n = 2 n +1<br />

jest liczba ↩<br />

pierwsza ↩<br />

, to jest postaci 3k + 2 . Wtedy b n = 2 n − 1 =<br />

(2 n + 1) − 2 = 3k + 2 − 2 = 3k , co oznacza, że b n nie jest liczba ↩<br />

pierwsza ↩<br />

. Analogicznie uzasadniamy, że jeśli b n jest liczba ↩<br />

pierwsza ↩<br />

,<br />

to a n nie jest liczba ↩<br />

pierwsza ↩<br />

.<br />

2.1.6. Przypuśćmy, że 6k + 1 = p 1 − p 2 , gdzie p 1 i p 2 sa ↩<br />

liczbami pierwszymi. Jeżeli p 2 = 2 , to p 1 = 6k + 3 = 3(2k + 1) ,<br />

czyli p 1 nie jest liczba ↩<br />

pierwsza ↩<br />

. Jeżeli p 2 = 2s + 1 , to p 1 =<br />

6k + 1 + 2s + 1 = 2((3k + s) + 1) , co oznacza, że p 1 nie jest liczba ↩<br />

pierwsza ↩<br />

.<br />

2.1.7. Liczba pierwsza p może być postaci 3k + 1 lub 3k +<br />

2 , lub p = 3 . Jeżeli p = 3k + 1 , to liczba 8(3k + 1) 2 + 1 =<br />

3(24k 2 + 16k + 3) jest liczba ↩<br />

z̷lożona ↩<br />

. Jeżeli p = 3k + 2 , to liczba<br />

8(3k + 2) 2 + 1 = 3(24k 2 + 32k + 11) jest także liczba ↩<br />

z̷lożona ↩<br />

. Jeśli<br />

p = 3 , to 8p 2 +1 = 73 jest liczba ↩<br />

pierwsza ↩<br />

. Wtedy 8p 2 +2p+1 = 79<br />

też jest liczba ↩<br />

pierwsza ↩<br />

.


Liczby pierwsze 67<br />

Uwaga. W rozwia ↩<br />

zaniu tego zadania można też wykorzystać<br />

zadanie 2.1.4. Istotnie, z zadania 2.1.4 wynika, że jeżeli p > 3 ,<br />

to p 2 ≡ 1(mod 12 ) . Mnoża ↩<br />

c obie strony tej kongruencji przez 8,<br />

a naste ↩<br />

pnie dodaja ↩<br />

c 1, mamy 8p 2 + 1 ≡ 9(mod 12 ) . Sta ↩<br />

d wynika,<br />

że 3|8p 2 + 1 , co oznacza, że liczba 8p 2 + 1 nie jest liczba ↩<br />

pierwsza ↩<br />

,<br />

gdy p > 3 . Zatem liczba 8p 2 + 1 jest liczba ↩<br />

pierwsza ↩<br />

wtedy i tylko<br />

wtedy, gdy p = 3 . Wówczas liczba 8p 2 +2p+1 = 79 jest też liczba ↩<br />

pierwsza ↩<br />

.<br />

2.1.8. Niech n be ↩<br />

dzie liczba ↩<br />

naturalna ↩<br />

. Jeśli jest to liczba<br />

pierwsza, nasz problem jest rozwia ↩<br />

zany, ponieważ n|n . Za̷lóżmy, że<br />

liczba n jest z̷lożona, czyli że istnieja ↩<br />

liczby k , takie że 1 < k < n ,<br />

które dziela ↩<br />

n . Niech q be ↩<br />

dzie najmniejsza ↩<br />

z nich. Pokażemy, że<br />

q jest liczba ↩<br />

pierwsza ↩<br />

. Istotnie, jeśli 1 < r < q oraz r|q , to także<br />

r|n . Zatem r jest mniejszym od q dzielnikiem w̷laściwym liczby<br />

n . Przeczy to wyborowi q . Otrzymana sprzeczność dowodzi, że q<br />

jest liczba ↩<br />

pierwsza ↩<br />

.<br />

2.2. Ile jest liczb pierwszych<br />

2.2.1. Jeśli p j |p 1 p 2 . . . p k + 1 dla pewnego 1 ≤ j ≤ k , to<br />

ponieważ p j |p 1 p 2 . . . p k , wie ↩<br />

c mamy p j |1 . Jest to niemożliwe.<br />

2.2.2. Za̷lóżmy, że q|n! + 1 oraz q ≤ n . Wówczas, ponieważ<br />

q|n! , musia̷loby być q|1 , wie ↩<br />

c q = 1 .<br />

2.2.3. Przypuśćmy, że istnieje liczba naturalna n taka, że<br />

dla każdej liczby pierwszej p zachodzi nierówność p ≤ n . Mamy<br />

p̸ | n!+1 (patrz poprzednie zadanie). Z drugiej strony, istnieje liczba<br />

pierwsza q , która dzieli n! + 1 (z zadania 2.1.8). Zatem liczba q<br />

musi być wie ↩<br />

ksza od n .<br />

2.2.4. Za̷lóżmy, że m = n + k oraz NWD(F n , F m ) = d > 1 dla<br />

n ≠ m . Istnieje zatem liczba pierwsza p , która dzieli d . Zatem<br />

p|F n oraz p|F m . Oznacza to, że istnieje liczba t taka, że pt − 1 =<br />

2 2n . Sta ↩<br />

d (pt − 1) 2k = 2 2n+k = 2 2m . Stosuja ↩<br />

c wzór dwumianowy<br />

Newtona zauważamy, że (pt − 1) 2k = pu + 1 dla pewnego u .<br />

Sta ↩<br />

d p|2 2m − 1 , czyli p|F m − 2 . Z drugiej strony mamy, że p|F m ,<br />

wie ↩<br />

c p|2 . Ponieważ p nie może być równe 2 (bo F n jest liczba ↩<br />

nieparzysta ↩<br />

), wie ↩<br />

c jeśli p|d , to p = 1 . Otrzymaliśmy sprzeczność,<br />

wie ↩<br />

c również d jest równe 1, czyli F n i F m sa ↩<br />

wzgle ↩<br />

dnie pierwsze.


68 Cze ↩<br />

ść II – Rozwia ↩<br />

zania<br />

2.2.5.<br />

(a) Przypuśćmy, że liczb pierwszych jest skończona ilość.<br />

Możemy je wie ↩<br />

c wypisać. Przypuśćmy, że sa ↩<br />

to p 1 ,<br />

p 2 , . . . , p k . Ale wówczas liczba p 1 p 2 . . . p k +1 nie dzieli<br />

sie ↩<br />

przez żadna ↩<br />

z liczb pierwszych (zadanie 2.2.1), choć<br />

musi (zadanie 2.1.8). Sta ↩<br />

d sprzeczność.<br />

(b) Wybierzemy indukcyjnie cia ↩<br />

g nieskończony liczb pierwszych.<br />

Niech p 1 = 2 (jest to, oczywiście, liczba pierwsza).<br />

Z zadania 2.2.3 wynika, że istnieje liczba pierwsza<br />

p 2 > p 1 . Przypuśćmy, że wybraliśmy już liczby pierwsze<br />

p 1 , p 2 , . . . , p k . Z zadania 2.2.3 wynika, że istnieje liczba<br />

pierwsza p k+1 wie ↩<br />

ksza od p k . W̷la ↩<br />

czamy ja ↩<br />

do naszego<br />

cia ↩<br />

gu. Na podstawie indukcji matematycznej możemy<br />

wie ↩<br />

c wybrać nieskończenie wiele różnych liczb pierwszych.<br />

Zatem tych liczb jest nieskończenie wiele.<br />

(c) Mamy cia ↩<br />

g nieskończony liczb Fermata (F n ) . Wyrazy<br />

tego cia ↩<br />

gu sa ↩<br />

parami wzgle ↩<br />

dnie pierwsze (zadanie 2.2.4),<br />

wie ↩<br />

c każdemu z nich możemy przyporza ↩<br />

dkować jego najmniejszy<br />

dzielnik pierwszy p n . ̷Latwo zauważyć, że przyporza<br />

↩<br />

dkowanie F n → p n ze zbioru liczb Fermata do<br />

zbioru liczb pierwszych jest różnowartościowe. Zatem<br />

liczb pierwszych jest przynajmniej tyle, ile liczb Fermata,<br />

czyli nieskończenie wiele.<br />

2.3. Wnioski z zasadniczego twierdzenia arytmetyki<br />

2.3.1. W przedstawieniu liczby naturalnej n w postaci<br />

iloczynu liczb pierwszych (zobacz twierdzenie na pocza ↩<br />

tku 2.3)<br />

moga ↩<br />

wyste ↩<br />

pować równe liczby pierwsze. Oznaczmy przez p 1 ,<br />

p 2 , . . . , p s różne czynniki pierwsze, a przez α 1 , α 2 , . . . , α s<br />

cze ↩<br />

stości wyste ↩<br />

powania ich w rozk̷ladzie liczby n . Wówczas<br />

n = p α 1<br />

1 pα 2<br />

2 . . . pα s<br />

s .<br />

2.3.2. Jeżeli a = p α 1<br />

1 pα 2<br />

2 . . . pα k<br />

k<br />

oraz b = p β 1<br />

1 pβ 2<br />

2 . . . pβ k<br />

k<br />

, gdzie<br />

α ≥ 0 i β ≥ 0 , to NWD(a, b) = p min(α 1,β 1 )<br />

1 p min(α 2,β 2 )<br />

2 . . . p min(α k,β k )<br />

k<br />

,<br />

NWW(a, b) = p max(α 1,β 1 )<br />

1 p max(α 2,β 2 )<br />

2 . . . p max(α k,β k )<br />

k<br />

.<br />

2.3.3. Zauważmy, że min(α i ,β i ) + max(α i ,β i ) = α i + β i .<br />

Korzystaja ↩<br />

c z poprzedniego zadania otrzymujemy teze ↩<br />

.


Liczby pierwsze 69<br />

2.3.4. Niech √ n = p q<br />

, gdzie p,q ∈ N oraz NWD(p, q) = 1 .<br />

Wtedy nq 2 = p 2 . Ponieważ NWD(p, q) = 1 , wie ↩<br />

c NWD(<br />

p 2 , q 2) = 1<br />

(zobacz 1.8.2). Sta ↩<br />

d wobec równości nq 2 = p 2 mamy p 2 |n , czyli<br />

n = p 2 m dla pewnego m ∈ N . Po podstawieniu mamy p 2 mq 2 =<br />

p 2 , sta ↩<br />

d mq 2 = 1 , czyli m = 1 oraz q = 1 . Zatem n = p 2 . Jeżeli<br />

p = p α 1<br />

1 pα 2<br />

2 . . . pα s<br />

s , to n = p 2 = p 2α 1<br />

1 p 2α 2<br />

2 . . . p 2α s<br />

s .<br />

2.3.5.<br />

(a) Z treści zadania wynika, że m = p α m 1 oraz n = p β n 1 ,<br />

przy czym p̸ | m 1 oraz p̸ | n 1 . Sta ↩<br />

d mn = p α+β m 1 n 1 ,<br />

przy czym p̸ | m 1 n 1 . Zatem p α+β ||mn .<br />

(b) Bez zmniejszenia ogólności możemy za̷lożyć, że α < β ,<br />

wie ↩<br />

c α = min(α,β) . Ponieważ m = p α m 1 , n = p β n 1 ,<br />

przy czym p̸ | m 1 oraz p̸ | n 1 , wie ↩<br />

c m + n = p α (m 1 +<br />

p β−α n 1 ) . Zauważmy, że p̸ | (m 1 + p β−α n 1 ) . Istotnie,<br />

gdyby p|(m 1 + p β−α n 1 ) , to wówczas p dzieli̷loby m 1 ,<br />

ponieważ p|p β−α . Zatem p α ||m + n .<br />

(c) Aby sie ↩<br />

przekonać, że za̷lożenia tego nie można pomina ↩<br />

ć,<br />

wystarczy wzia ↩<br />

ć m = n = p = 2 .<br />

2.3.6. Każda ↩<br />

liczbe ↩<br />

wymierna ↩<br />

możemy zapisać jako iloraz<br />

liczby ca̷lkowitej w przez naturalna ↩<br />

v , przy czym NWD(w, v) = 1 .<br />

Zapiszmy v = r γ 1<br />

1 rγ 2<br />

2 . . . rγ t<br />

t , |w| = q β 1<br />

1 qβ 2<br />

2 . . . qβ u<br />

u oraz ε = w<br />

|w| .<br />

Wówczas w v = εqβ 1<br />

1 qβ 2<br />

2 . . . qβ u<br />

u r −γ 1<br />

1 r −γ 2<br />

2 . . . r −γ t<br />

t .<br />

2.3.7. Na pocza ↩<br />

tku zauważmy, że suma podana w treści zadania<br />

jest w istocie skończona, [ ponieważ od pewnego momentu liczby<br />

p i n<br />

sa ↩<br />

wie ↩<br />

ksze od n , wie ↩<br />

c = 0 . Rozważmy teraz cia ↩<br />

g 1 ,<br />

p i ]<br />

2 , . . . , n . Liczby tego cia ↩<br />

gu podzielne przez p i (dla ustalonego i )<br />

sa ↩<br />

postaci lp i , gdzie l jest pewna ↩<br />

liczba ↩<br />

ca̷lkowita [ ↩<br />

nieujemna ↩<br />

, taka ↩<br />

że lp i n<br />

≤ n . Zatem liczb tych jest dok̷ladnie . Najwyższy<br />

wyk̷ladnik α pote ↩<br />

gi liczby p , która dzieli n! otrzymamy biora ↩<br />

c<br />

liczbe [ ] ↩<br />

wyrazów cia ↩<br />

gu 1 , 2 , . . . , n podzielnych przez p (czyli<br />

n<br />

p<br />

), dodaja ↩<br />

c do niej liczbe ↩<br />

wyrazów podzielnych przez p 2 (czyli<br />

[ ]<br />

[ ]<br />

n<br />

p<br />

), naste 2 ↩<br />

pnie liczbe ↩<br />

wyrazów podzielnych przez p 3 n<br />

(czyli<br />

p<br />

) 3<br />

itd. Sta ↩<br />

d α = ∑ [<br />

∞ n<br />

i=1<br />

.<br />

p i ]<br />

p i ]


70 Cze ↩<br />

ść II – Rozwia ↩<br />

zania<br />

2.3.8. Zgodnie z zadaniem 2.3.7, wyk̷ladniki tych pote ↩<br />

g sa ↩<br />

równe<br />

(a) α = ∑ ∞<br />

[ 100<br />

]<br />

i=1 2 = 50 + 25 + 12 + 6 + 3 + 1 = 97 .<br />

i<br />

(b) α = ∑ ∞<br />

[ 100<br />

]<br />

i=1 5 = 20 + 4 = 24 .<br />

i<br />

(c) α = ∑ ∞<br />

[ 100<br />

]<br />

i=1 97 = 1 .<br />

i<br />

2.3.9.<br />

(a) Najwie ↩<br />

ksza pote ↩<br />

ga liczby 6 , która dzieli 100! jest iloczynem<br />

najwie ↩<br />

kszej pote ↩<br />

gi liczby 3, która dzieli 100! oraz<br />

takiej samej pote ↩<br />

gi liczby 2. Zatem jest to 6 48 .<br />

(b) Skoro najwie ↩<br />

ksza ↩<br />

pote ↩<br />

ga ↩<br />

dwójki dziela ↩<br />

ca ↩<br />

100! jest 97,<br />

wie ↩<br />

c najwie ↩<br />

ksza ↩<br />

pote ↩<br />

ga ↩<br />

czwórki dziela ↩<br />

ca ↩<br />

100! jest 43.<br />

Natomiast najwie ↩<br />

ksza ↩<br />

pote ↩<br />

ga ↩<br />

siódemki dziela ↩<br />

ca ↩<br />

100! jest<br />

16. Zatem szukana ↩<br />

pote ↩<br />

ga ↩<br />

liczby 28 jest 28 16 .<br />

2.3.10. Ilość zer danej liczby n jest równa wyk̷ladnikowi<br />

najwie ↩<br />

kszej pote ↩<br />

gi liczby 10, która dzieli n , czyli wyk̷ladnikowi<br />

najwie ↩<br />

kszej pote ↩<br />

gi liczby 5, która dzieli n . W przypadku n = 100!<br />

wyk̷ladnik ten jest równy 24.<br />

Ponieważ<br />

2.4. Uwagi o funkcji π(x)<br />

2.4.1. Z nierówności a < π(x) :<br />

x<br />

ln x<br />

lim<br />

x→∞<br />

a<br />

ln x = lim<br />

x→∞<br />

a<br />

ln x < π(x)<br />

x < b<br />

ln x .<br />

b<br />

ln x = 0 , wie ↩c<br />

< b wynika, że<br />

π(x)<br />

lim<br />

x→∞ x = 0 .<br />

2.4.2. Liczby 10 n , 2 · 10 n , 4 · 10 n oraz 8 · 10 n ( n ≥ 0 )<br />

maja ↩<br />

po tyle samo cyfr. W myśl twierdzenia Czebyszewa, możemy<br />

,,wetkna ↩<br />

ć” pomie ↩<br />

dzy nie co najmniej po jednej liczbie pierwszej.<br />

Wszystkie ,,wetknie ↩<br />

te” liczby pierwsze be ↩<br />

da ↩<br />

mia̷ly po tyle samo cyfr.<br />

2.4.3. Zastosujemy tu indukcje ↩<br />

ze wzgle ↩<br />

du na k . Dla k = 3<br />

mamy p 3 = 5 = 2 + 3 = p 1 + p 2 . Za̷lóżmy teraz, że teza jest<br />

spe̷lniona dla wszystkich k ≤ m . Z twierdzenia Czebyszewa wynika,<br />

że istnieje przynajmniej jedna liczba pierwsza pomie ↩<br />

dzy p m+1 i<br />

2p m+1 . Najmniejsza z tych liczb to oczywiście p m+2 . Z za̷lożenia<br />

indukcyjnego mamy, że p m+2 < 2p m+1 = p m+1 + p m+1 ≤ p 1 + p 2 +<br />

· · · + p m + p m+1 . Zatem na podstawie indukcji teza jest prawdziwa.


Liczby pierwsze 71<br />

2.4.4. Zauważmy najpierw, że faktycznie 5 = p 3 = p 2 + p 1<br />

i, oczywiście, ani p 2 , ani p 1 nie sa ↩<br />

sumami wszystkich liczb<br />

pierwszych mniejszych od nich. Teraz zauważmy, że z twierdzenia<br />

Czebyszewa oraz poprzedniego zadania wynika, iż dla k > 3 mamy<br />

p k < 2p k−1 ≤ p k−1 + p k−2 + . . . + p 1 , a skoro zachodzi ostra<br />

nierówność, to nie może zachodzić równość.<br />

2.4.5. Najpierw zobaczmy, że 1, 2, 3 oraz 4 nie sa ↩<br />

suma ↩<br />

wszystkich liczb pierwszych mniejszych od nich, a 5 jest. Za̷lóżmy<br />

teraz, że n > 5 . Niech p k ( k-ta z kolei liczba pierwsza be ↩<br />

dzie<br />

najwie ↩<br />

ksza ↩<br />

liczba ↩<br />

pierwsza ↩<br />

mniejsza ↩<br />

od n . Zatem n ≤ p k+1 <<br />

2p k ≤ p 1 + p 2 + · · · + p k , na podstawie twierdzenia Czebyszewa<br />

i zadania 2.4.3. Tak wie ↩<br />

c n nie może być suma ↩<br />

k pierwszych liczb<br />

pierwszych, czyli wszystkich liczb pierwszych mniejszych od n .<br />

2.4.6. Ponieważ jeśli k ≤ m+1 , to k|(m+1)!+k , wie ↩<br />

c każda<br />

z liczb w cia ↩<br />

gu ((m + 1)! + k) 2≤k≤m+1<br />

jest z̷lożona. Elementów<br />

cia ↩<br />

gu jest dok̷ladnie m . By rozwia ↩<br />

zać zadanie wystarczy wie ↩<br />

c za n<br />

wybrać taka ↩<br />

liczbe ↩<br />

, że p n jest najwie ↩<br />

ksza ↩<br />

liczba ↩<br />

pierwsza ↩<br />

mniejsza ↩<br />

od (m+1)!+2 . Wtedy musi być p n+1 > (m+1)!+m+1 ≥ p n +m .<br />

2.4.7.<br />

(a) Z nierówności π(p) < p mamy −p < −π(p) .<br />

pπ(p) − p < pπ(p) − π(p) = (p − 1)π(p) , czyli<br />

π(p)<br />

p<br />

Sta ↩<br />

d<br />

π(p)−1<br />

p−1<br />

<<br />

. Ponieważ π(p) − 1 = π(p − 1) , wie ↩<br />

c π(p−1)<br />

p−1<br />

(b) Jeżeli m jest liczba ↩<br />

z̷lożona ↩<br />

, to π(m−1) = π(m) . Zatem<br />

π(m)<br />

m<br />

= π(m−1)<br />

m<br />

< π(m−1)<br />

m−1<br />

.<br />

2.5. Twierdzenie Dirichleta<br />

2.5.1. (a) m = 2 , n = 2 ; (b) m = 4 , n = 4 .<br />

< π(p)<br />

p<br />

.<br />

2.5.2. Ponieważ tylko 2 jest liczba ↩<br />

pierwsza ↩<br />

parzysta ↩<br />

, wie ↩<br />

c pozosta̷le<br />

liczby pierwsze (których jest nieskończenie wiele) musza ↩<br />

być<br />

liczbami nieparzystymi.<br />

2.5.3.<br />

(a) Podzielmy wszystkie liczby naturalne na trzy klasy. Do<br />

pierwszej zaliczmy liczby postaci 3k , do drugiej 3k + 1<br />

i do trzeciej 3k + 2 . Wystarczy pokazać, że w rozk̷ladzie


72 Cze ↩<br />

ść II – Rozwia ↩<br />

zania<br />

każdej liczby trzeciej klasy wyste ↩<br />

puje przynajmniej jedna<br />

liczba trzeciej klasy. Ponieważ 3̸ | 3k + 2 , wie ↩<br />

c liczby<br />

pierwszej klasy nie moga ↩<br />

wyste ↩<br />

pować w rozk̷ladach liczb<br />

trzeciej klasy. Zauważmy teraz, że iloczyn dwóch liczb<br />

drugiej klasy jest liczba ↩<br />

drugiej klasy. Istotnie, (3k 1 +<br />

1)(3k 2 +1) = 3(3k 1 k 2 +k 1 +k 2 )+1 . Zatem, gdyby rozk̷lad<br />

pewnej liczby sk̷lada̷l sie ↩<br />

tylko z liczb drugiej klasy, to ta<br />

liczba też musia̷laby być liczba ↩<br />

drugiej klasy. Zatem w rozk̷ladzie<br />

liczby postaci 3k + 2 przynajmniej jedna liczba<br />

jest postaci 3k + 2 .<br />

(b) Z cze ↩<br />

ści (a) wynika, że liczba n ma dzielniki postaci<br />

3k + 2 . Niech p oznacza najmniejszy z tych dzielników.<br />

Przypuśćmy, że p nie jest liczba ↩<br />

pierwsza ↩<br />

. Istnieja ↩<br />

wtedy<br />

liczby naturalne r , s takie, że p = r·s , gdzie 1 < r < p ,<br />

1 < s < p . Liczba p jest postaci 3k + 2 , wie ↩<br />

c z cze ↩<br />

ści (a)<br />

wynika, że r jest postaci 3k+2 lub s jest postaci 3k+2 .<br />

Niech, na przyk̷lad, r be ↩<br />

dzie postaci 3k + 2 . Wówczas r<br />

jest dzielnikiem postaci 3k +2 liczby n mniejszym od p ,<br />

co jest sprzeczne z wyborem liczby p .<br />

2.5.4. Oznaczmy P = p 1 p 2 . . . p n . Ponieważ liczba p m , gdzie<br />

1 ≤ m ≤ n , dzieli P, a nie dzieli 2, wie ↩<br />

c p m̸ | 3P +2 . Podobnie, 2|2 ,<br />

ale 2̸ | 3P , ponieważ 3 oraz wszystkie p m sa ↩<br />

nieparzyste. Zatem<br />

2̸ | 3P + 2 .<br />

2.5.5. Przypuśćmy, że tak nie jest i że p 0 = 2 , p 1 , p 2 . . . , p n<br />

sa ↩<br />

wszystkimi liczbami pierwszymi postaci 3k+2 . Rozważmy liczbe ↩<br />

L = 3p 1 p 2 . . . p n + 2 . Ponieważ dla 0 ≤ m ≤ n mamy p m < L<br />

oraz L jest postaci 3k + 2 , wie ↩<br />

c L musi być liczba ↩<br />

z̷lożona ↩<br />

. Jednakże<br />

w jej rozk̷ladzie musi znajdować sie ↩<br />

liczba pierwsza p postaci<br />

3k + 2 (zadanie 2.5.3). Z drugiej strony p ≠ p m dla 0 ≤ m ≤ n ,<br />

ponieważ p|L (zadanie 2.5.4). Zatem p jest liczba ↩<br />

pierwsza ↩<br />

postaci<br />

3k + 2 różna ↩<br />

od każdej z p m . Sta ↩<br />

d sprzeczność.<br />

2.5.6. Podobnie jak w zadaniu 2.5.3 pokazujemy, że w rozk̷ladzie<br />

liczby postaci 4k + 3 przynajmniej jedna liczba jest postaci<br />

4k + 3 . Naste ↩<br />

pnie, podobnie jak w zadaniu 2.5.4, uzasadniamy, że<br />

jeśli p 0 = 3 , p 1 , p 2 . . . , p n sa ↩<br />

liczbami pierwszymi postaci 4k+3 ,<br />

to liczba 4p 1 p 2 . . . p n + 3 nie dzieli sie ↩<br />

przez żadna ↩<br />

z liczb p 0 , p 1 ,<br />

p 2 , . . . , p n . Wreszcie rozwia ↩<br />

zujemy niniejsze zadanie rozumuja ↩<br />

c<br />

podobnie jak w zadaniu 2.5.5.


Liczby pierwsze 73<br />

2.5.7. Rozwia ↩<br />

zujemy podobnie jak poprzednie zadanie.<br />

2.6. Liczba dzielników oraz funkcja Eulera<br />

2.6.1.<br />

n 1 2 3 4 5 6 7 8 9 1011121314151617181920<br />

θ(n) 1 2 2 3 2 4 2 4 3 4 2 6 2 4 4 5 2 6 2 6<br />

2.6.2. Za̷lóżmy, że m|n . W rozk̷ladzie liczby m sa ↩<br />

tylko liczby<br />

pierwsze p , które sa ↩<br />

dzielnikami n . Ponieważ w rozk̷ladzie n<br />

liczba p i wyste ↩<br />

puje z wyk̷ladnikiem α i , wie ↩<br />

c w rozk̷ladzie liczby m<br />

musi ona wyste ↩<br />

pować z wyk̷ladnikiem 0 < β i ≤ α i lub nie<br />

wyste ↩<br />

pować. Zatem każdy dzielnik n jest ża ↩<br />

danej postaci. W druga<br />

↩<br />

strone ↩<br />

, ̷latwo sie ↩<br />

przekonać, że p β 1<br />

1 pβ 2<br />

2 . . . pβ s<br />

s dzieli n dla 0 ≤<br />

β i ≤ α i .<br />

2.6.3. Korzystaja ↩<br />

c z poprzedniego zadania wnioskujemy, że<br />

dzielnikami liczby n = p α 1<br />

1 pα 2<br />

2 . . . pα s<br />

s sa ↩<br />

te i tylko te liczby, które<br />

w swym rozk̷ladzie maja ↩<br />

tylko liczby pierwsze be ↩<br />

da ↩<br />

ce dzielnikami n<br />

w pote ↩<br />

gach nie wie ↩<br />

kszych od α i . Ponieważ mamy α i +1 możliwości<br />

wyboru takich pote ↩<br />

g, wie ↩<br />

c θ(n) = (α 1 + 1)(α 2 + 1) . . . (α s + 1) .<br />

2.6.4. θ(100) = θ(2 2 · 5 2 ) = (2 + 1)(2 + 1) = 9 ;<br />

θ(1024) = θ(2 10 ) = 11 .<br />

2.6.5. Z uwagi na zadanie 2.6.3 mamy<br />

(α 1 + 1)(α 2 + 1) . . . (α s + 1) = 3.<br />

Zatem musi być s = 1 oraz α 1 = 2 . Sta ↩<br />

d jedynymi liczbami n<br />

takimi, że θ(n) = 3 , sa ↩<br />

kwadraty liczb pierwszych.<br />

2.6.6.<br />

n 1 2 3 4 5 6 7 8 9 1011121314151617181920<br />

φ(n) 1 1 2 2 4 2 6 4 6 4 10 4 12 6 8 8 16 6 18 8<br />

2.6.7. Za̷lóżmy, że liczba n jest pierwsza. Wówczas każda liczba<br />

mniejsza od n jest z nia ↩<br />

wzgle ↩<br />

dnie pierwsza, wie ↩<br />

c φ(n) = n − 1 .<br />

Jeżeli teraz φ(n) = n − 1 , to wszystkie liczby naturalne mniejsze<br />

od n musza ↩<br />

być wzgle ↩<br />

dnie pierwsze z n . W szczególności, żadna<br />

z liczb mniejszych od n nie może być dzielnikiem n (z wyja ↩<br />

tkiem 1).<br />

Czyli n jest liczba ↩<br />

pierwsza ↩<br />

.


74 Cze ↩<br />

ść II – Rozwia ↩<br />

zania<br />

2.6.8. Ponieważ p oraz q sa ↩<br />

liczbami pierwszymi, wie ↩<br />

c<br />

NWD(pq, m) jest równe 1, p , q lub pq dla dowolnej liczby naturalnej<br />

m . Zatem liczbami, które nie sa ↩<br />

wzgle ↩<br />

dnie pierwsze z pq<br />

sa ↩<br />

wielokrotności p (tych mniejszych od pq jest dok̷ladnie q − 1 )<br />

oraz wielokrotności q (tych mniejszych od pq jest p−1 ). Ponieważ<br />

liczba pq nie jest wzgle ↩<br />

dnie pierwsza z pq , wie ↩<br />

c<br />

φ(pq) = pq − (p − 1) − (q − 1) − 1 = (p − 1)(q − 1).<br />

Uwaga. Czytelniku, jeśli nie zrozumia̷leś rozwia ↩<br />

zania zadania 2.6.8,<br />

to przeanalizuj rozwia ↩<br />

zanie zadania 2.6.9.<br />

2.6.9. Aby obliczyć φ(p n ) , należy z cia ↩<br />

gu 1, 2, 3 . . . , p n<br />

usuna ↩<br />

ć wszystkie te liczby, które nie sa ↩<br />

wzgle ↩<br />

dnie pierwsze z p n ,<br />

czyli te, które nie sa ↩<br />

wzgle ↩<br />

dnie pierwsze z p . Liczby, które nie sa ↩<br />

wzgle ↩<br />

dnie pierwsze z p sa ↩<br />

wielokrotnościami liczby p . Sa ↩<br />

to wie ↩<br />

c<br />

liczby postaci kp . Ponieważ musza ↩<br />

one być mniejsze lub równe p n ,<br />

wie ↩<br />

c sa ↩<br />

to liczby p , 2p , . . . , p n−1 p . Jest ich p n−1 . Po usunie ↩<br />

ciu<br />

tych p n−1 liczb z cia ↩<br />

gu 1, 2, 3 . . . , p n pozostaje w tym cia ↩<br />

gu<br />

p n − p n−1 liczb. Zatem φ(p n ) = p n − p n−1 .<br />

2.6.10. Za̷lóżmy, że dla pewnych 0 ≤ k ≤ h ≤ m − 1 liczby<br />

kn + r oraz hn + r daja ↩<br />

te ↩<br />

sama ↩<br />

reszte ↩<br />

przy dzieleniu przez m .<br />

Zatem (hn+r)−(kn+r) = (h−k)n jest liczba ↩<br />

podzielna ↩<br />

przez m .<br />

Ponieważ n i m sa ↩<br />

wzgle ↩<br />

dnie pierwsze, wie ↩<br />

c m|h − k . Jednakże<br />

0 ≤ h − k ≤ m − 1 , czyli musi być k − h = 0 , tj. k = h . Sta ↩<br />

d<br />

wynika, że reszty z dzielenia liczb r , n+r , . . . , (m−1)n+r przez<br />

m sa ↩<br />

różne i jest ich m . Zatem różnia ↩<br />

sie ↩<br />

od cia ↩<br />

gu 0 , 1, . . . , m−1<br />

co najwyżej porza ↩<br />

dkiem.<br />

2.6.11. Zauważmy, że w danej kolumnie wszystkie liczby sa ↩<br />

albo wzgle ↩<br />

dnie pierwsze z n , albo wszystkie maja ↩<br />

wspólny dzielnik z<br />

n wie ↩<br />

kszy od 1. Mamy dok̷ladnie φ(n) kolumn, w których wszystkie<br />

liczby sa ↩<br />

wzgle ↩<br />

dnie pierwsze z n . Z drugiej strony, w kolumnie<br />

maja ↩<br />

cej wszystkie liczby wzgle ↩<br />

dnie pierwsze z n jest dok̷ladnie<br />

φ(m) liczb wzgle ↩<br />

dnie pierwszych z m (na mocy zadania 2.6.10).<br />

Zatem φ(mn) = φ(m)φ(n) .<br />

2.6.12. Ponieważ każde dwie liczby pierwsze sa ↩<br />

wzgle ↩<br />

dnie<br />

pierwsze, wie ↩<br />

c<br />

φ (p α 1<br />

1 pα 2<br />

2 . . . pα s<br />

s ) =


Liczby pierwsze 75<br />

= φ (p α 1<br />

1 ) φ (pα 2<br />

2 ) . . . φ (pα s<br />

s )<br />

= ( p α 1<br />

1 − ) ( pα 1−1<br />

1 p<br />

α 2<br />

2 − ) ( pα 2−1<br />

2 . . . p<br />

α s<br />

s − p α )<br />

s−1<br />

s<br />

= p α 1<br />

1 pα 2<br />

2 . . . pα s<br />

s<br />

(1 − 1 ) (1 − 1 )<br />

· · ·<br />

(1 − 1 )<br />

p 1 p 2 p s<br />

= n<br />

(1 − 1 p 1<br />

) (1 − 1 p 2<br />

)<br />

· · ·<br />

(1 − 1 p s<br />

)<br />

.<br />

2.6.13. Niech NWD(m, n) = d i niech p be ↩<br />

dzie liczba ↩<br />

pierwsza ↩<br />

dziela ↩<br />

ca ↩<br />

d . Po zastosowaniu wzoru z 2.6.12 stwierdzamy, że w iloczynie<br />

opisuja ↩<br />

cym φ(mn) pojawia sie ↩<br />

czynnik 1 − 1 p<br />

jednokrotnie,<br />

natomiast w iloczynie opisuja ↩<br />

cym φ(m)φ(n) ten sam czynnik pojawia<br />

sie ↩<br />

( 2<br />

dwukrotnie. Ponieważ 1 − 1 p < 1 , wie ↩c 1 − p) 1 < 1 −<br />

1<br />

p .<br />

Zatem φ(m)φ(n) < φ(mn) .<br />

2.6.14.<br />

(a) φ(4n + 2) = φ(2(2n + 1)) = φ(2)φ(2n + 1) = φ(2n + 1) ;<br />

(b) Jeżeli n = 2 α k , gdzie NWD(2, k) = 1 , to φ(4n) =<br />

φ(2 α+2 k) = φ(2 α+2 )φ(k) = 2 α+1 φ(k) = 2φ(2 α+1 )φ(k) =<br />

2φ(2 α+1 k) = 2φ(2(2 α k)) = 2φ(2n) .<br />

Jeżeli n = 2k + 1 , to φ(4n) = φ(4(2k + 1)) = φ(4)φ(2k +<br />

1) = 2φ(n) .<br />

2.7. Rozk̷lad na czynniki dużych liczb naturalnych<br />

2.7.1. Niech<br />

A = { a ∈ N : a|n i a ≤ √ n } ,<br />

B = { a ∈ N : a|n i a ≥ √ n } .<br />

Funkcja f określona wzorem f(a) = n a<br />

jednoznacznie zbiór A na B .<br />

odwzorowuje wzajemnie<br />

2.7.2. Niech<br />

B = { a ∈ N : a|n i a ≥ √ n } ,<br />

(<br />

a −<br />

n<br />

a<br />

))<br />

odw-<br />

C = { (s,t) : s,t ∈ N i n = s 2 − t 2} .<br />

Funkcja f określona wzorem f(a) = ( ( )<br />

1<br />

2 a +<br />

n<br />

a ;<br />

1<br />

2<br />

zorowuje wzajemnie jednoznacznie zbiór B na C .


76 Cze ↩<br />

ść II – Rozwia ↩<br />

zania<br />

2.7.3. 945 = 7 · 5 · 3 3 , wie ↩<br />

c wszystkie dzielniki liczby 945 ,<br />

to 1, 3, 5, 7, 9, 15, 21, 27, 35, 45, 63, 105, 135, 189, 315, 945.<br />

Stosuja ↩<br />

c oznaczenia z rozwia ↩<br />

zań zadań 2.7.1 oraz 2.7.2 mamy: B =<br />

{35,45,63,105,135,189,315,945} . Wykorzystuja ↩<br />

c funkcje ↩<br />

z rozwia ↩<br />

zania<br />

zadania 2.7.2 otrzymujemy<br />

945 = 31 2 − 4 2 = 33 2 − 12 2 = 39 2 − 24 2 = 57 2 − 48 2<br />

= 71 2 − 64 2 = 97 2 − 92 2 = 159 2 − 156 2<br />

= 473 2 − 472 2 .<br />

2.7.4. 8633 = 93 2 − 4 2 = 89 · 97 , 809009 = 903 2 − 80 2 =<br />

823 · 983 , 92296873 = 9613 2 − 336 2 = 9277 · 9949 , 88169891 =<br />

9390 2 − 47 2 = 9343 · 9437 , 4601 = 75 2 − 32 2 = 43 · 107 .<br />

2.7.5.<br />

(a) 3 · 141467 = 655 2 − 68 2 = 587 · 723 i 141467 = 587 · 241 ;<br />

(b) 3 · 29895581 = 9472 2 − 179 2 = 9293 · 9651 i 29895581 =<br />

9293 · 3217 ;<br />

(c) 3 · 68987 = 455 2 − 8 2 = 463 · 447 i 68987 = 149 · 463 ;<br />

(d) 5 · 19578079 = 9894 2 − 29 2 = 9923 · 9865 i 19578079 =<br />

9923 · 1973 .


3. Liczby w różnych systemach pozycyjnych<br />

3.1. Poje ↩<br />

cie pozycyjnego systemu zapisu liczb<br />

3.1.1. Dobrze znanym niepozycyjnym systemem zapisu liczb<br />

jest system rzymski. Cyframi sa ↩<br />

tu I, V, X, L, C, D, M. Jednak to nie<br />

pozycja cyfry określa jej znaczenie w liczbie. Na przyk̷lad XIX (19)<br />

jest liczba ↩<br />

mniejsza ↩<br />

od XX (20) chociaż XIX ma wie ↩<br />

cej cyfr. Wie ↩<br />

cej<br />

informacji na temat liczb rzymskich, a także podobnego systemu<br />

liczb greckich znajdzie Czytelnik w ksia ↩<br />

żce Georges’a Ifraha Dzieje<br />

liczby, czyli historia wielkiego wynalazku (Zak̷lad Narodowy im.<br />

Ossolińskich, Wroc̷law 1990).<br />

3.1.2. Odpowiedzi wynikaja ↩<br />

z prostych obliczeń:<br />

(a) (3423) 5 = 3 · 125 + 4 · 25 + 2 · 5 + 3 = 488 ;<br />

(b) (11000111) 2 = 128 + 64 + 4 + 2 + 1 = 199 ;<br />

(c) Ponieważ w systemie o podstawie 11 mamy 11 cyfr, wyste<br />

↩<br />

puje tu dwuznaczność. Mianowicie, 10 może być traktowane<br />

jako pojedyncza cyfra lub też jako dwie cyfry.<br />

W pierwszym przypadku, (910) 11 = 9 · 11 + 10 = 109 .<br />

Natomiast w drugim, 9 · 121 + 11 = 1100 . Zobacz<br />

w naste ↩<br />

pnym zadaniu jak pomina ↩<br />

ć wspomniana ↩<br />

wieloznaczność.<br />

3.1.3.<br />

(a) Z uwag poczynionych na pocza ↩<br />

tku zadania wynika, że<br />

(910) 11 ma trzy cyfry, wie ↩<br />

c w systemie dziesie ↩<br />

tnym o-<br />

znacza liczbe ↩<br />

1100 (zobacz poprzednie rozwia ↩<br />

zanie).<br />

(b) W systemie o podstawie 16 cyfra A odpowiada liczbie 10,<br />

natomiast D liczbie 13. Zatem (1AD) 16 = 256+10·16+<br />

13 = 429 .<br />

(c) W systemie o podstawie 12, cyfra A odpowiada liczbie 10,<br />

a B liczbie 11. Sta ↩<br />

d (A2B) 12 = 10 · 144 + 2 · 12 + 11 =<br />

1475 .<br />

3.1.4. Jest wiele sposobów na zamiane ↩<br />

zapisu liczby n<br />

w systemie dziesie ↩<br />

tnym na zapis w systemie o podstawie b . Dwa<br />

najcze ↩<br />

ściej stosowane, to:<br />

1) Wypisujemy kolejne pote ↩<br />

gi liczby b , dopóki nie przekroczymy<br />

liczby n . Znajdziemy w ten sposób najwie ↩<br />

ksza ↩<br />

liczbe ↩<br />

k


78 Cze ↩<br />

ść II – Rozwia ↩<br />

zania<br />

taka ↩<br />

, że b k ≤ n . Z określenia liczby k wynika, że b k+1 > n<br />

(w przeciwnym wypadku, k nie by̷loby najwie ↩<br />

ksza ↩<br />

liczba ↩<br />

taka ↩<br />

,<br />

że b k ≤ n ). Zatem b k ≤ n < b k+1 . Dzielimy teraz n przez<br />

b k i oznaczamy wynik dzielenia przez a k , a otrzymana ↩<br />

reszte ↩<br />

przez r 1 . Mamy zatem n = a k b k + r 1 , gdzie 0 ≤ r 1 < b k .<br />

Zauważmy, że 1 ≤ a k < b . Gdyby a k = 0 , to mielibyśmy<br />

n = r 1 < b k . Natomiast gdyby a k ≥ b , to n = a k b k +<br />

r 1 > a k b k ≥ b · b k = b k+1 . Zatem liczba a k jest cyfra ↩<br />

w<br />

systemie o podstawie b , a dok̷ladnie, pierwsza ↩<br />

cyfra ↩<br />

liczby n .<br />

Jeżeli r 1 = 0 , to mamy n = a k b k + 0b k−1 + · · · + 0 . Jeśli<br />

natomiast r 1 > 0 , to dzielimy r 1 przez b k−1 i otrzymujemy<br />

r 1 = a k−1 b k−1 + r 2 , gdzie 0 ≤ r 2 < b k−1 oraz 0 ≤ a k−1 < b .<br />

Liczba a k−1 jest druga ↩<br />

cyfra ↩<br />

liczby n . Powtarzamy opisana ↩<br />

procedure ↩<br />

, aż otrzymamy wszystkie cyfry liczby n w systemie<br />

o podstawie b .<br />

2) W przeciwieństwie do poprzedniego algorytmu, tutaj szukamy<br />

najpierw ostatniej cyfry. Dok̷ladniej, szukamy reszty z dzielenia<br />

n przez b . Ta reszta, powiedzmy a 0 , jest ostatnia ↩<br />

cyfra ↩<br />

n<br />

w zapisie o podstawie b . W celu znalezienia przedostatniej<br />

cyfry, dzielimy n−a 0 przez b i dla otrzymanej liczby szukamy<br />

reszty a 1 z dzielenia jej przez b . Wtedy a 1 jest przedostatnia ↩<br />

cyfra ↩<br />

. Poste ↩<br />

pujemy tak dalej aż do znalezienia wszystkich cyfr<br />

liczby n .<br />

Zastosujemy teraz powyższe algorytmy w praktyce.<br />

(a) Poszukamy najpierw zapisu liczby 437 w systemie trójkowym<br />

stosuja ↩<br />

c pierwszy algorytm. Kolejnymi pote ↩<br />

gami<br />

liczby 3 sa ↩<br />

1, 3, 9, 27, 81, 243, 729, . . . . Zatem k = 5 .<br />

Mamy teraz 437 = 1 · 3 5 + 194 . Zatem pierwsza ↩<br />

cyfra ↩<br />

jest<br />

1. Teraz mamy 194 = 2·3 4 +32 , a naste ↩<br />

pnie 32 = 1·3 3 +5 .<br />

Tym razem 3 2 > 5 , wie ↩<br />

c kolejna ↩<br />

(trzecia ↩<br />

) cyfra ↩<br />

jest 0,<br />

a pozosta̷le cyfry to 1 i 2. Zatem 437 = (121012) 3 . Nasze<br />

rozumowanie możemy zapisać w skrócie naste ↩<br />

puja ↩<br />

co<br />

437 = 1 · 3 5 + 194<br />

= 1 · 3 5 + 2 · 3 4 + 32<br />

= 1 · 3 5 + 2 · 3 4 + 1 · 3 3 + 5<br />

= 1 · 3 5 + 2 · 3 4 + 1 · 3 3 + 0 · 3 2 + 1 · 3 1 + 2 · 3 0 .


Liczby w różnych systemach pozycyjnych 79<br />

Zademonstrujemy teraz drugi algorytm. Mamy tutaj<br />

437 = 145 · 3 + 2 , wie ↩<br />

c ostatnia ↩<br />

cyfra ↩<br />

jest 2. Znajdujemy<br />

teraz reszte ↩<br />

z dzielenia 145 na 3. Mamy 145 = 48 · 3 + 1 .<br />

Druga ↩<br />

od końca cyfra ↩<br />

jest wie ↩<br />

c 1. 48 dzieli sie ↩<br />

przez 3<br />

(reszta jest wtedy równa 0), wie ↩<br />

c naste ↩<br />

pna ↩<br />

cyfra ↩<br />

jest 0.<br />

48 : 3 = 16 , czyli szukamy teraz reszty z dzielenia 16 na<br />

3. Dostajemy 16 = 5 · 3 + 1 , a zaraz potem 5 = 1 · 3 + 2<br />

oraz 1 = 0 · 3 + 1 . Zbieraja ↩<br />

c wszystkie reszty dostajemy<br />

zapis 121012. W skrócie możemy zapisać to naste ↩<br />

puja ↩<br />

co:<br />

437 = 145 · 3 + 2<br />

= (48 · 3 + 1) · 3 + 2<br />

= ((16 · 3) · 3 + 1) · 3 + 2<br />

= (((5 · 3 + 1) · 3) · 3 + 1) · 3 + 2<br />

= ((((1 · 3 + 2) · 3 + 1) · 3) · 3 + 1) · 3 + 2<br />

= 1 · 3 5 + 2 · 3 4 + 1 · 3 3 + 0 · 3 2 + 1 · 3 1 + 2 · 3 0 .<br />

(b) Zastosujemy tu pierwszy algorytm zamiany zapisu dziesie<br />

↩<br />

tnego na system o podstawie b . Mamy<br />

437 = 3 · 5 3 + 62<br />

= 3 · 5 3 + 2 · 5 2 + 12<br />

= 3 · 5 3 + 2 · 5 2 + 2 · 5 1 + 2 · 5 0 ,<br />

zatem 437 = (3222) 5 .<br />

(c) Wykorzystamy tu drugi z opisanych algorytmów. Mamy<br />

437 = 27 · 16 + 5<br />

= (1 · 16 + 11) · 16 + 5<br />

= 1 · 16 2 + 11 · 16 1 + 5 · 16 0 .<br />

Ponieważ 11 oznaczamy przez B , wie ↩<br />

c 437 = (1B5) 16 .<br />

(d) Skorzystamy tu z (c). Mianowicie, 16 = 2 4 , 11 = 2 3 +<br />

2 1 + 2 0 , a 5 = 2 2 + 2 0 . Sta ↩<br />

d<br />

1 · 16 2 + 11·16 1 + 5 · 16 0<br />

= 1 · (2 4 ) 2 + (2 3 + 2 1 + 2 0 ) · 2 4 + 2 2 + 2 0<br />

= 2 8 + 2 7 + 2 5 + 2 4 + 2 2 + 2 0<br />

= 1 · 2 8 + 1 · 2 7 + 0 · 2 6 + 1 · 2 5 + 1 · 2 4<br />

+ 0 · 2 3 + 1 · 2 2 + 0 · 2 1 + 1 · 2 0 .


80 Cze ↩<br />

ść II – Rozwia ↩<br />

zania<br />

Zatem 437 zapisujemy w systemie o podstawie dwa jako<br />

(110110101) 2 .<br />

3.1.5. Ponieważ sposób opisany w treści zadania nie wymaga<br />

szczegó̷lowego wyjaśnienia, do rozwia ↩<br />

zania zastosujemy inne metody.<br />

(a) Ponieważ w (12) 3 = 1 · 3 + 2 , a 3 = 2 + 1 , możemy<br />

zapisać (12) 3 jako 2 + 1 + 2 = 2 2 + 1 , co daje nam zapis<br />

dwójkowy (101) 2 .<br />

(b) (1A) 12 = 1 · 12 + 10 = 12 + 4 + 6 = 16 + 6 . Wie ↩<br />

c<br />

(1A) 12 = (16) 16 .<br />

(c) Tym razem mamy liczbe ↩<br />

3 · 6 3 +4 · 6 2 + 5 · 6 + 5<br />

= 3 · 216 + 4 · 36 + 5 · 6 + 5<br />

= 3 · (3 · 4 3 + 1 · 4 2 + 2 · 4) + 4 · (2 · 4 2 + 4)<br />

+ 5 · (4 + 2) + 4 + 1<br />

= 11 · 4 3 + 4 · 4 2 + 12 · 4 + 11<br />

= (2 · 4 + 3) · 4 3 + 4 3 + 3 · 4 2 + 2 · 4 + 3<br />

= 3 · 4 4 + 3 · 4 2 + 2 · 4 + 3,<br />

co w systemie o podstawie 4 zapisujemy (30323) 4 .<br />

(d) Liczba (101011101011) 2 oznacza 2 11 + 2 9 + 2 7 + 2 6 +<br />

2 5 + 2 3 + 2 + 1 . Ale 2 4 = 16 , wie ↩<br />

c możemy zapisać<br />

(2 3 + 2) · 16 2 + (2 3 + 2 2 + 2) · 16 + 2 3 + 2 + 1 . Ostatnie<br />

wyrażenie upraszczamy i otrzymujemy 10·16 2 +14·16+11 ,<br />

co oznacza (AEB) 16 .<br />

3.1.6.<br />

(a) Ponieważ 27 = 3 3 , grupujemy cyfry 1112 w bloki trzycyfrowe.<br />

Sa ↩<br />

to 001 oraz 112. Zauważmy, że ponieważ<br />

nasza liczba ma 4 cyfry, musimy dodać z przodu 2 zera,<br />

aby liczba cyfr po dodaniu zer by̷la podzielna przez 3. 001<br />

oznacza cyfre ↩<br />

1, a (112) 3 = 3 2 + 3 + 2 = 14 oznacza cyfre ↩<br />

E w systemie o podstawie 27. Zatem (1112) 3 = (1E) 27 .<br />

(b) Mamy 144 = 12 2 , wie ↩<br />

c nasza ↩<br />

liczbe ↩<br />

dzielimy na bloki<br />

po dwie cyfry. Ponieważ 1A ma już dwie cyfry, wie ↩<br />

c


Liczby w różnych systemach pozycyjnych 81<br />

jest to już cyfra w zapisie o podstawie 144. Ale 12 +<br />

10 = 22 , wie ↩<br />

c jest to M . Zauważmy, że aby liczby od<br />

10 do 143 zasta ↩<br />

pić literami, brakuje nam znaków w alfabecie<br />

̷lacińskim. Ponieważ generalnie nie używamy takich<br />

dużych podstaw, ma̷lo kto sie ↩<br />

martwi, jak oznaczyć<br />

cyfry 36, 37 itd. Można wykorzystywać jeszcze ma̷le litery,<br />

ale i ich nie starczy. Może wie ↩<br />

c najkorzystniej jest zrezygnować<br />

z liter i stosować nawiasy W takim wypadku, cyfre ↩<br />

M oznaczymy po prostu (22) .<br />

(c) Ponieważ 16 = 2 4 , grupujemy cyfry naszej liczby w bloki<br />

po cztery znaki: 1010, 1110 i 1011. Oznaczaja ↩<br />

one<br />

odpowiednio 10, 14 i 11, co w systemie szesnastkowym<br />

oznacza odpowiednio cyfry A , E oraz B . Zatem<br />

(101011101011) 2 = (AEB) 16 .<br />

3.1.7. (Zobacz też rozwia ↩<br />

zanie 3.1.4(d)) Aby zamienić liczbe ↩<br />

x = x 0 + x 1 (b s ) 1 + x 2 (b s ) 2 + · · · + x k (b s ) k zapisana ↩<br />

w systemie<br />

o podstawie b s na zapis w systemie o podstawie b , każda ↩<br />

cyfre ↩<br />

x i<br />

zapisujemy w systemie o podstawie b , tj. piszemy x i = x i0 +x i1 b 1 +<br />

x i2 b 2 +· · ·+x i s−1 b s−1 . Zauważmy, że x i nie może mieć wie ↩<br />

cej cyfr<br />

w systemie o podstawie b niż s , ponieważ x i < b s . Zatem każdy x i<br />

zamieniamy w blok s cyfr. W celu uzyskania zapisu naszej liczby<br />

w systemie o podstawie b , bloki ̷la ↩<br />

czymy, a naste ↩<br />

pnie usuwamy<br />

ewentualne zera wyste ↩<br />

puja ↩<br />

ce z lewej strony zapisu.<br />

(a) Mamy 2 = 0·4+2 oraz B = 2·4+3 , zatem otrzymaliśmy<br />

bloki 02 i 23 . Po po̷la ↩<br />

czeniu otrzymujemy ża ↩<br />

dany zapis<br />

0223 , co po usunie ↩<br />

ciu zbe ↩<br />

dnych zer daje (223) 4 .<br />

(b) Tutaj mamy A = 2 · 5 + 0 , B = 2 · 5 + 1 , C = 2 · 5 + 2<br />

i F = 3 · 5 + 0 , co nam daje bloki 20, 21, 22, 30 oraz zapis<br />

(20212230) 5 .<br />

(c) Zapisujemy<br />

3 = 0 · 2 3 + 0 · 2 2 + 1 · 2 + 1,<br />

A = 1 · 2 3 + 0 · 2 2 + 1 · 2 + 0,<br />

5 = 0 · 2 3 + 1 · 2 2 + 0 · 2 + 1,<br />

6 = 0 · 2 3 + 1 · 2 2 + 1 · 2 + 0,<br />

co nam daje bloki 0011 , 1010, 0101 oraz 0110. Ostatecznie<br />

otrzymujemy wie ↩<br />

c (11101001010110) 2 .


82 Cze ↩<br />

ść II – Rozwia ↩<br />

zania<br />

3.1.8. Ponieważ d i ≤ b − 1 , wie ↩<br />

c<br />

p = d k b k + d k−1 b k−1 + · · · + d 1 b + d 0<br />

≤ (b − 1)b k + (b − 1)b k−1 + · · · + (b − 1)b + (b − 1)<br />

= (b − 1)(b k + b k−1 + · · · + b + 1)<br />

= (b − 1) bk+1 − 1<br />

b − 1<br />

= b k+1 − 1 < b k+1 .<br />

W za̷lożeniach mamy k < n , zatem k + 1 ≤ n . Sta ↩<br />

d<br />

p < b k+1 ≤ b n ≤ c n b n + c n−1 b n−1 + · · · + c 1 b + c 0 = m.<br />

3.2. Wykonywanie obliczeń w różnych systemach pozycyjnych<br />

3.2.1.<br />

(a)<br />

AB 1 2 3<br />

+ C D A<br />

AC 0 1 D<br />

(b)<br />

1111001101<br />

+ 1101101101<br />

11100111010<br />

(c)<br />

12321<br />

+ 23132<br />

41003<br />

3.2.2. Przy obliczaniu różnicy dwóch liczb sprawdzamy najpierw,<br />

czy odjemna jest wie ↩<br />

ksza od odjemnika. Jeżeli nie, to zamieniamy<br />

nasze liczby rolami, tj. odejmujemy odjemna ↩<br />

od odjemnika<br />

i pamie ↩<br />

tamy, że ostateczny wynik ma być ujemny. Przypuśćmy,<br />

że<br />

m = c n b n + c n−1 b n−1 + · · · c 1 b + c 0<br />

p = d k b k + d k−1 b k−1 + · · · d 1 b + d 0 ,<br />

przy czym m > p , wie ↩<br />

c także n ≥ k .<br />

Obliczaja ↩<br />

c różnice ↩<br />

m − p również posuwamy sie ↩<br />

od strony<br />

prawej do lewej. Jeżeli c 0 ≥ d 0 , to c 0 −d 0 jest ostatnia ↩<br />

cyfra ↩<br />

różnicy<br />

m − p . W przeciwnym wypadku ,,pożyczamy” 1 od cyfry wyższego<br />

rze ↩<br />

du (tu przedostatniej) i dostajemy (c 1 b + c 0 ) − (d 1 b + d 0 ) =<br />

(c 1 − d 1 − 1)b + (b + c 0 − d 0 ) , gdzie 0 < b + c 0 − d 0 < b − 1 , zatem<br />

b + c 0 − d 0 jest ostatnia ↩<br />

cyfra ↩<br />

liczby m − p . Tak samo poste ↩<br />

pujemy<br />

z kolejnymi cyframi.


Liczby w różnych systemach pozycyjnych 83<br />

(a)<br />

1 1 1 0 1 0 1 1 1 0<br />

− 1 1 1 0 1 0 1 1<br />

1 0 1 1 0 0 0 0 1 1<br />

(b)<br />

4532<br />

− 3401<br />

1131<br />

(c)<br />

AB 1 2<br />

− C DA<br />

9 E 3 8<br />

3.2.3. Opiszemy najpierw, jak mnożymy liczbe ↩<br />

wielocyfrowa ↩<br />

y = c n b n +c n−1 b n−1 +· · ·+c 1 b+c 0 przez jednocyfrowa ↩<br />

d . Ponieważ<br />

yd = (c n d)b n + (c n−1 d)b n−1 + · · · + (c 1 d)b + (c 0 d),<br />

wie ↩<br />

c jeżeli iloczyn dc 0 jest liczba ↩<br />

mniejsza ↩<br />

od b , to jest ona ostatnia ↩<br />

cyfra ↩<br />

liczby yd . Jeśli dc 0 ≥ b , to wyodre ↩<br />

bniamy odpowiednia ↩<br />

liczbe ↩<br />

wyrazów wyższego rze ↩<br />

du dodaja ↩<br />

c ja ↩<br />

do dc 1 itd.<br />

(a)<br />

2 3 4<br />

× 4<br />

2 1 0 1<br />

(c)<br />

A B 3 2<br />

× 9<br />

7 6 A 2 5<br />

Aby pomnożyć liczbe ↩<br />

wielocyfrowa ↩<br />

y przez liczbe ↩<br />

wielocyfrowa ↩<br />

x mnożymy y przez kolejne (licza ↩<br />

c od końca) cyfry x i zapisujemy<br />

iloczyny tak, by ich ostatnie cyfry znajdowa̷ly sie ↩<br />

dok̷ladnie pod<br />

ta ↩<br />

cyfra ↩<br />

liczby x , przez która ↩<br />

mnożyliśmy liczbe ↩<br />

y . Naste ↩<br />

pnie<br />

dodajemy zapisane iloczyny i otrzymujemy ostateczny wynik.<br />

3.2.4. Przypuśćmy, że<br />

(b)<br />

1 1 1 1 0 1 0 1<br />

× 1 1 0 1<br />

1 1 1 1 0 1 0 1<br />

0<br />

1 1 1 1 0 1 0 1<br />

+ 1 1 1 1 0 1 0 1<br />

1 1 0 0 0 1 1 1 0 0 0 1<br />

m = c n b n + c n−1 b n−1 + · · · + c 1 b + c 0<br />

p = d k b k + d k−1 b k−1 + · · · + d 1 b + d 0 ,<br />

przy czym m > p > 0 . Dzielenie liczby m przez liczbe ↩<br />

p polega<br />

na znalezieniu liczb ca̷lkowitych q i r takich, że m = pq + r ,


84 Cze ↩<br />

ść II – Rozwia ↩<br />

zania<br />

gdzie 0 ≤ r < p . Be ↩<br />

dziemy szukali przedstawienia liczb q oraz r<br />

w systemie o podstawie b . W tym celu rozpatrujemy liczby<br />

c n , c n b + c n−1 , c n b 2 + c n−1 b + c n−2 , . . .<br />

i szukamy pierwszej liczby w tym cia ↩<br />

gu, która jest wie ↩<br />

ksza od p .<br />

Za̷lóżmy, że jest to liczba<br />

s 1 = c n b j + c n−1 b j−1 + · · · + c n−j .<br />

Naste ↩<br />

pnie rozpatrujemy kolejne iloczyny 0 , p , 2p , 3p , . . . (b −<br />

1)p i znajdujemy najwie ↩<br />

kszy z nich nie przekraczaja ↩<br />

cy s 1 . Niech<br />

to be ↩<br />

dzie a n−j p . Wtedy s 1 = a n−j p + r 1 , gdzie r 1 jest reszta ↩<br />

z dzielenia s 1 przez p . Możemy to zapisać w sposób naste ↩<br />

puja ↩<br />

cy:<br />

a n−j<br />

c n c n−1 . . . c n−j c n−j−1 . . . c 1 c 0 : d k d k−1 . . . d 1 d 0<br />

□ □ . . . □<br />

e j e j−1 . . . e 0<br />

Kwadraciki oznaczaja ↩<br />

tu cyfry iloczynu a n−j p . Po odje ↩<br />

ciu otrzymamy<br />

liczbe ↩<br />

r 1 = s 1 − a n−j p = e j b j + e j−1 b j−1 + · · · + e 0 .<br />

Naste ↩<br />

pnie do cyfr e j , e j−1 , . . . , e 0 dopisujemy cyfre ↩<br />

c n−j−1 , co<br />

oznacza, że rozważamy liczbe ↩<br />

s 2 = e j b j+1 + e j−1 b j + · · · + e 0 b + c n−j−1 .<br />

Podobnie jak poprzednio, szukamy liczb a n−j−1 oraz r 2 takich, że<br />

s 2 = a n−j−1 p+r 2 , gdzie 0 ≤ r 2 < p . Kontynuuja ↩<br />

c to poste ↩<br />

powanie<br />

otrzymujemy w końcu q i r .<br />

(a)<br />

1 0 0 1<br />

1 1 0 1 1 : 11<br />

1 1<br />

0 0 1 1<br />

1 1<br />

0<br />

(b)<br />

1 3 3<br />

2 3 6 1 : 15<br />

1 5<br />

5 6<br />

5 1<br />

5 1<br />

5 1<br />

0<br />

(c)<br />

1 C A<br />

A B C : 6<br />

6<br />

4 B<br />

4 8<br />

3 C<br />

3 C<br />

0


Liczby w różnych systemach pozycyjnych 85<br />

3.2.5. Liczbe ↩<br />

(11011101) 2 dzielimy przez b = 3 = (11) 2<br />

otrzymuja ↩<br />

c<br />

(11011101) 2 = (1001001) 2 · (11) 2 + (10) 2 .<br />

Wynik powyższego dzielenia dzielimy znów przez b:<br />

(1001001) 2 = (11000) 2 · (11) 2 + 1.<br />

Poste ↩<br />

pujemy podobnie z kolejnymi wynikami:<br />

(11000) 2 = (1000) 2 · (11) 2<br />

(1000) 2 = (10) 2 · (11) 2 + (10) 2<br />

(10) 2 = 0 · (11) 2 + (10) 2 .<br />

Reszty z tych dzieleń wypisane w odwrotnej kolejności, tj.<br />

(10) 2 (10) 2 (0) 2 (1) 2 (10) 2<br />

daja ↩<br />

nam szukane cyfry rozważanej liczby. Zatem (11011101) 2 =<br />

(22012) 3 .<br />

3.2.6. Szukamy sumy MCMXXIV + CDXCIII. Liczba MCMXXIV, to<br />

jeden pe̷lny tysia ↩<br />

c oraz drugi tysia ↩<br />

c bez jednej setki i coś tam jeszcze.<br />

Bierzemy zatem jedna ↩<br />

setke ↩<br />

z drugiej liczby i dok̷ladamy ja ↩<br />

do<br />

pierwszej. Nasza ↩<br />

sume ↩<br />

możemy teraz zapisać jako MMXXIV + CCCXCIII<br />

i podliczamy wyste ↩<br />

puja ↩<br />

ce w niej setki. Drugi sk̷ladnik ma ich<br />

trzy plus jedna ↩<br />

bez dziesia ↩<br />

tki. Bierzemy wie ↩<br />

c brakuja ↩<br />

ca ↩<br />

dziesia ↩<br />

tke ↩<br />

z pierwszej liczby i otrzymujemy równoważna ↩<br />

sume ↩<br />

MMXIV + CDIII.<br />

Teraz już ̷latwo obliczamy dziesia ↩<br />

tki i jedności naszej sumy. Ostatecznie<br />

otrzymujemy MMCDXVII (2417). Autorom ten sposób liczenia<br />

nie wydaje sie ↩<br />

̷latwy i dlatego nie próbowali mnożenia.<br />

3.3. U̷lamki w różnych systemach pozycyjnych<br />

3.3.1.<br />

(a) Cze ↩<br />

ścia ↩<br />

ca̷lkowita ↩<br />

liczby 1 3 jest 0, a cze 1<br />

↩ścia ↩<br />

u̷lamkowa ↩ 3 .<br />

Mamy 1 3 · 3 = 1 , wie ↩c jedyna ↩<br />

cyfra ↩<br />

po przecinku be ↩<br />

dzie 1.<br />

Zatem 1 3<br />

w systemie trójkowym, to 0,1 .<br />

Cze ↩<br />

ścia ↩<br />

ca̷lkowita ↩<br />

0,5 jest ponownie 0. 0,5 · 3 = 1,5 , wie ↩<br />

c<br />

pierwsza ↩<br />

cyfra ↩<br />

po przecinku be ↩<br />

dzie 1. Ponieważ cze ↩<br />

ścia ↩


86 Cze ↩<br />

ść II – Rozwia ↩<br />

zania<br />

u̷lamkowa ↩<br />

liczby 1,5 jest ponownie 0,5 , druga ↩<br />

cyfra ↩<br />

też<br />

jest 1 i znów sytuacja sie ↩<br />

powtórzy. Zatem w zapisie<br />

trójkowym, 0,5 to 0,(1) (w nawiasie zapisujemy okres<br />

u̷lamka).<br />

(b) Mamy π = 3,1415 . . . Cze ↩<br />

ść ca̷lkowita π to 11 w systemie<br />

dwójkowym. Mamy teraz 0,1415 · 2 = 0,283 , wie ↩<br />

c<br />

d −1 = 0 ; 0,283 · 2 = 0,566 , zatem także d −2 = 0 ;<br />

0,566 · 2 = 1,132 i d −3 = 1 ; 0,132 · 2 = 0,264 , a to<br />

daje d −4 = 0 i wreszcie 0,264 · 2 = 0,528 , co nam daje<br />

d −5 = 0 . Zatem w systemie dwójkowym liczba π ma<br />

rozwinie ↩<br />

cie rozpoczynaja ↩<br />

ce sie ↩<br />

od 11,00100 . Zauważmy<br />

tu, że nasze obliczenie jest dok̷ladne, tzn. nie zależy od<br />

dalszych cyfr rozwinie ↩<br />

cia dziesie ↩<br />

tnego π .<br />

(c) By obliczyć kolejne cyfry rozwinie ↩<br />

cia pia ↩<br />

tkowego liczby 5 13<br />

5<br />

wykonujemy naste ↩<br />

puja ↩<br />

ce mnożenia.<br />

13 · 5 = 1 12<br />

13 , 12<br />

13 ·<br />

5 = 4 8 13 , 8<br />

13 · 5 = 3 1<br />

13 , 1<br />

13 · 5 = 5<br />

13<br />

i wróciliśmy do<br />

punktu wyjścia. Zatem rozwinie ↩<br />

cie liczby 5 13 w systemie<br />

pia ↩<br />

tkowym jest u̷lamkiem okresowym 0,(1430) .<br />

Oczywiście, możemy zapisywać też u̷lamki zwyk̷le stosuja ↩<br />

c<br />

różne podstawy. Wtedy u̷lamki z punktu (a) zapiszemy jako 1 10<br />

1<br />

i<br />

2 , a 5<br />

13 w zapisie dziesie ↩tnym to 10<br />

23<br />

w systemie o podstawie 5.<br />

a<br />

3.3.2. Niech<br />

b−1<br />

be ↩dzie naszym u̷lamkiem. Ponieważ jest on<br />

u̷lamkiem w̷laściwym, wie ↩<br />

c 1 ≤ a ≤ b−2 oraz jego cze ↩<br />

ścia ↩<br />

ca̷lkowita ↩<br />

jest 0. Mnożymy zatem<br />

a<br />

b−1 przez b otrzymuja ab<br />

↩c<br />

b−1 = a + a<br />

b−1 .<br />

a<br />

Cze ↩<br />

ścia ↩<br />

ca̷lkowita ↩<br />

jest tu a , a u̷lamkowa ↩<br />

ponownie<br />

b−1 . Zatem<br />

rozwinie ↩<br />

cie u̷lamka w systemie o podstawie b to 0,(a) .<br />

a<br />

b−1<br />

a<br />

3.3.3. Rozważamy tu u̷lamek<br />

b+1<br />

, gdzie 1 ≤ a ≤ b , zapisany<br />

w systemie dziesie ↩<br />

tnym. Chcemy znaleźć jego rozwinie ↩<br />

cie w systemie<br />

a<br />

b+1−a<br />

o podstawie b . Mnoża ↩<br />

c przez b mamy<br />

b+1 · b = (a − 1) +<br />

b+1<br />

,<br />

ska ↩<br />

d b+1−a<br />

a<br />

b+1 ·b = (b−a)+<br />

b+1 . Ponownie mamy wie a<br />

↩c<br />

b+1 . Otrzymujemy<br />

zatem rozwinie ↩<br />

cie 0,((a − 1)(b − a)) . Na przyk̷lad, w systemie<br />

2<br />

dziesie ↩<br />

tnym,<br />

11 = 0,(18) .<br />

3.3.4. Za̷lóżmy najpierw, że u̷lamek a c<br />

, zapisany w postaci<br />

nieskracalnej, ma skończone rozwinie ↩<br />

cie w systemie o podstawie b .


Liczby w różnych systemach pozycyjnych 87<br />

Możemy za̷lożyć, że a c<br />

jest u̷lamkiem w̷laściwym. Zapiszmy jego<br />

rozwinie ↩<br />

cie w postaci 0,d 1 d 2 . . . d k , czyli<br />

a<br />

c = d 1b −1 + d 2 b −2 + · · · + d k b −k = d 1<br />

b + d 2<br />

b 2 + · · · + d k<br />

b k .<br />

Po sprowadzeniu do wspólnego mianownika otrzymujemy<br />

a<br />

c = d 1b k−1 + d 2 b k−2 + · · · + d k<br />

b k .<br />

Po ewentualnym skróceniu dostajemy, że c jest dzielnikiem b k , wie ↩<br />

c<br />

jest iloczynem pote ↩<br />

g dzielników b .<br />

Na odwrót, jeżeli mianownik u̷lamka a c<br />

jest iloczynem pote ↩<br />

g<br />

dzielników b , to istnieje takie k , że c dzieli b k . Zatem w k-tym<br />

kroku algorytmu na szukanie rozwinie ↩<br />

cia a c<br />

w systemie o podstawie<br />

b , otrzymujemy cze ↩<br />

ść u̷lamkowa ↩<br />

równa ↩<br />

0. Zatem a c<br />

ma<br />

rozwinie ↩<br />

cie skończone.<br />

3.3.5. Za̷lóżmy, że a c<br />

jest zapisany w postaci nieskracalnej<br />

i ma w systemie o podstawie b rozwinie ↩<br />

cie okresowe czyste o okresie<br />

d 1 d 2 . . . d f . Niech d = d 1 b −1 +d 2 b −2 +· · ·+d f b −f . Wtedy możemy<br />

zapisać<br />

a<br />

c = d + d b f + d<br />

b 2f + · · ·<br />

Korzystaja ↩<br />

c ze wzoru na sume ↩<br />

wszystkich wyrazów cia ↩<br />

gu geometrycznego<br />

dostajemy<br />

a<br />

c = d · 1 dbf<br />

=<br />

1 − b−f b f − 1 .<br />

Ponieważ a c<br />

jest zapisany w postaci nieskracalnej, wie ↩c c musi być<br />

dzielnikiem b f − 1 .<br />

Za̷lóżmy teraz, że b f −1 jest wielokrotnościa ↩<br />

c . Zatem u̷lamek<br />

a<br />

c możemy rozszerzyć do d<br />

b f −1 . Ale a < c , wie ↩c d < b f − 1 , a sta ↩<br />

d<br />

wynika, że liczba d ma w zapisie w systemie o podstawie b co<br />

najwyżej f cyfr. Możemy wie ↩<br />

c ja ↩<br />

zapisać d = d 1 b 0 + d 2 b 1 + · · · +<br />

d f b f−1 . Ale<br />

d<br />

b f − 1 = d b f · 1<br />

1 − b −f = d b f + d<br />

b 2f + d<br />

b 3f + · · · .<br />

Skorzystaliśmy tu ze wzoru na sume ↩<br />

wszystkich wyrazów cia ↩<br />

gu geometrycznego<br />

o ilorazie b −f i pierwszym wyrazie db −f . Zatem<br />

a<br />

w systemie o podstawie b ,<br />

c = d<br />

b f −1 = 0,(d f . . . d 2 d 1 ) .


88 Cze ↩<br />

ść II – Rozwia ↩<br />

zania<br />

3.3.6. Przy dodawaniu i odejmowaniu pisemnym pamie ↩<br />

tamy,<br />

że przecinek musi sie ↩<br />

znajdować dok̷ladnie pod przecinkiem. Poza<br />

tym algorytmy sa ↩<br />

podobne do opisanych w 3.2.<br />

(a)<br />

(b)<br />

111,101010<br />

121,2101<br />

+ 1,11<br />

− 12,01211<br />

1001,011010<br />

102,12022<br />

Przy dzieleniu, najpierw przesuwamy przecinki w dzielniku<br />

i w dzielnej tak, by rozwinie ↩<br />

cia nie mia̷ly cyfr po przecinku. Jeśli<br />

pozostaje reszta, to dopisujemy do niej 0 i dalej wykonujemy nasz algorytm,<br />

przy czym kolejne cyfry dopisujemy już po przecinku. Dzielenie<br />

kontynuujemy aż dostaniemy wymagana ↩<br />

dok̷ladność lub okres.<br />

Rozwia ↩<br />

zania przyk̷ladów (c) i (d) przedstawiamy w obliczeniach 3.<br />

(c)<br />

1 C,A<br />

ABC : 60<br />

6 0<br />

4 BC<br />

4 8 0<br />

3 C 0<br />

3 C 0<br />

0<br />

Obliczenia 3.<br />

(d)<br />

12400,13 (13)<br />

43201 : 3<br />

3<br />

13<br />

11<br />

22<br />

22<br />

0010<br />

3<br />

20<br />

14<br />

1<br />

Przy mnożeniu ignorujemy najpierw przecinki, a naste ↩<br />

pnie liczymy<br />

miejsca po przecinku w obu czynnikach i dodajemy wyniki.<br />

Suma ta wyznaczy pozycje ↩<br />

przecinka w iloczynie.<br />

(e)<br />

0,0 0 012<br />

× 0 ,0023<br />

102<br />

+ 30<br />

0, 0000 0 1 002


4.1. Szukanie NWD<br />

4. Algorytm Euklidesa<br />

4.1.1. Zauważmy, że znalezienie dzielników liczby 341 nie jest<br />

̷latwym zadaniem.<br />

704 2 341 11<br />

352 2 31 31<br />

176 2 1<br />

88 2<br />

44 2<br />

22 2<br />

11 11<br />

1<br />

Zatem NWD(704, 341) = 11 .<br />

4.1.2. Nie korzystaja ↩<br />

c z algorytmu Euklidesa otrzymujemy<br />

wyniki przedstawione w obliczeniach 4.<br />

128 2 1024 2<br />

64 2 562 2<br />

32 2 256 2<br />

16 2 128 2<br />

8 2 64 2<br />

4 2 32 2<br />

2 2 16 2<br />

1 8 2<br />

4 2<br />

2 2<br />

1<br />

Mamy wie ↩<br />

c NWD(1024, 128) = 2 · 2 · 2 · 2 · 2 · 2 · 2 = 128 .<br />

Obliczenia 4.<br />

Natomiast, jeśli sie ↩<br />

zdecydujemy wykorzystać algorytm Euklidesa,<br />

dostajemy 1024 = 8 · 128 + 0 . Ostatnia ↩<br />

niezerowa ↩<br />

reszta ↩<br />

jest<br />

wie ↩<br />

c tu r 0 = 128 . Zatem NWD(1024, 128) = 128 .<br />

4.1.3. (a) Mamy tutaj naste ↩<br />

puja ↩<br />

cy cia ↩<br />

g dzieleń:<br />

26 = 1 · 19 + 7 (1)


90 Cze ↩<br />

ść II – Rozwia ↩<br />

zania<br />

19 = 2 · 7 + 5 (2)<br />

7 = 1 · 5 + 2 (3)<br />

5 = 2 · 2 + 1 (4)<br />

2 = 2 · 1 + 0.<br />

Zatem NWD(26, 19) = 1 . Aby znaleźć odpowiednie liczby x oraz y ,<br />

zauważamy, że po przekszta̷lceniu równości (4) otrzymujemy<br />

1 = 1 · 5 + (−2) · 2. (5)<br />

Przekszta̷lcamy teraz (3) w celu ,,obliczenia” 2 dostaja ↩<br />

c 2 = 1 · 7 +<br />

(−1)·5 . Po podstawieniu do (5) mamy 1 = 1·5+(−2)(1·7+(−1)·5) ,<br />

czyli<br />

1 = 3 · 5 + (−2) · 7. (6)<br />

Podobnie ,,wyliczamy” 5 z (2) i wstawiamy do (6) otrzymuja ↩<br />

c<br />

1 = 3 · 19 + (−8) · 7. (7)<br />

Ostatecznie wykorzystujemy (1), by otrzymać 7 = 26 − 19 i po<br />

podstawieniu do (7) dostajemy ża ↩<br />

dana ↩<br />

kombinacje ↩<br />

1 = (−8) · 26 + 11 · 19.<br />

(b) Wykonujemy dzielenia:<br />

187 = 5 · 34 + 17<br />

34 = 2 · 17 + 0.<br />

Zatem NWD(187, 34) = 17 oraz 17 = 1 · 187 + (−5) · 34 .<br />

(c) Mamy tutaj:<br />

841 = 5 · 160 + 41<br />

160 = 3 · 41 + 37<br />

41 = 1 · 37 + 4<br />

37 = 9 · 4 + 1<br />

4 = 4 · 1 + 0


Algorytm Euklidesa 91<br />

Sta ↩<br />

d NWD(841, 160) = 1 . Aby wskazać odpowiednia ↩<br />

kombinacje ↩<br />

,<br />

obliczamy<br />

1 = 1 · 37 + (−9) · 4<br />

(d) Podobnie,<br />

1 = 10 · 37 + (−9) · 41<br />

1 = 10 · 160 + (−39) · 41<br />

1 = 205 · 160 + (−39) · 841.<br />

oraz<br />

2613 = 1 · 2171 + 442<br />

2171 = 4 · 442 + 403<br />

442 = 1 · 403 + 39<br />

403 = 10 · 39 + 13<br />

39 = 3 · 13 + 0<br />

13 = 1 · 403 + (−10) · 39<br />

13 = 11 · 403 + (−10) · 442<br />

13 = 11 · 2171 + (−54) · 442<br />

13 = 65 · 2171 + (−54) · 2613.<br />

4.1.4. Jeśli mamy już, że r k+1 ≤ 1 2 r k , to ponieważ r k+2 <<br />

r k+1 , mamy ża ↩<br />

dana ↩<br />

nierówność. Za̷lóżmy wie ↩<br />

c, że r k+1 > 1 2 r k .<br />

Wówczas przy naste ↩<br />

pnym dzieleniu mamy r k = q k+2 r k+1 + r k+2 .<br />

Ale tutaj musi być q k+2 = 1 , ponieważ r k < 2r k+1 . Zatem r k+2 =<br />

r k − r k+1 < r k − 1 2 r k = 1 2 r k , co należa̷lo pokazać.<br />

4.1.5. Jeśli wykorzystamy proponowany wariant algorytmu<br />

Euklidesa, wykonamy naste ↩<br />

puja ↩<br />

ce dzielenia:<br />

(a) 26 = 1 · 19 + 7 (b) 187 = 5 · 34 + 17<br />

19 = 2 · 7 + 5 34 = 2 · 17 + 0<br />

7 = 1 · 5 + 2<br />

5 = 2 · 2 + 1<br />

2 = 2 · 1 + 0<br />

(c) 841 = 5 · 160 + 41 (d) 2613 = 1 · 2171 + 442<br />

160 = 4 · 41 − 4 2171 = 5 · 442 − 39<br />

41 = 10 · 4 + 1 442 = 11 · 39 + 13<br />

4 = 4 · 1 + 0 39 = 3 · 13 + 0


92 Cze ↩<br />

ść II – Rozwia ↩<br />

zania<br />

4.1.6. Za̷lóżmy, że<br />

oraz<br />

r k−2 = q k r k−1 + r ′ k (1)<br />

r k−2 = q ′ kr k−1 − r ′′<br />

k. (2)<br />

Zauważmy, że q<br />

k ′ = q k + 1 . Podstawiaja ↩<br />

c to do (2) i porównuja ↩<br />

c<br />

z (1), otrzymujemy r<br />

k ′ = r k−1 − r<br />

k ′′ . Zatem (ponieważ wszystkie<br />

trzy liczby sa ↩<br />

dodatnie) liczby r<br />

k<br />

′ i r′′ k sa ↩<br />

równe po̷lowie r k−1 lub<br />

jedna z nich jest mniejsza niż po̷lowa r k−1 . Ponieważ wybieramy<br />

zawsze te ↩<br />

mniejsza ↩<br />

liczbe ↩<br />

, wie ↩<br />

c r k ≤ 1 2 r k−1 .<br />

4.1.7.<br />

(a) Ponieważ kolejne dzielenia sa ↩<br />

naste ↩<br />

puja ↩<br />

ce:<br />

10n + 9 = 9(n + 1) + n,<br />

n + 1 = 1 · n + 1,<br />

n = n · 1 + 0,<br />

wie ↩<br />

c NWD(10n + 9, n + 1) = 1 .<br />

(b) Kolejne dzielenia sa ↩<br />

naste ↩<br />

puja ↩<br />

ce:<br />

10n + 3 = 3(3n + 1) + n,<br />

3n + 1 = 3 · n + 1,<br />

n = n · 1 + 0.<br />

Zatem NWD(10n + 3, 3n + 1) = 1 .<br />

4.1.8. Zauważmy, że zachodza ↩<br />

równości<br />

nq = 10aq + bq<br />

= 10aq + a − a + bq<br />

= a(10q + 1) − (a − bq)<br />

= am − (a − bq).<br />

Za̷lóżmy, że m|n . Zatem m|nq , i z powyższej równości wynika, że<br />

m|a − bq . W druga ↩<br />

strone ↩<br />

, za̷lóżmy, że m|a − bq . Istnieje wie ↩<br />

c taka<br />

liczba ca̷lkowita k , że a − bq = mk . Sta ↩<br />

d<br />

n = 10a + b = 10(mk + bq) + b = 10mk + (10q + 1)b = m(10k + b).<br />

Zatem m|n .


Algorytm Euklidesa 93<br />

4.1.9. Z treści zadania wynika, że istnieja ↩<br />

liczby naturalne a<br />

oraz b takie, że x = 12a , y = 12b , przy czym NWD(a, b) = 1 .<br />

Ponieważ 12a + 12b = 96 , wie ↩<br />

c a + b = 8 . Uwzgle ↩<br />

dniaja ↩<br />

c warunek<br />

NWD(a, b) = 1 , otrzymujemy naste ↩<br />

puja ↩<br />

ce pary (1,7) , (3,5) , (5,3) ,<br />

(7,1) spe̷lniaja ↩<br />

ce równanie a+b = 8 . Zatem rozwia ↩<br />

zaniami naszego<br />

uk̷ladu sa ↩<br />

pary liczb (12,84) , (36,60) , (60,36) , (84,12) .<br />

4.1.10. x = kb , gdzie k ∈ Z jest taka, że NWD(<br />

k,<br />

a<br />

b<br />

)<br />

= 1 .<br />

4.1.11. Tak, gdyż NWD(a, b) = NWD(|a|, |b|) .<br />

4.1.12. Suma dwóch kolejnych liczb naturalnych jest równa<br />

s = n+(n+1) , a suma ich kwadratów jest równa t = n 2 +(n+1) 2 =<br />

2n 2 + 2n + 1 . Zauważmy, że<br />

2t − s 2 = 2(2n 2 + 2n + 1) − (4n 2 + 4n + 1) = 1.<br />

Z otrzymanej równości wynika, że NWD(s, t) = 1 .<br />

4.2. Równania liniowe<br />

4.2.1. Obliczamy kolejno najwie ↩<br />

ksze wspólne dzielniki wspó̷lczynników<br />

równań, a naste ↩<br />

pnie patrzymy, czy sa ↩<br />

one dzielnikami<br />

wyrazów wolnych.<br />

(a) NWD(12, 18) = 6 oraz 6|36 , wie ↩<br />

c równanie ma rozwia ↩<br />

zanie.<br />

(b) NWD(65, 39) = 13 , ale 13̸ | 16 , zatem to równanie nie ma<br />

rozwia ↩<br />

zań w liczbach ca̷lkowitych.<br />

(c) NWD(2613, 2171) = 13 , co jest dzielnikiem 39. Zatem to<br />

równanie ma rozwia ↩<br />

zanie.<br />

(d) NWD(12, 15) = 3 , co jest dzielnikiem 333, wie ↩<br />

c to równanie<br />

także ma rozwia ↩<br />

zanie.<br />

(e) NWD(119, 105) = 7 oraz 7|28 , wie ↩<br />

c i to równanie ma<br />

rozwia ↩<br />

zanie.<br />

(f) NWD(28, 35) = 7 , ale 7 nie jest dzielnikiem liczby 347.<br />

Zatem to równanie nie ma rozwia ↩<br />

zań ca̷lkowitych.<br />

4.2.2. Ponieważ 2|c wtedy i tylko wtedy, gdy c jest liczba ↩<br />

parzysta ↩<br />

, wie ↩<br />

c równanie ax + by = c ma rozwia ↩<br />

zanie wtedy i tylko<br />

wtedy, gdy c jest liczba ↩<br />

parzysta ↩<br />

.


94 Cze ↩<br />

ść II – Rozwia ↩<br />

zania<br />

4.2.3. Jeżeli liczby a i b sa ↩<br />

wzgle ↩<br />

dnie pierwsze, to oznacza<br />

to, iż NWD(a, b) = 1 . Ponieważ 1 jest dzielnikiem każdej liczby<br />

ca̷lkowitej, wie ↩<br />

c równanie ax + by = c ma rozwia ↩<br />

zanie dla dowolnego<br />

c ∈ Z . Zatem, ponieważ zbiór A możemy zapisać jako<br />

{ax + by : x,y ∈ Z} , wie ↩<br />

c A = Z .<br />

4.2.4. Niech a ∈ A , a ≠ 0 . Z określenia zbioru A wynika, że<br />

0 = a − a ∈ A oraz −a = 0 − a ∈ A . Ponieważ jedna z liczb a , −a<br />

należa ↩<br />

cych do A jest liczba ↩<br />

naturalna ↩<br />

, wie ↩<br />

c zbiór A zawiera liczby<br />

naturalne. Niech m be ↩<br />

dzie najmniejsza ↩<br />

liczba ↩<br />

naturalna ↩<br />

zbioru A<br />

i niech x ∈ A . Dziela ↩<br />

c x przez m mamy x = qm + r , gdzie<br />

0 ≤ r < m . Wtedy r = x − qm ∈ A . Gdyby r > 0 , to r by̷loby<br />

liczba ↩<br />

naturalna ↩<br />

mniejsza ↩<br />

od m należa ↩<br />

ca ↩<br />

do A . Zatem r = 0 ,<br />

czyli m|x .<br />

4.2.5. Wystarczy napisać równanie, które nie ma rozwia ↩<br />

zania<br />

w liczbach ca̷lkowitych. Na przyk̷lad 2x + 4y = 3 .<br />

4.3. Rozwia ↩<br />

zywanie równań liniowych<br />

4.3.1. Rozwia ↩<br />

żemy tylko równania (a), (c) i (e), a w pozosta̷lych<br />

przyk̷ladach ograniczymy sie ↩<br />

tylko do podania odpowiedzi.<br />

Zauważmy, że każde równanie może mieć kilka różnych przedstawień<br />

rozwia ↩<br />

zania.<br />

(a) Wykonujemy dzielenie 18 = 1 · 12 + 6 i podstawiamy do<br />

naszego równania otrzymuja ↩<br />

c<br />

12(x + y) + 6y = 36.<br />

Oznaczamy teraz x 1 = x + y , y 1 = y i otrzymujemy<br />

równanie 12x 1 + 6y 1 = 36 , które możemy obustronnie<br />

podzielić przez 6 i otrzymać<br />

2x 1 + y 1 = 6.<br />

Podstawiamy teraz x 1 = t i obliczamy kolejno y 1 = 6 −<br />

2t , y = 6 − 2t oraz x = −6 + 3t .<br />

(b) x = −2 + 3t , y = 4 − 5t .<br />

(c) Wykonujemy najpierw dzielenie 2613 = 1 · 2171 + 442 .<br />

Naste ↩<br />

pnie podstawiamy x 1 = x + y , y 1 = x i otrzymujemy<br />

2171x 1 + 442y 1 = 39.


Algorytm Euklidesa 95<br />

Po podzieleniu 2171 = 4 · 442 + 403 i podstawieniu x 2 =<br />

4x 1 + y 1 , y 2 = x 1 dostajemy<br />

442x 2 + 403y 2 = 39.<br />

Teraz podstawimy x 3 = x 2 + y 2 oraz y 3 = x 2 i otrzymamy<br />

403x 3 + 39y 3 = 39.<br />

Wreszcie podzielimy 403 = 10·39+13 , podstawimy x 4 =<br />

10x 3 + y 3 , y 4 = x 3 i dostaniemy<br />

39x 4 + 13y 4 = 39.<br />

Ostatnie równanie podzielimy obustronnie przez 13 i za x 4<br />

przyjmiemy parametr t . Wycofuja ↩<br />

c sie ↩<br />

dostajemy kolejno<br />

x 4 = t , y 4 = 3 − 3t ,<br />

x 3 = 3 − 3t , y 3 = −30 + 31t ,<br />

x 2 = −30 + 31t , y 2 = 33 − 34t ,<br />

x 1 = 33 − 34t , y 1 = −162 + 167t ,<br />

x = −162 + 167t , y = 195 − 201t .<br />

(d) x = −111 + 5t , y = 111 − 4t .<br />

(e) Dzielimy obustronnie nasze równanie przez 4, otrzymuja ↩<br />

c<br />

rówoważne równanie x + y = 2 . Jego rozwia ↩<br />

zaniami sa ↩<br />

x = t , y = 2 − t .<br />

(f) x = −28 + 15t , y = 32 − 17t .<br />

4.3.2. Na pocza ↩<br />

tku zauważmy, że nasze równanie ma zawsze<br />

rozwia ↩<br />

zanie, ponieważ NWD(12, 15) = 3 . Poste ↩<br />

pujemy wie ↩<br />

c zgodnie<br />

z naszym algorytmem, tj. podstawiamy x 1 = x + y oraz y 1 = y<br />

i zapisujemy 12x 1 +3y 1 = 3(7u−22) . Otrzymane równanie dzielimy<br />

obustronnie przez 3 i za x 1 przyjmujemy parametr t . Zatem y 1 =<br />

7u − 4t − 22 i ostatecznie y = 7u − 4t − 22 oraz x = −7u + 5t + 22 .<br />

4.3.3.<br />

(a) Zgodnie ze wskazówka ↩<br />

rozwia ↩<br />

zujemy równanie 3t + 7z =<br />

11 , otrzymuja ↩<br />

c t = −22 + 7u oraz z = 11 − 3u .<br />

Mamy zatem obliczona ↩<br />

niewiadoma ↩<br />

z . Aby obliczyć x<br />

i y , rozwia ↩<br />

zujemy równanie 12x + 15y = 3(7u − 22) ,


96 Cze ↩<br />

ść II – Rozwia ↩<br />

zania<br />

otrzymuja ↩<br />

c x = −7u + 5t + 22 oraz y = 7u − 4t − 22<br />

(patrz poprzednie zadanie).<br />

(b) Zapiszmy nasze równanie w postaci 6x + 10y = 4 − 12z .<br />

Zatem NWD(6, 10) = 2 musi dzielić 4−12z . Otrzymujemy<br />

wie ↩<br />

c równanie 2t + 12z = 4 , którego rozwia ↩<br />

zaniami sa ↩<br />

z = u oraz t = 2−6u . Aby obliczyć x i y rozwia ↩<br />

zujemy<br />

równanie 6x + 10y = 2(2 − 6u) i otrzymujemy<br />

x = 4 − 12u − 5t oraz y = −2 + 6u + 3t.<br />

4.3.4. Podstawiamy x 1 = x + y i y 1 = x , otrzymuja ↩<br />

c<br />

równanie<br />

18x 1 + 6y 1 = 19.<br />

W dalszym cia ↩<br />

gu podstawiamy x 2 = 3x 1 +y 1 i dostajemy równanie<br />

6x 2 = 19 , które nie ma rozwia ↩<br />

zania w liczbach ca̷lkowitych, ponieważ<br />

6̸ | 19 .<br />

Zauważmy, że jeśli równanie nie ma rozwia ↩<br />

zania w liczbach<br />

ca̷lkowitych, to algorytm rozwia ↩<br />

zywania zawsze ,,zacina sie ↩<br />

” w tym<br />

miejscu. Jest tak, ponieważ wspó̷lczynnik przy niewiadomej jest<br />

równy najwie ↩<br />

kszemu wspólnemu dzielnikowi wspó̷lczynników równania<br />

wyjściowego, a wyraz wolny pozostaje nie zmieniony.<br />

4.3.5. Niech (p,q) bedzie dowolnym rozwia ↩<br />

zaniem równania<br />

(1). Wtedy zachodza ↩<br />

równości ax 0 + by 0 = c oraz ap + bq = c ,<br />

sta ↩<br />

d<br />

a(p − x 0 ) = −b(q − y 0 ). (*)<br />

Niech d = NWD(a, b) i niech a = a 1 d , b = b 1 d . Wówczas z (*)<br />

mamy<br />

a 1 (p − x 0 ) = −b 1 (q − y 0 ), (**)<br />

gdzie NWD(a 1 , b 1 ) = 1 . Sta ↩<br />

d wynika, że a 1 |q − y 0 i b 1 |p − x 0 .<br />

Istnieja ↩<br />

wie ↩<br />

c liczby ca̷lkowite t oraz s takie, że q − y 0 = a 1 s oraz<br />

p − x 0 = b 1 t . Podstawiaja ↩<br />

c do (**) ̷latwo zauważymy, że s =<br />

b<br />

−t . Zatem p = x 0 + b 1 t , q = y 0 − a 1 t , gdzie b 1 =<br />

NWD(a,b) ,<br />

a<br />

a 1 =<br />

NWD(a,b) . Na zakończenie zauważmy, że dla dowolnej liczby<br />

ca̷lkowitej t , para (p,q) , gdzie p = x 0 + b 1 t , q = y 0 − a 1 t jest<br />

rozwia ↩<br />

zaniem równania (1).


Algorytm Euklidesa 97<br />

4.3.6. Wystarczy znaleźć jedno rozwia ↩<br />

zanie (x 0 ,y 0 ) tego<br />

równania i zastosować wzory z 4.3.5. Dla znalezienia rozwia ↩<br />

zania<br />

(x 0 ,y 0 ) zastosujemy algorytm Euklidesa.<br />

852 = 4 · 192 + 84<br />

192 = 2 · 84 + 24<br />

84 = 3 · 24 + 12<br />

24 = 2 · 12 + 0.<br />

Sta ↩<br />

d NWD(852, 192) = 12 = 84 − 3 · 24 = 84 − 3 · (192 − 2 · 84) =<br />

7 · 84 − 3 · 192 = 7(852 − 4 · 192) − 3 · 192 = 7 · 852 − 31 · 192 , a sta ↩<br />

d<br />

852·(7·2)+192(−31·2) = 24. Otrzymana równość oznacza, że para<br />

(14,−62) jest rozwia ↩<br />

zaniem równania 852x + 192y = 24 . Zatem<br />

wszystkie rozwia ↩<br />

zania dane sa ↩<br />

wzorami x = 14+16t , y = −62−71t ,<br />

gdzie t ∈ Z .


5. Kongruencje<br />

5.1. Podstawowe w̷lasności kongruencji<br />

5.1.1. Ponieważ liczba a − a = 0 jest podzielna przez każda ↩<br />

liczbe ↩<br />

naturalna ↩<br />

m , wie ↩<br />

c a ≡ a(mod m ) . Jeżeli m|a − b , to<br />

także m|b−a , co oznacza, że z kongruencji a ≡ b(mod m ) wynika<br />

kongruencja b ≡ a(mod m ) . Niech a ≡ b(mod m ) oraz b ≡<br />

c(mod m ) . Wtedy m|a − b i m|b − c . Sta ↩<br />

d m|(a − b) + (b − c) ,<br />

czyli m|a − c . Zatem, ostatecznie, a ≡ c(mod m ) .<br />

5.1.2. Za̷lóżmy, że a ≡ b(mod m ) . Niech a = mx + r 1 oraz<br />

b = my + r 2 , przy czym 0 ≤ r 1 < m oraz 0 ≤ r 2 < m . Z tego, że<br />

m|a − b oraz a − b = m(x − y) + (r 1 − r 2 ) wynika, że m|r 1 − r 2 .<br />

Ponieważ r 1 ,r 2 ∈ {0,1,2,. . . ,m − 1} , wie ↩<br />

c r 1 = r 2 .<br />

Za̷lóżmy teraz, że liczby a i b przy dzieleniu przez m maja ↩<br />

takie same reszty, tzn. a = mq 1 + r i b = mq 2 + r . Wtedy<br />

a − b = m(q 1 − q 2 ) , czyli a ≡ b(mod m ) .<br />

Niech teraz a be ↩<br />

dzie dowolna ↩<br />

liczba ↩<br />

ca̷lkowita ↩<br />

. Możemy<br />

napisać a = mq + r , gdzie r ∈ {0,1,2,. . . ,m − 1} . Zatem m|a − r ,<br />

czyli a ≡ r(mod m ) . Sta ↩<br />

d mamy, że klasa abstrakcji elementu a<br />

ma reprezentanta w zbiorze {0,1,2,. . . ,m − 1} . Z pierwszej cze ↩<br />

ści<br />

rozwia ↩<br />

zania wynika, że taki reprezentant jest dok̷ladnie jeden.<br />

5.1.3. Z kongruencji a ≡ b(mod m ) wobec 5.1.2 wynika, że<br />

a = mq 1 + r oraz b = mq 2 + r , sta ↩<br />

d a − b = m(q 1 − q 2 ) , czyli<br />

a = b + mt , gdzie t = q 1 − q 2 . Odwrotnie, z równości a = b + mt<br />

wynika, że a − b = mt , czyli a ≡ b(mod m ) .<br />

5.1.4. Jeżeli a 1 ≡ b 1 (mod m ) , a 2 ≡ b 2 (mod m ) , to a 1 =<br />

b 1 + mt 1 , a 2 = b 2 + mt 2 . Zatem a 1 + a 2 = b 1 + b 2 + m(t 1 + t 2 ) ,<br />

a 1 − a 2 = b 1 − b 2 + m(t 1 − t 2 ) oraz<br />

a 1 · a 2 = (b 1 + mt 1 )(b 2 + mt 2 ) = b 1 b 2 + m(b 1 t 2 + b 2 t 1 + mt 1 t 2 ).<br />

Sta ↩<br />

d wynika, że (a 1 + a 2 ) ≡ (b 1 + b 2 )(mod m ) , (a 1 − a 2 ) ≡ (b 1 −<br />

b 2 )(mod m ) oraz (a 1 · a 2 ) ≡ (b 1 · b 2 )(mod m ) .<br />

5.1.5.<br />

(a) Kongruencja ac ≡ bc(mod mc ) oznacza, iż istnieje taka<br />

liczba ca̷lkowita d , że ac − bc = mcd , ska ↩<br />

d a − b = md ,<br />

co oznacza a ≡ b(mod m ) .


Kongruencje 99<br />

(b) Ponieważ m|(a − b)c oraz NWD(m, c) = 1 , wie ↩<br />

c m|a − b<br />

(zobacz 1.8).<br />

(c) Niech NWW(s, t) = ss 1 = tt 1 . Mamy a ss 1<br />

≡ 1(mod m )<br />

oraz a tt 1<br />

≡ 1(mod n ) , czyli<br />

m|a NWW(s,t) − 1 oraz n|a NWW(s,t) − 1.<br />

Sta ↩<br />

d, wobec 1.7.4 oraz 1.7.6, mamy mn|a NWW(s,t) − 1 .<br />

(d) Jeżeli m|a − b oraz n|a − b , to NWW(m, n)|a − b (wynika<br />

to z 1.7.6).<br />

5.1.6. Należy pokazać, że 2 55 ≡ −1(mod 11 ) . Ponieważ<br />

2 5 = 32 ≡ −1(mod 11 ) , wie ↩<br />

c 2 55 = ( 2 5) 11<br />

≡ (−1) 11 ≡<br />

(−1)(mod 11 ) . Ostatnia ↩<br />

cyfra ↩<br />

liczby 2 55 + 1 w systemie o podstawie<br />

11 jest 0.<br />

5.1.7. Ponieważ 78 = 6·13 oraz NWD(6, 13) = 1 , wie ↩<br />

c wystarczy<br />

pokazać, że 6|53 103 +103 53 oraz 13|53 103 +103 53 . Bezpośrednio<br />

sprawdzamy, że 53 ≡ −1(mod 6 ) oraz 103 ≡ 1(mod 6 ) . Sta ↩<br />

d<br />

53 103 ≡ −1(mod 6 ) oraz 103 53 ≡ 1(mod 6 ) . Dodaja ↩<br />

c stronami<br />

otrzymane kongruencje, mamy 53 103 +103 53 ≡ −1+1 ≡ 0(mod 6 ) .<br />

Podobnie stwierdzamy, że 53 103 + 103 53 ≡ 0(mod 13 ) .<br />

5.1.8.<br />

(a) Jeżeli n 0 jest ostatnia ↩<br />

cyfra ↩<br />

liczby n , to zachodzi kongruencja<br />

n ≡ n 0 (mod 10 ) . Ponieważ 2 5 ≡ 2(mod 10 ) ,<br />

wie ↩<br />

c 2 5·5·5 ≡ 2(mod 10 ) , natomiast 2 8 = 2 3 · 2 5 ≡<br />

6(mod 10 ) . Sta ↩<br />

d 2 1000 = ( 2 5·5·5) 8<br />

≡ 2 8 ≡ 6(mod 10 ) .<br />

Zatem ostatnia ↩<br />

cyfra ↩<br />

liczby 2 1000 jest 6.<br />

(b) F 0 = 3 , F 1 = 5 . Jeżeli n ≥ 2 , to<br />

2 2n =<br />

(<br />

2 22) 2 n−2 ≡ 6(mod 10 ).<br />

Zatem dla n ≥ 2 ostatnia ↩<br />

cyfra ↩<br />

liczby F n jest 7.<br />

5.1.9. Niech a − 1 , a , a + 1 be ↩<br />

da ↩<br />

trzema kolejnymi liczbami<br />

naturalnymi. Wówczas (a − 1) 3 + a 3 + (a + 1) 3 = 3a 3 + 6a =<br />

3(a 3 + 2a) . Ponieważ 18 = 3 · 6 , wie ↩<br />

c wystarczy zbadać kiedy<br />

a 3 + 2a ≡ 0(mod 6 ) . Zauważmy, że jeżeli a ≡ 0(mod 6 ) lub


100 Cze ↩<br />

ść II – Rozwia ↩<br />

zania<br />

a ≡ 2(mod 6 ) lub a ≡ 4(mod 6 ) , to a 3 + 2a ≡ 0(mod 6 ) .<br />

Odwrotnie, jeżeli 6|a 3 + 2a , to 6|a(a 2 + 2) . Gdyby liczba a by̷la<br />

nieparzysta, iloczyn a(a 2 +2) by̷lby liczba ↩<br />

nieparzysta ↩<br />

. Zatem suma<br />

sześcianów trzech kolejnych liczb naturalnych jest podzielna przez<br />

18 , gdy druga z tych liczb jest parzysta.<br />

5.1.10. Ponieważ 3 ≡ −4(mod 7 ) , wie ↩<br />

c 3 2 ≡ 2(mod 7 ) .<br />

Zatem dla dowolnej liczby naturalnej n mamy (3 2 ) n ≡ 2 n (mod 7 ) .<br />

Sta ↩<br />

d 3 · 3 2n ≡ −4 · 2 n (mod 7 ) , czyli 3 2n+1 + 2 n+2 ≡ 0(mod 7 ) .<br />

5.1.11.<br />

(a) Zauważmy, że ( )<br />

p<br />

k = p· (p−1)!<br />

k!(p−k)!<br />

jest liczba ↩<br />

naturalna ↩<br />

, czyli<br />

k!(p−k)!|p(p−1)! . Ponieważ NWD(k!(p − k)!, p) = 1 , wie ↩<br />

c<br />

k!(p − k)!|(p − 1)! (zobacz 1.8).<br />

(p−1)!<br />

k!(p−k)!<br />

jest liczba ↩<br />

ca̷lkowita ↩<br />

. Zatem p| ( )<br />

p<br />

k .<br />

(b) Z w̷lasności symbolu Newtona wynika, że<br />

( ) p<br />

n<br />

sta ↩<br />

d<br />

k<br />

( ) p<br />

n<br />

k<br />

k<br />

= pn k<br />

Sta ↩<br />

d wynika, że liczba<br />

( p n − 1<br />

k − 1<br />

)<br />

,<br />

( p<br />

= p n n )<br />

− 1<br />

.<br />

k − 1<br />

Zauważmy, że prawa strona ostatniej równości dzieli sie ↩<br />

przez p n , zatem i lewa strona dzieli sie ↩<br />

przez p n . Jeżeli<br />

1 ≤ k ≤ p n − 1 , to p n̸ | k , sta ↩<br />

d p| ( )<br />

p n<br />

k .<br />

(c) Wynika z wzoru dwumianowego Newtona oraz cze ↩<br />

ści (b).<br />

5.1.12. Liczba naturalna n jest postaci 3k + 1 , 3k + 2<br />

lub 3k + 3 , gdzie k ∈ N 0 . Ponieważ 2 3 ≡ 1(mod 7 ) , wie ↩<br />

c<br />

2 3k ≡ 1(mod 7 ) . Sta ↩<br />

d 2 3k+1 ≡ 2(mod 7 ) , 2 3k+2 ≡ 4(mod 7 )<br />

i 2 3k+3 ≡ 1(mod 7 ) .<br />

Ostatnia ↩<br />

cyfra ↩<br />

liczby 2 n + 1 w systemie o podstawie 7 może<br />

wie ↩<br />

c być 2, 3 lub 5.<br />

5.2. Kongruencje a wielomiany<br />

5.2.1. Rozważmy wielomian W (x) = 1 + x + 2x 2 + . . . + nx n .<br />

Ponieważ A = W (8) , B = W (3) oraz 8 ≡ 3(mod 5 ) , wie ↩<br />

c A ≡<br />

B(mod 5 ) .


Kongruencje 101<br />

5.2.2. Z za̷lożenia mamy naste ↩<br />

puja ↩<br />

ce kongruencje: W (2) ≡<br />

0(mod 5 ) oraz W (5) ≡ 0(mod 2 ) . Wiemy, że 5 ≡ −3(mod 2 )<br />

oraz 2 ≡ −3(mod 5 ) , wie ↩<br />

c W (5) ≡ W (−3)(mod 2 ) oraz W (2) ≡<br />

W (−3)(mod 5 ) . Sta ↩<br />

d i z za̷lożenia W (−3) ≡ 0(mod 2 ) oraz<br />

W (−3) ≡ 0(mod 5 ) . Z zadania 5.1.5(d) wynika, że W (−3) ≡<br />

0(mod 10 ) . Z kongruencji 7 ≡ −3(mod 10 ) wynika natomiast, że<br />

W (7) ≡ W (−3)(mod 10 ) . Ponieważ W (−3) ≡ 0(mod 10 ) , wie ↩<br />

c<br />

W (7) ≡ 0(mod 10 ) .<br />

5.2.3. Ponieważ n ≡ n 0 + n 1 + . . . + n s (mod 3 ) , wie ↩<br />

c jeżeli<br />

suma n 0 + n 1 + . . . + n s dzieli sie ↩<br />

przez 3 , to n dzieli sie ↩<br />

przez 3 .<br />

5.2.4. Wychodzimy z kongruencji 10 ≡ 1(mod 9 ) i rozumujemy<br />

analogicznie jak w 5.2.3.<br />

5.2.5. Rozumuja ↩<br />

c analogicznie jak w 5.2.3, stwierdzamy, że<br />

jeżeli n = n 0 + n 1 10 + · · · + n s 10 s , to n ≡ n 0 − n 1 + n 2 − · · · +<br />

(−1) s n s (mod 11 ) . Zatem, jeżeli 11|n 0 − n 1 + n 2 − · · · + (−1) s n s ,<br />

to 11|n .<br />

5.2.6. Za̷lóżmy, że istnieje wielomian W spe̷lniaja ↩<br />

cy warunki<br />

zadania. Ponieważ a ≡ b(mod (a − b) ) , wie ↩<br />

c<br />

W (a) ≡ W (b)(mod (a − b) ),<br />

sta ↩<br />

d a−b|b−c . Podobnie stwierdzamy, że b−c|c−a oraz c−a|a−b .<br />

Sta ↩<br />

d |a − b| ≤ |b − c| , |b − c| ≤ |c − a| oraz |c − a| ≤ |a − b| , a sta ↩<br />

d<br />

|a − b| = |b − c| = |c − a|.<br />

Liczby a − b i b − c sa ↩<br />

wie ↩<br />

c równe lub przeciwne. Gdyby by̷ly<br />

przeciwne, czyli gdyby a − b = −(b − c) , to a = c , co przeczy<br />

za̷lożeniu. Zatem a − b = b − c . Podobnie stwierdzamy, że c − a =<br />

b − c . Wobec tego 3(b − c) = (a − b) + (b − c) + (c − a) = 0 . Sta ↩<br />

d<br />

b = c , co znowu jest sprzeczne z za̷lożeniem. Zatem nie istnieje<br />

wielomian spe̷lniaja ↩<br />

cy warunki zadania.<br />

5.2.7. Przypuśćmy, że istnieje taki wielomian W . Wtedy<br />

W (1) = p jest liczba ↩<br />

pierwsza ↩<br />

. Niech b k = 1 + p · k , gdzie k ∈ N .<br />

Ponieważ b k ≡ 1(mod p ) , wie ↩<br />

c W (b k ) ≡ W (1) = p ≡ 0(mod p ) .<br />

Wobec tego, że p|W (b k ) oraz W (b k ) jest liczba ↩<br />

pierwsza ↩<br />

, W (b k ) =<br />

p . Oznacza to, że wielomian W przyjmuje wartość p nieskończenie<br />

wiele razy, co jest niemożliwe.


102 Cze ↩<br />

ść II – Rozwia ↩<br />

zania<br />

5.2.8. Niech wybrana ↩<br />

liczba ↩<br />

be ↩<br />

dzie n = (n t n t−1 . . . n 0 ) 10<br />

i niech b = (b s b s−1 . . . b 0 ) 10 = n − (n t + n t−1 + . . . + n 0 ) . Wtedy<br />

9|b s + b s−1 + · · · + b 0 .<br />

5.3. Kongruencje a równania<br />

5.3.1. Przypuśćmy, że istnieje liczba ca̷lkowita c taka, że<br />

W (c) = 0 . Wówczas istnieje r ∈ {0,1} takie, że W (r) ≡<br />

0(mod 2 ) , czyli W (0) ≡ 0(mod 2 ) lub W (1) ≡ 0(mod 2 ) .<br />

Ponieważ W (0) oraz W (1) sa ↩<br />

nieparzyste, wie ↩<br />

c otrzymaliśmy<br />

sprzeczność.<br />

5.3.2. Niech W (x) = 100x 6 − 80x 5 + 36x 4 + 12x 3 + x 2 + 1001 .<br />

Przypuśćmy, że istnieje liczba ca̷lkowita c taka, że W (c) = 0 .<br />

Wówczas istnieje r ∈ {0,1,2,3} takie, że W (r) ≡ 0(mod 4 ) .<br />

Ponieważ 4 dzieli każda ↩<br />

z liczb 100, −80 , 36 oraz 12, wie ↩<br />

c<br />

W (r) ≡ r 2 + 1 ≡ 0(mod 4 ) . Otrzymaliśmy sprzeczność, bo<br />

W (0) ≡ 1(mod 4 ) , W (1) ≡ 2(mod 4 ) , W (2) ≡ 1(mod 4 ) ,<br />

a W (3) ≡ 2(mod 4 ) .<br />

5.3.3. Przypuśćmy, że para liczb ca̷lkowitych (a,b) jest<br />

rozwia ↩<br />

zaniem równania 2x 2 − 215y 2 = 1, czyli że zachodzi równość<br />

2a 2 − 215b 2 =1. Sta ↩<br />

d (2a) 2 = 430b 2 + 2 . Z otrzymanej równości<br />

wynika, że dla dowolnej liczby naturalnej m zachodzi kongruencja<br />

(2a) 2 ≡ 430b 2 + 2(mod m ) . Dla dowolnego b mamy<br />

430b 2 + 2 ≡ 2(mod 10 ) . Ponieważ 2a ≡ 0(mod 10 ) lub 2a ≡<br />

±2(mod 10 ) , lub 2a ≡ ±4(mod 10 ) , wie ↩<br />

c (2a) 2 ≡ 0(mod 10 )<br />

lub (2a) 2 ≡ 4(mod 10 ) , lub (2a) 2 ≡ 6(mod 10 ) . Sta ↩<br />

d (2a) 2 ≢<br />

430b 2 + 2(mod10) , co oznacza, że równanie 2x 2 − 215y 2 = 1 nie ma<br />

rozwia ↩<br />

zań ca̷lkowitych.<br />

5.3.4. Niech W be ↩<br />

dzie wielomianem o wspó̷lczynnikach<br />

ca̷lkowitych, takim że W (x) = (x 2 − 12x + 11)g(x) + 990x − 889 .<br />

Przypuśćmy, że wielomian ten ma pierwiastek ca̷lkowity a , czyli<br />

że W (a) = 0 . Istnieje wie ↩<br />

c r ∈ {0,1,2,3,4,5} takie, że W (r) ≡<br />

0(mod 6 ) . ̷Latwo sprawdzimy, że W (1) = (1 − 12 + 11)g(1) +<br />

990 − 889 ≡ 1(mod 6 ) , W (2) = (4 − 24 + 11)g(2) + 1980 − 889 =<br />

(−9)g(2) + 1091 ≢ 0(mod6) (gdyż 3̸ | 1091 ). Podobnie W (3) ≢<br />

0(mod6) , W (4) ≢ 0(mod6) , W (5) ≢ 0(mod6) i W (0) ≢ 0(mod6) .<br />

Otrzymaliśmy wie ↩<br />

c sprzeczność.


5.4. Ma̷le Twierdzenie Fermata<br />

Kongruencje 103<br />

5.4.1. Dla n = 1 fakt jest oczywisty.<br />

Za̷lóżmy, że twierdzenie jest prawdziwe dla pewnego n > 1 .<br />

Wówczas n p ≡ n(mod p ) , a sta ↩<br />

d n p +1 ≡ n+1(mod p ) . Z 5.1.11<br />

otrzymujemy (n + 1) p ≡ n p + 1(mod p ) . Sta ↩<br />

d, wobec za̷lożenia<br />

indukcyjnego, mamy (n + 1) p ≡ n + 1(mod p ) . To znaczy, że<br />

twierdzenie jest prawdziwe dla n + 1 .<br />

5.4.2.<br />

(a) Wystarczy pokazać, że 2|n 5 −n , 3|n 5 −n i 5|n 5 −n , czyli<br />

że n 5 ≡ n(mod 2 ) , n 5 ≡ n(mod 3 ) i n 5 ≡ n(mod 5 ) .<br />

Z wniosku z MTF wynika, że n 2 ≡ n(mod 2 ) , sta ↩<br />

d n 4 ≡<br />

n 2 ≡ n(mod 2 ) , a sta ↩<br />

d n 5 ≡ n 2 ≡ n(mod 2 ) . Pozosta̷le<br />

kongruencje też wynikaja ↩<br />

z wniosku z MTF.<br />

(b) Z wniosku z MTF mamy n 3 ≡ n(mod 3 ) , sta ↩<br />

d n 7 ≡<br />

n 5 (mod 3 ) , a sta ↩<br />

d n 7 −n 3 ≡ n 5 −n(mod 3 ) , czyli 3|n 7 −<br />

n 5 − n 3 + n . Podobnie z kongruencji n 5 ≡ n(mod 5 )<br />

otrzymujemy n 7 ≡ n 3 (mod 5 ) , a sta ↩<br />

d n 7 − n 5 ≡ n 3 −<br />

n(mod 5 ) , czyli 5|n 7 − n 5 − n 3 + n .<br />

5.4.3. Jeżeli p > 5 , to p̸ | 10 , wie ↩<br />

c z MTF mamy 10 p−1 ≡<br />

1(mod p ) , co oznacza p|10 p−1 − 1 . Zauważmy, że 10 p−1 − 1 =<br />

99 . . . 9 = 9 · n p . Ponieważ p|9 · n p oraz p̸ | 9 , wie ↩<br />

c p|n p .<br />

5.4.4. Z wniosku z MTF wynika, że a p 1 ≡ a 1(mod p ) ,<br />

a p 2 ≡ a 2(mod p ) , . . . a p k ≡ a k(mod p ) . Dodaja ↩<br />

c stronami te<br />

kongruencje mamy a p 1 + ap 2 + . . . + ap k ≡ a 1 + a 2 + . . . + a k (mod p ) .<br />

Z 5.1.2 wynika, że suma a p 1 + ap 2 + . . . + ap k<br />

dzieli sie ↩<br />

przez p wtedy<br />

i tylko wtedy, gdy suma a 1 + a 2 + . . . + a k dzieli sie ↩<br />

przez p .<br />

5.4.5. Dodaja ↩<br />

c stronami kongruencje a 2 ≡ a(mod 2 ) , b 2 ≡<br />

b(mod 2 ) , c 2 ≡ c(mod 2 ) i d 2 ≡ d(mod 2 ) mamy a 2 + b 2 +<br />

c 2 + d 2 ≡ a + b + c + d(mod 2 ) . Sta ↩<br />

d oraz z za̷lożenia wynika, że<br />

2(a 2 + b 2 ) ≡ a + b + c + d(mod 2 ) , czyli 2|a + b + c + d . Ponieważ<br />

liczba a + b + c + d jest wie ↩<br />

ksza od 2 , wie ↩<br />

c nie jest ona liczba ↩<br />

pierwsza ↩<br />

.<br />

5.4.6. Jeżeli p = 2 , to każda liczba postaci 2 2k − 2k , gdzie<br />

k ∈ N , dzieli sie ↩<br />

przez 2 . Jeżeli p > 2 , to z MTF wynika, że


104 Cze ↩<br />

ść II – Rozwia ↩<br />

zania<br />

2 p−1 ≡ 1(mod p ) , sta ↩<br />

d 2 (p−1)m ≡ 1(mod p ) dla m ∈ N . Biora ↩<br />

c<br />

za m liczby postaci kp − 1 , gdzie k ∈ N , mamy<br />

2 (p−1)(kp−1) ≡ 1 ≡ (kp − 1)(p − 1)(mod p ).<br />

Sta ↩<br />

d p|2 (p−1)(kp−1) − (p − 1)(kp − 1) .<br />

5.4.7. Ponieważ 30 = 2 · 3 · 5 , wie ↩<br />

c wystarczy pokazać, że<br />

K ≡ 0(mod 2 ) , K ≡ 0(mod 3 ) oraz K ≡ 0(mod 5 ) . Jeżeli<br />

a ≡ 0(mod 2 ) lub b ≡ 0(mod 2 ) , to K ≡ 0(mod 2 ) . Jeżeli<br />

a ≡ 1(mod 2 ) i b ≡ 1(mod 2 ) , to a 2 − b 2 ≡ 0(mod 2 ) , a wie ↩<br />

c<br />

K ≡ 0(mod 2 ) . Jeżeli a ≡ 0(mod 3 ) lub b ≡ 0(mod 3 ) , to<br />

K ≡ 0(mod 3 ) . Jeżeli 3̸ | a i 3̸ | b , to a 2 ≡ 1(mod 3 ) i b 2 ≡<br />

1(mod 3 ) , wie ↩<br />

c a 2 −b 2 ≡ 0(mod 3 ) , czyli K ≡ 0(mod 3 ) . Jeżeli<br />

a ≡ 0(mod 5 ) lub b ≡ 0(mod 5 ) , to K ≡ 0(mod 5 ) . Jeżeli<br />

5̸ | a i 5̸ | b , to a 4 ≡ 1(mod 5 ) i b 4 ≡ 1(mod 5 ) , wie ↩<br />

c a 4 − b 4 ≡<br />

0(mod 5 ) , zatem K ≡ 0(mod 5 ) .<br />

5.4.8. Z MTF wynika, że a · a p−2 ≡ 1(mod p ) . Niech b<br />

be ↩<br />

dzie reszta ↩<br />

z dzielenia a p−2 przez p . Wtedy b ≡ a p−2 (mod p ) ,<br />

sta ↩<br />

d ba ≡ a p−1 ≡ 1(mod p ) . Wykazaliśmy, że istnieje takie b .<br />

Gdyby istnia̷ly b 1 i b 2 takie, że ab 1 ≡ 1(mod p ) oraz ab 2 ≡<br />

1(mod p ) , przy czym 1 ≤ b 1 ≤ b 2 ≤ p − 1 , to odejmuja ↩<br />

c stronami<br />

powyższe kongruencje otrzymalibyśmy a(b 2 − b 1 ) ≡ 0(mod p ) .<br />

Ponieważ NWD(a, p) = 1 , wie ↩<br />

c p|b 2 − b 1 . Sta ↩<br />

d wobec 1 ≤ b 2 − b 1 ≤<br />

p − 1 mamy b 1 = b 2 .<br />

5.4.9. (i) ⇒ (ii) . Jeżeli istnieje liczba n taka, że n 2 −n+3 ≡<br />

0(mod p ) , to k̷lada ↩<br />

c m = 3n−1 , mamy m 2 −m+25 ≡ 0(mod p ) .<br />

(ii) ⇒ (i) . Za̷lóżmy teraz, że istnieje liczba m , taka że m 2 −<br />

m + 25 ≡ 0(mod p ) . Z 5.4.8 wynika, że istnieje liczba ca̷lkowita b ,<br />

taka że 3b ≡ 1(mod p ) . K̷lada ↩<br />

c n = b(m + 1) , mamy<br />

n 2 − n + 3 = b 2 (m + 1) 2 − b(m + 1) + 3<br />

= b 2 (m 2 + 2m + 1) − b(m + 1) + 3<br />

= b 2 (m 2 + 2m + 1) + 24b 2 − 24b 2 − 3mb 2<br />

+ 3mb 2 − mb − b + 3<br />

= b 2 (m 2 − m + 25) − 24b 2 + 3mb 2 − mb − b + 3.


Kongruencje 105<br />

Z kongruencji 3b ≡ 1(mod p ) wynika, że 3mb 2 ≡ mb(mod p )<br />

oraz 24b 2 ≡ 8b(mod p ) , a także 9b ≡ 3(mod p ) . Uwzgle ↩<br />

dniaja ↩<br />

c<br />

powyższe kongruencje oraz za̷lożenie, mamy<br />

n 2 − n + 3 ≡ b 2 (m 2 − m + 25) − 8b + mb − mb − b + 9b ≡ 0(mod p ).<br />

5.4.10. Przypuśćmy, że liczb pierwszych postaci 4k + 1 jest<br />

tylko skończona ilość. Niech be ↩<br />

da ↩<br />

to liczby p 1 , p 2 , . . . , p n .<br />

Rozważmy liczbe ↩<br />

K = 2p 1 p 2 . . . p n , a naste ↩<br />

pnie liczbe ↩<br />

K 2 + 1 .<br />

Liczba K 2 + 1 jest postaci 4t + 1 . Niech p be ↩<br />

dzie liczba ↩<br />

pierwsza ↩<br />

dziela ↩<br />

ca ↩<br />

liczbe ↩<br />

K 2 + 1 . Wtedy<br />

K 2 ≡ −1(mod p ). (*)<br />

Liczba p nie może być równa 2, ponieważ K 2 + 1 jest liczba ↩<br />

nieparzysta ↩<br />

. Liczba p nie jest też postaci 4t + 1 , gdyż nie jest<br />

żadna ↩<br />

z liczb p 1 , p 2 , . . . , p n (żadna z liczb p 1 , p 2 , . . . , p n nie<br />

jest dzielnikiem K 2 + 1 ). Zatem p jest postaci 4t + 3 . Podnosza ↩<br />

c<br />

p−1<br />

kongruencje ↩<br />

(*) do pote ↩<br />

gi<br />

2<br />

= 2t + 1 , mamy<br />

(K 2 ) p−1<br />

2 ≡ (−1) p−1<br />

2 ≡ (−1) 2t+1 (mod p ),<br />

czyli K p−1 ≡ −1(mod p ) . Ponieważ p̸ | K , wie ↩<br />

c z MTF mamy<br />

K p−1 ≡ 1(mod p ) . Sta ↩<br />

d 1 ≡ −1(mod p ) , czyli p|2 . Otrzymaliśmy<br />

sprzeczność, która dowodzi, że liczb pierwszych postaci<br />

4k + 1 jest nieskończenie wiele.<br />

5.5. Pewne zastosowania twierdzenia Eulera<br />

5.5.1. Ponieważ n = qϕ(m) + r , wie ↩<br />

c a n = a qϕ(m)+r =<br />

(<br />

a<br />

ϕ(m) ) q<br />

a r ≡ a r (mod m ) .<br />

5.5.2. Ponieważ a k ≡ 1(mod m ) oraz a l ≡ 1(mod m ) ,<br />

wie ↩<br />

c z definicji wyk̷ladnika mamy w m (a)|k oraz w m (a)|l , co oznacza,<br />

że w m (a) jest wspólnym dzielnikiem liczb k i l . Wobec tego<br />

w m (a)|NWD(k, l) , czyli NWD(k, l) = w m (a)s . Otrzymujemy zatem<br />

a NWD(k,l) = a w m(a)s = ( a w m(a) ) s<br />

≡ 1(mod m ) .


106 Cze ↩<br />

ść II – Rozwia ↩<br />

zania<br />

5.5.3.<br />

(a) Ponieważ 317 ≡ 2(mod 15 ) , wie ↩<br />

c podnosza ↩<br />

c obie strony<br />

tej kongruencji do pote ↩<br />

gi 259 otrzymujemy 317 259 ≡<br />

2 259 (mod 15 ) . Z twierdzenia Eulera wynika, że 2 ϕ(15) ≡<br />

1(mod 15 ) , czyli 2 8 ≡ 1(mod 15 ) . Ponieważ 259 =<br />

32 · 8 + 3 , wie ↩<br />

c 2 259 = ( 2 8) 32<br />

· 2 3 ≡ 8(mod 15 ) . Zatem<br />

reszta z dzielenia 317 259 przez 15 wynosi 8 .<br />

(b) Reszta wynosi 7 .<br />

(c) Reszta wynosi 64 . Istotnie, ponieważ 4 22 ≡ 1(mod 23 ) ,<br />

wie ↩<br />

c 4 110 ≡ 1(mod 23 ) , a sta ↩<br />

d 4 112 ≡ 16(mod 23 ) .<br />

Zatem 4 113 ≡ 64(mod 92 ) .<br />

(d) Ponieważ 77 = 7 · 11 , ϕ(7) = 6 , ϕ(11) = 10 oraz<br />

NWW(6, 10) = 30 , wie ↩<br />

c z 5.1.5.(c) otrzymujemy 2 30 ≡<br />

1(mod 77 ) . Ponieważ 1000000 = 30 · 33333 + 10 , wie ↩<br />

c<br />

2 1000000 ≡ 2 10 ≡ 23(mod 77 ) .<br />

5.5.4. Ponieważ n − 1 = 2700 = 36 · 75 , wie ↩<br />

c wystarczy<br />

pokazać, że 2 36·75 ≡ 1(mod (37 · 73) ) . Z kongruencji 2 9 ≡<br />

2 6 · 2 3 ≡ −9 · 8 ≡ 1(mod 73 ) wynika, że 2 36 ≡ 1(mod 73 ) . Z<br />

twierdzenia Eulera wynika, że 2 36 ≡ 1(mod 37 ) . Z dwóch ostatnich<br />

kongruencji wynika, że 2 36 ≡ 1(mod (37 · 73) ) . Sta ↩<br />

d ostatecznie<br />

2 36·75 ≡ 1(mod (37 · 73) ) .<br />

5.5.5. Ponieważ 20 = 5 · 4 , wie ↩<br />

c wystarczy pokazać, że rozważana<br />

liczba dzieli sie ↩<br />

przez 5 oraz 4 . Dla wykazania podzielności<br />

przez 5 , wystarczy rozważyć przypadek, gdy NWD(n, 5) = 1 .<br />

W tym w̷laśnie przypadku mamy n 4 ≡ 1(mod 5 ) . Sta ↩<br />

d n 7 −<br />

14n 5 + 49n 3 − 36n ≡ n 3 + n − n 3 − n ≡ 0(mod 5 ) . Aby wykazać<br />

podzielność naszej liczby przez 4 , należy rozważyć dwa przypadki<br />

NWD(n, 4) = 1 oraz NWD(n, 4) = 2 . Jeżeli NWD(n, 4) = 1 , to<br />

n 2 ≡ 1(mod 4 ) . Wtedy n 7 − 14n 5 + 49n 3 − 36n ≡ n − 14n +<br />

49n − 36n ≡ 0(mod 4 ) . Jeżeli NWD(n, 4) = 2 , to n = 2k . Wtedy<br />

n(n 6 − 14n 4 + 49n 2 − 36) ≡ 0(mod 4 ) .<br />

5.5.6. Z wniosku z twierdzenia Eulera wynika, że najmniejsza<br />

liczba naturalna k taka, że a k ≡ 1(mod 7 ) znajduje sie ↩<br />

wśród<br />

dzielników liczby ϕ(7) = 6 . ̷Latwo sprawdzić, że jest nia ↩<br />

3, czyli<br />

2 3 ≡ 1(mod 7 ) . Sta ↩<br />

d 2 3k ≡ 1(mod 7 ) , gdzie k ∈ N .


Kongruencje 107<br />

5.5.7. W myśl twierdzenia Eulera, dla naturalnych k zachodzi<br />

kongruencja 2 kϕ(m) ≡ 1(mod m ) , ska ↩<br />

d 2 kϕ(m) = my k +1 , gdzie y k<br />

jest liczba ↩<br />

naturalna ↩<br />

; przyjmuja ↩<br />

c x k = kϕ(m) mamy 2 x k = my k + 1<br />

dla k ∈ N .<br />

5.5.8. Niech L oznacza iloczyn trzech kolejnych liczb naturalnych,<br />

z których środkowa jest sześcianem liczby naturalnej a > 1 ,<br />

czyli L = (a 3 −1)a 3 (a 3 +1) . Ponieważ 504 = 7·8·9 , wie ↩<br />

c wystarczy<br />

pokazać, że L ≡ 0(mod 7 ) , L ≡ 0(mod 8 ) oraz L ≡ 0(mod 9 ) .<br />

Jeżeli 7|a , to 7|L . Jeśli 7̸ | a , to z MTF wynika, że a 6 ≡<br />

1(mod 7 ) , czyli 7|(a 3 − 1)(a 3 + 1) . Zatem 7|L .<br />

Jeżeli 2|a , to 8|a 3 , wie ↩<br />

c 8|L . Jeśli 2̸ | a , to a ≡ 1(mod 2 ) ,<br />

sta ↩<br />

d a 3 ≡ 1(mod 2 ) , a także a 3 ≡ −1(mod 2 ) . Ponieważ a 3 − 1<br />

oraz a 3 +1 sa ↩<br />

kolejnymi liczbami parzystymi, wie ↩<br />

c jedna z nich jest<br />

podzielna przez 4. Istotnie, jeśli a 3 − 1 = 2t , to a 3 + 1 = 2t + 2 =<br />

2(t + 1) . Jeżeli t = 2s , to 4|a 3 − 1 . W przeciwnym wypadku, tj.<br />

gdy t = 2s + 1 , to a 3 + 1 = 4(s + 1) . Zatem 8|(a 3 − 1)(a 3 + 1) ,<br />

czyli 8|L .<br />

Jeżeli 3|a , to 9|a 3 . Natomiast, gdy 3̸ | a , to wówczas 9̸ | a ,<br />

wie ↩<br />

c na podstawie twierdzenia Eulera mamy a ϕ(9) ≡ 1(mod 9 ) ,<br />

czyli a 6 ≡ 1(mod 9 ) . Sta ↩<br />

d 9|L .<br />

5.5.9. Niech m ∈ N be ↩<br />

dzie liczba ↩<br />

wie ↩<br />

ksza ↩<br />

od 1 i niech r 1 ,<br />

r 2 , . . . , r ϕ(m) be ↩<br />

dzie cia ↩<br />

giem wszystkich liczb naturalnych mniejszych<br />

od m i wzgle ↩<br />

dnie pierwszych z m . Niech a be ↩<br />

dzie dowolna ↩<br />

liczba ↩<br />

ca̷lkowita ↩<br />

, taka ↩<br />

że NWD(m, a) = 1 . Oznaczmy przez ρ k reszte ↩<br />

z dzielenia liczby ar k przez m . Wiadomo, że<br />

ρ k ≡ ar k (mod m ) (*)<br />

dla k ∈ {1,2,. . . ,ϕ(m)} . Zauważmy, że liczby ρ 1 , ρ 2 , . . . , ρ ϕ(m)<br />

różnia ↩<br />

sie ↩<br />

od liczb r 1 , r 2 , . . . , r ϕ(m) co najwyżej porza ↩<br />

dkiem.<br />

W tym celu zobaczmy, że liczby ρ 1 , ρ 2 , . . . , ρ ϕ(m) sa ↩<br />

różne<br />

i wzgle ↩<br />

dnie pierwsze z m . Gdyby NWD(ρ k , m) = d > 1 , to z (*)<br />

wynika̷loby, że d jest wspólnym dzielnikiem liczb ar k oraz m ,<br />

które sa ↩<br />

wzgle ↩<br />

dnie pierwsze.<br />

Przypuśćmy, że dla pewnych k , l , takich że 1 ≤ k ≤ l ≤<br />

ϕ(m) , zachodzi równość ρ k = ρ l . Wtedy ar k ≡ ar l (mod m ) , co<br />

oznacza, że m|a(r k − r l ) . Sta ↩<br />

d, wobec tego, że NWD(a, m) = 1 ,


108 Cze ↩<br />

ść II – Rozwia ↩<br />

zania<br />

mamy m|r k − r l . Ponieważ liczby naturalne r k , r l sa ↩<br />

mniejsze<br />

od m , wie ↩<br />

c r k = r l , co jest możliwe tylko wtedy, gdy k = l .<br />

Z (*) wynika, że<br />

ρ 1 ρ 2 . . . ρ ϕ(m) ≡ a ϕ(m) r 1 r 2 . . . r ϕ(m) (mod m ).<br />

Zatem, ponieważ ρ 1 ρ 2 . . . ρ ϕ(m) = r 1 r 2 . . . r ϕ(m) , wie ↩<br />

c<br />

m|(a ϕ(m) − 1)r 1 r 2 . . . r ϕ(m) .<br />

Ponieważ NWD(<br />

m, r1 r 2 . . . r ϕ(m)<br />

)<br />

= 1 , wie↩ c m|(a ϕ(m) − 1) , tzn.<br />

a ϕ(m) ≡ 1(mod m ) .<br />

5.6. Rozwinie ↩<br />

cie okresowe a kongruencje<br />

5.6.1. Z równości (*) wynika, że<br />

10a ≡ r 1 (mod b)<br />

10r 1 ≡ r 2 (mod b)<br />

. . . . . . . . . . . . . . . . . .<br />

10r k−1 ≡ r k (mod b).<br />

Mnoża ↩<br />

c stronami powyższe kongruencje mamy<br />

10 k ar 1 r 2 . . . r k−1 ≡ r 1 r 2 . . . r k−1 r k (mod b ).<br />

Ponieważ NWD(r 1 r 2 . . . r k−1 , b) = 1 , wie ↩<br />

c gdy k = w b (10) , tzn. gdy<br />

10 k ≡ 1(mod b ) otrzymujemy, że a ≡ r k (mod b ) . Uwzgle ↩<br />

dniaja ↩<br />

c<br />

to, że 0 < a < b oraz 0 < r k < b , mamy a = r k . Otrzymaliśmy<br />

wie ↩<br />

c reszte ↩<br />

równa ↩<br />

licznikowi u̷lamka a b<br />

. Od tego momentu reszty<br />

be ↩<br />

da ↩<br />

sie ↩<br />

powtarzać: r k+1 = r 1 , r k+2 = r 2 , . . . . Sta ↩<br />

d q k+1 = q 1 .<br />

5.6.2. Wynika z 5.6.1 oraz wniosku z twierdzenia Eulera.<br />

5.6.3. Rozważania prowadzone w tym podrozdziale pozostana ↩<br />

prawdziwe, gdy liczbe ↩<br />

10 zasta ↩<br />

pimy dowolna ↩<br />

liczba ↩<br />

naturalna ↩<br />

g ≥ 2 .<br />

Tak wie ↩<br />

c kiedy chcemy sie ↩<br />

dowiedzieć, ile cyfr w okresie ma rozwinie<br />

↩<br />

cie w systemie o podstawie 2 u̷lamek 1 5<br />

, szukamy najmniejszej<br />

liczby naturalnej spe̷lniaja ↩<br />

cej kongruencje ↩<br />

ta ↩<br />

jest x = 4 .<br />

2 x ≡ 1(mod 5 ) . Liczba ↩


5.7. Zastosowania twierdzenia Wilsona<br />

Kongruencje 109<br />

5.7.1. W myśl twierdzenia Wilsona mamy 22! + 1 ≡<br />

0(mod 23 ) . Ponieważ<br />

22! = 18! · 19 · 20 · 21 · 22 ≡ 18!(−4)(−3)(−2)(−1) ≡ 18! · 24<br />

≡ 18!(mod 23).<br />

Sta ↩<br />

d 18! + 1 ≡ 0(mod 23 ) .<br />

5.7.2. Ponieważ 719 jest liczba ↩<br />

pierwsza ↩<br />

, wie ↩<br />

c z twierdzenia<br />

Wilsona wynika, że 718! + 1 ≡ 0(mod 719 ) . Ponieważ 718! =<br />

712! · 713 · 714 · 715 · 716 · 717 · 718 oraz<br />

713 ≡ −6(mod 719 ) 714 ≡ −5(mod 719 ) 715 ≡ −4(mod 719 )<br />

716 ≡ −3(mod 719 ) 717 ≡ −2(mod 719 ) 718 ≡ −1(mod 719 ),<br />

wie ↩<br />

c 718! ≡ 712! · 6! ≡ 712!(mod 719 ) ska ↩<br />

d 712! + 1 ≡ 718! + 1 ≡<br />

0(mod 719 ) . Oznacza to, że 719|712! + 1 .<br />

5.7.3. Niech liczba q ∈ N be ↩<br />

dzie taka, że 1 ≤ q < n − 1 oraz<br />

q|n . Wtedy q|(n − 1)! oraz q|(n − 1)! + 1 (bo n|(n − 1)! + 1 oraz<br />

q|n ). Zatem q|(n − 1)! + 1 − (n − 1)! , wie ↩<br />

c q = 1 . Sta ↩<br />

d wynika, że<br />

n jest liczba ↩<br />

pierwsza ↩<br />

.<br />

5.7.4. Jeżeli liczba p > 1 jest liczba ↩<br />

pierwsza ↩<br />

, to z twierdzenia<br />

Wilsona mamy (p − 1)! ≡ −1(mod p ) . Ponieważ p(p − 2)! ≡<br />

0(mod p ) , wie ↩<br />

c odejmuja ↩<br />

c ostatnie dwie kongruencje stronami<br />

mamy p(p − 2)! − (p − 1)! ≡ 1(mod p ) , ska ↩<br />

d (p − 2)!(p − p + 1) ≡<br />

1(mod p ) , czyli (p−2)! ≡ 1(mod p ) . Odwrotnie, jeżeli (p−2)! ≡<br />

1(mod p ) dla p > 1 , to (p − 2)!(p − 1) ≡ p − 1 ≡ −1(mod p ) .<br />

Sta ↩<br />

d i z 5.7.3 wynika, że p jest liczba ↩<br />

pierwsza ↩<br />

.<br />

5.7.5. W myśl twierdzenia Wilsona, dla wszystkich liczb n =<br />

p − 1 , gdzie p jest liczba ↩<br />

pierwsza ↩<br />

wie ↩<br />

ksza ↩<br />

od 3, mamy n! + 1 ≡<br />

0(mod n + 1 ) i n! + 1 > n + 1 . Zatem (p − 1)! + 1 jest liczba ↩<br />

z̷lożona ↩<br />

dla dowolnej liczby pierwszej p > 3 .<br />

5.7.6. Jeżeli n = p 2 , gdzie p ≥ 3 jest liczba ↩<br />

pierwsza ↩<br />

, to<br />

w iloczynie 1·2·3 · · · (p 2 −1) znajduja ↩<br />

sie ↩<br />

liczby p oraz 2p , ponieważ<br />

p 2 −1 > 2p . Zatem p 2 |(p 2 −1)! . Jeżeli n = ab , gdzie a ≥ 3 i b ≥ 3<br />

oraz a ≠ b , to w iloczynie 1 · 2 · 3 · · · (n − 1) wyste ↩<br />

puja ↩<br />

czynniki a<br />

i b , wie ↩<br />

c (n − 1)! ≡ 0(mod n ) .


110 Cze ↩<br />

ść II – Rozwia ↩<br />

zania<br />

5.7.7. Jeżeli n = p , gdzie p jest liczba ↩<br />

pierwsza ↩<br />

nieparzysta ↩<br />

,<br />

to z twierdzenia Wilsona wynika, że n̸ | (n − 1)! , wie ↩<br />

c n 2̸ | (n − 1)! ,<br />

czyli (n − 1)! ≢ 0(modn 2 ) . Jeżeli n = p 2 , gdzie p ≥ 5 jest liczba ↩<br />

pierwsza ↩<br />

, to w iloczynie 1·2·3 · · · (p 2 −1) znajduja ↩<br />

sie ↩<br />

liczby p , 2p ,<br />

3p oraz 4p , gdyż p 2 − 1 > 4p , wie ↩<br />

c p 4 |(p 2 − 1)! , czyli (n − 1)! ≡<br />

0(mod n 2 ) .<br />

Jeżeli n = 9 , to (9 − 1)! ≢ 0(mod9 2 ) , co można ̷latwo<br />

sprawdzić. Jeśli n = ab oraz a ≥ 3 i b ≥ 3 oraz a ≠ b , to w<br />

iloczynie 1 · 2 · 3 · · · (n − 1) wyste ↩<br />

puja ↩<br />

czynniki a , 2a , b i 2b ,<br />

wie ↩<br />

c (n − 1)! ≡ 0(mod n 2 ) . Zatem liczby nieparzyste pierwsze<br />

oraz liczba 9 spe̷lniaja ↩<br />

warunek (n − 1)! ≢ 0(modn 2 ) .<br />

5.8. Jeszcze jedno twierdzenie o kongruencjach<br />

5.8.1. W dowodzie chińskiego twierdzenia o resztach k̷ladziemy<br />

n 1 = 3 , n 2 = 4 , n 3 = 5, n = n 1 n 2 n 3 = 60 ,<br />

m 1 = 20 , m 2 = 15 , m 3 = 12,<br />

a 1 = a , a 2 = b , a 3 = c.<br />

Ponieważ 2m 1 +(−13)n 1 = 1 , 3m 2 +(−11)n 2 = 1 , 3m 3 +(−7)n 3 =<br />

1 , wie ↩<br />

c za x 0 należy przyja ↩<br />

ć 2·20a+3·15b+3·12c = 40a+45b+36c ,<br />

a za x reszte ↩<br />

z dzielenia liczby x 0 przez 60.<br />

5.8.2. Ponieważ NWD(11, 4) = 1 , wie ↩<br />

c z chińskiego twierdzenia<br />

o resztach wynika, że w przedziale [1,44] istnieje dok̷ladnie jedna<br />

taka liczba (jest nia ↩<br />

23). Kolejne liczby spe̷lniaja ↩<br />

ce wymagane<br />

warunki przystaja ↩<br />

modulo 44. Zatem takich liczb jest 45.<br />

5.8.3. Liczba 4 ma ża ↩<br />

dana ↩<br />

w̷lasność, ale nie jest liczba ↩<br />

trzycyfrowa ↩<br />

. Z chińskiego twierdzenia o resztach wynika, że każda<br />

inna liczba daja ↩<br />

ca wymagane reszty musi sie ↩<br />

różnić od 4 o wielokrotność<br />

liczby 7 · 9 · 11 = 693 . Jedyna ↩<br />

taka ↩<br />

liczba ↩<br />

trzycyfrowa ↩<br />

jest<br />

4 + 693 = 697 .<br />

5.8.4. Liczba −10 ma ża ↩<br />

dane w̷lasności, ale nie jest liczba ↩<br />

dodatnia ↩<br />

. Z chińskiego twierdzenia o resztach wynika, że kolejna ↩<br />

liczba ↩<br />

maja ↩<br />

ca ↩<br />

ża ↩<br />

dane w̷lasności jest liczba −10 + 11 · 12 · 13 = 1706 .<br />

Jest to najmniejsza liczba dodatnia, która daje reszte ↩<br />

1 przy dzieleniu<br />

przez 11, reszte ↩<br />

2 przy dzieleniu przez 12 i reszte ↩<br />

3 przy dzieleniu<br />

przez 13.


Kongruencje 111<br />

5.8.5. Dla dowolnego naturalnego n zachodzi 2 n + 2 n =<br />

2 n+1 . Z chińskiego twierdzenia o resztach wynika, że istnieje liczba<br />

naturalna n 0 taka, że n 0 ≡ 0(mod p ) , n 0 ≡ 0(mod q ) , n 0 ≡<br />

−1(mod r ) . Wtedy<br />

( )<br />

n 0 p ( )<br />

n 0 q ( )<br />

n 0 +1 r<br />

2<br />

p<br />

+ 2<br />

q r<br />

= 2 .<br />

Przyjmuja ↩<br />

c x 0 = 2 n 0<br />

p<br />

, y 0 = 2 n 0<br />

q<br />

, z 0 = 2 n 0 +1<br />

r<br />

, mamy<br />

x p 0 + yq 0 = zr 0.<br />

5.8.6. Jeżeli n = 1 , to dla dowolnego c ∈ N para (c a 2<br />

,c a 1<br />

)<br />

jest rozwia ↩<br />

zaniem równania x a 1<br />

1 = x a 2<br />

2 . Niech n > 1 . Na mocy<br />

chińskiego twierdzenia o resztach istnieje liczba naturalna z ∈ [1,a 1·<br />

a 2 · · · a n+1 ] , taka że z ≡ 0(mod a i ) dla i ∈ {1,2,. . . ,n} oraz z ≡<br />

−1(mod a n+1 ) (takich liczb naturalnych z istnieje oczywiście nieskończenie<br />

wiele). Istnieja ↩<br />

wie ↩<br />

c liczby naturalne y 1 , y 2 , . . . , y n ,<br />

y n+1 takie, że z = a i y i dla i ∈ {1,2,. . . ,n} , z + 1 = a n+1 y n+1 .<br />

K̷lada ↩<br />

c x i = n y i<br />

dla i ∈ {1,2,. . . ,n,n + 1} mamy<br />

x a 1<br />

1 + xa 2<br />

2 + · · · + xa n<br />

n<br />

= (n y 1<br />

) a 1<br />

+ (n y 2<br />

) a 2<br />

+ · · · + (n y n<br />

) a n<br />

= n z + n z + · · · + n z = n · n z = n z+1<br />

= (n y n+1<br />

) a n+1<br />

= x a n+1<br />

n+1 ,<br />

co oznacza, że (n y 1<br />

,n y 2<br />

,. . . ,n y n+1<br />

) jest rozwia ↩<br />

zaniem naszego równania.<br />

Dla każdej liczby z (których jest nieskończenie wiele) można<br />

skonstruować inne rozwia ↩<br />

zanie.


BIBLIOGRAFIA<br />

[1] Browkin J.: Zbiór zadań z olimpiad matematycznych. T. 5. WSiP, Warszawa<br />

1980.<br />

[2] Browkin J.: Zbiór zadań z olimpiad matematycznych. T. 6. WSiP, Warszawa<br />

1983.<br />

[3] Browkin J., Remba̷la J., Straszewicz S.: 25 lat olimpiady matematycznej.<br />

WSiP, Warszawa 1975.<br />

[4] Ifrah G.: Dzieje liczby, czyli historia wielkiego wynalazku. Zak̷lad Narodowy<br />

im. Ossolińskich – Wydawnictwo, Wroc̷law–Warszawa–Kraków–Gdańsk–<br />

̷Lódź 1990.<br />

[5] Koblitz N.: Wyk̷lad z teorii liczb i kryptografii. Wydawnictwa Naukowo-<br />

-Techniczne, Warszawa 1995.<br />

[6] Kumanduri R., Romero C.: Number Theory With Computer Applications.<br />

Prentice Hall, London 1998.<br />

[7] Mazur M.: Metoda kongruencji w teorii liczb. Matematyka, Spo̷leczeństwo,<br />

Nauczanie 1993, nr 10.<br />

[8] Narkiewicz W.: Teoria liczb. Biblioteka Matematyczna. T. 50. PWN, Warszawa<br />

1990.<br />

[9] Schroeder M.R.: Number Theory in Science and Communication. Springer-<br />

-Verlag, Berlin–Heidelberg 1997.<br />

[10] Sierpiński W.: Arytmetyka teoretyczna. Biblioteka Matematyczna. T. 7.<br />

PWN, Warszawa 1959.<br />

[11] Sprawozdanie Komitetu G̷lównego: IV Olimpiada Matematyczna. PZWS,<br />

Warszawa 1954.<br />

[12] Sprawozdanie Komitetu G̷lównego: V Olimpiada Matematyczna. PZWS, Warszawa<br />

1955.


Bibliografia 113<br />

[13] Sprawozdanie Komitetu G̷lównego: XIII Olimpiada Matematyczna. PZWS,<br />

Warszawa 1963.<br />

[14] Sprawozdanie Komitetu G̷lównego: XIV Olimpiada Matematyczna. PZWS,<br />

Warszawa 1964.<br />

[15] Sprawozdanie Komitetu G̷lównego: XIX Olimpiada Matematyczna. PZWS,<br />

Warszawa 1969.<br />

[16] Sprawozdanie Komitetu G̷lównego: XLI Olimpiada Matematyczna. WSiP,<br />

Warszawa 1993.<br />

[17] Sprawozdanie Komitetu G̷lównego: XV Olimpiada Matematyczna. PZWS,<br />

Warszawa 1965.<br />

[18] Sprawozdanie Komitetu G̷lównego: XVI Olimpiada Matematyczna. PZWS,<br />

Warszawa 1966.<br />

[19] Sprawozdanie Komitetu G̷lównego: XVII Olimpiada Matematyczna. PZWS,<br />

Warszawa 1967.<br />

[20] Sprawozdanie Komitetu G̷lównego: XXI Olimpiada Matematyczna. PZWS,<br />

Warszawa 1971.<br />

[21] Sprawozdanie Komitetu G̷lównego: XXII Olimpiada Matematyczna. PZWS,<br />

Warszawa 1972.<br />

[22] Sprawozdanie Komitetu G̷lównego: XXIV Olimpiada Matematyczna. WSiP,<br />

Warszawa 1974.<br />

[23] Sprawozdanie Komitetu G̷lównego: XXV Olimpiada Matematyczna. WSiP,<br />

Warszawa 1975.<br />

[24] Sprawozdanie Komitetu G̷lównego: XXX Olimpiada Matematyczna. WSiP,<br />

Warszawa 1981.<br />

[25] Sprawozdanie Komitetu G̷lównego: XXXI Olimpiada Matematyczna. WSiP,<br />

Warszawa 1982.<br />

[26] Sprawozdanie Komitetu G̷lównego: XXXII Olimpiada Matematyczna. WSiP,<br />

Warszawa 1983.<br />

[27] Sprawozdanie Komitetu G̷lównego: XXXIII Olimpiada Matematyczna. WSiP,<br />

Warszawa 1984.<br />

[28] Sprawozdanie Komitetu G̷lównego: XXXIX Olimpiada Matematyczna. WSiP,<br />

Warszawa 1991.<br />

[29] Sprawozdanie Komitetu G̷lównego: XXXVI Olimpiada Matematyczna. WSiP,<br />

Warszawa 1988.<br />

[30] Sprawozdanie Komitetu G̷lównego: XXXVII Olimpiada Matematyczna.<br />

WSiP, Warszawa 1989.


114 Bibliografia<br />

[31] Sprawozdanie Komitetu G̷lównego: XXXVIII Olimpiada Matematyczna.<br />

WSiP, Warszawa 1990.<br />

[32] Straszewicz S.: Zadania z Drugiej Olimpiady Matematycznej. PZWS, Warszawa<br />

1952.<br />

[33] Straszewicz S.: Zadania z olimpiad matematycznych. T. 1. PZWS, Warszawa<br />

1956.<br />

[34] Straszewicz S.: Zadania z olimpiad matematycznych. T. 2. PZWS, Warszawa<br />

1961.<br />

[35] Straszewicz S.: Zadania z olimpiad matematycznych. T. 3. PZWS, Warszawa<br />

1966.<br />

[36] Straszewicz S.: Zadania z olimpiad matematycznych. T. 4. PZWS, Warszawa<br />

1972.<br />

[37] Straszewicz S.: Zadania z Pierwszej Olimpiady Matematycznej. PZWS, Warszawa<br />

1951.

Hooray! Your file is uploaded and ready to be published.

Saved successfully!

Ooh no, something went wrong!